TORT Flashcards

1
Q

A man has a heart attack at home and his wife calls for an ambulance. It takes the ambulance four hours to arrive as all local ambulances are dealing with a multiple car crash. The local Ambulance Service has recently suffered cuts in funding. By the time the ambulance arrives, the man is dead. Which of the following statements is most accurate regarding whether a duty of care was owed to this man by the ambulance service?
A duty of care was not owed as policy reasons justify the delay.
A duty of care was owed given the severity of his illness.
A duty of care was owed to arrive within a reasonable time.
A duty of care was owed to arrive as soon as possible given the proximate relationship between the man and ambulance service.
A duty of care was owed as operational failures have led to the delay.

A

A duty of care was owed to arrive within a reasonable time.
Correct:There is clear precedent that on acceptance of a 999 call by the ambulance service, they owe a duty of care to respond within a reasonable time (Kent v Griffiths & Others[2000] 2 All ER 474).
While the other options might sound plausible, they are each incorrect.
A duty of care will always be owed on acceptance of the 999 call by the ambulance service (Kent v Griffiths & Others[2000] 2 All ER 474). Whether policy reasons (cuts in funding) are the cause of the delay is an argument relevant to breach of duty.
There is established precedent for the ambulance service owing a duty of care on acceptance of the 999 call (Kent v Griffiths & Others[2000] 2 All ER 474), so the courts would not have to discuss whether there was a proximate relationship between the claimant and defendant.
A duty of care will always be owed on acceptance of the 999 call by the ambulance service (Kent v Griffiths & Others[2000] 2 All ER 474). Whether operational failures are the cause of the delay is an argument relevant to breach of duty. For example, should they have sent one of the ambulances at the car crash to the man having a heart attack?
A duty of care will always be owed on acceptance of the 999 call by the ambulance service (Kent v Griffiths & Others[2000] 2 All ER 474), regardless of the severity of the illness. Decisions made regarding who to prioritise would be an argument relevant to breach of duty.

How well did you know this?
1
Not at all
2
3
4
5
Perfectly
2
Q

A woman is in a supermarket when she collapses from a heart attack. The store manager puts a call out over the tannoy system asking if there is a doctor present. A customer hears the announcement and being a nurse comes forward and tries to help. The nurse fails to put the woman in the recovery position and the woman later dies.

Which of the following statements is the court most likely to agree with in relation to duty of care?

The store manager owed the woman a duty of care as he took steps that suggested he had assumed responsibility for her welfare. The nurse owed a duty of care once she started to help.

The store manager did not owe the woman a duty of care as he omitted to help her. The nurse owed a duty of care once she started to help.

The store manager owed the woman a duty of care as he was in a position of responsibility. The nurse owed a duty of care once she started to help.

The store manager did not owe a duty as he could not foresee the harm. The nurse owed the woman a duty of care as she assumed responsibility for her welfare.

The store manager did not owe the woman a duty of care as they were strangers. The nurse, as a qualified medical professional, owed a duty to assist.

A

The store manager owed the woman a duty of care as he took steps that suggested he had assumed responsibility for her welfare. The nurse owed a duty of care once she started to help.

Correct. Had the store manager done nothing then no duty of care would be owed. However, as he made an announcement over the tannoy he has arguably taken steps that suggest he has assumed responsibility for the woman’s welfare (Barrett v Ministry of Defence [1995] 1 WLR 1217). The nurse owed a duty of care once she started to assist (Barrett v Ministry of Defence).

While the other options might sound plausible, they are each incorrect.

Even if the claimant and defendant are complete strangers, a duty of care can be owed where the defendant assumes responsibility for the claimant’s welfare. The fact the nurse was a qualified medical professional does not mean she owed a duty to assist. The duty only arose once she started to assist.

The store manager’s position at work did not mean he owed a duty.

A reasonable person would be able to foresee personal injury if a woman collapsed.

The store manager has not omitted to help her. He has taken steps that suggest he has assumed responsibility for her welfare.

How well did you know this?
1
Not at all
2
3
4
5
Perfectly
3
Q

Two friends are in a sailboat. The boat begins to sink and the two friends shout ‘help’ repeatedly to Friend A who is watching from the shore. Friend A hears them but does nothing to help them as he cannot swim.

Which of the following statements best describes whether a duty of care was owed by friend A to the two friends?

Friend A owed a duty of care to assist once he heard the two friends shouting for help.

Friend A owed a duty of care as he/she could foresee the risk of personal injury.

Friend A owed a duty of care to assist as Friend A had assumed responsibility for the welfare of the two friends.

Friend A did not owe a duty of care as there was nothing he/she reasonably could have done to help given he/she could not swim.

Friend A did not owe a duty of care, his failure was one of omission.

A

Friend A did not owe a duty of care, his failure was one of omission.

Correct
Correct. As a general rule, the law of tort does not impose a duty of care for a mere failure to act (Smith v Littlewoods Organisation Ltd [1987] AC 241). None of the exceptions to this general rule apply.

While the other options might sound plausible, they are each incorrect.

It does not matter that Friend A heard the two friends shouting help. Friend A would only owe a duty of care once he started to assist and had assumed responsibility for their welfare.

The fact pattern does not suggest that Friend A assumed responsibility for the two friends – he/she omitted to do anything.

Friend A may well have foreseen the risk of personal injury if he or she did nothing. However, a duty of care would not be imposed on this basis alone. There would have to be some kind of ‘proximity’ between the parties.

The fact that Friend A could not swim is irrelevant. Friend A was not obliged to help his/her friends even if he/she could swim.

How well did you know this?
1
Not at all
2
3
4
5
Perfectly
4
Q

The door to an off licence is left unlocked one night by a cleaner. Some youths enter, consume large quantities of alcohol and cause lots of damage. When leaving the off licence, the youths vandalise the expensive garden ornaments belonging to the neighbour of the off licence. This has never happened before.

The cleaner owes the off licence a duty of care as he/she created the dangerous situation.

The cleaner owes the off licence a duty of care as the damage to the off licence was reasonably foreseeable.

The cleaner does not owe the off licence a duty of care as this would be unfair, unjust and unreasonable.

The cleaner owes the off licence a duty of care due to their contractual relationship.

The cleaner does not owe the off licence a duty of care as she is unlikely to be able to pay any damages.

The off licence does not owe the neighbour a duty of care as they could not foresee the damage caused by the vandals.

A

The cleaner owes the off licence a duty of care due to their contractual relationship.

The off licence does not owe the neighbour a duty of care as they could not foresee the damage caused by the vandals.

Correct. The cleaner has omitted to lock the door. As a general rule in the law of tort, no duty of care is owed for an omission (Smith v Littlewoods Organisation Ltd [1987] AC 241). However, this case falls within one of the exceptions to the general rule and Stansbie v Troman [1948] 2 KB 48can be relied on as precedent. The cleaner has failed to perform a quasi/contractual obligation.

The off licence would not owe the neighbour a duty of care because they could not foresee the property damage caused by the vandals. The facts are comparable to Smith v Littlewoods Organisation Ltd [1987] AC 241. Geographic proximity will not be enough to create a duty of care between the off licence and neighbour. There needs to be a closer relationship (see exceptions) for a duty of care to be imposed for a failure to prevent a third-party cause harm.

While the other options might sound plausible, they are each incorrect.

The damage to the off licence was reasonably foreseeable to the cleaner. However, the courts would not have to consider this as there is precedent they can rely on in relation to the cleaner owing the off licence a duty of care.

Geographic proximity will not be enough to create a duty of care between the off licence and neighbour. There needs to be a closer relationship (see exceptions) for a duty of care to be imposed for a failure to prevent a third-party cause harm.

The courts have precedent to rely on in relation to the cleaner owing the off licence a duty of care, so they would not have to consider whether it was fair, just and reasonable to impose a duty. If they did consider policy, arguably it would be fair to impose a duty on the cleaner. See above regarding geographic proximity.

The off licence probably does not owe a duty of care, partly, because they would not have foreseen the damage caused by the youths (Smith v Littlewoods Organisation Ltd [1987] AC 241). However, given there is precedent, it is irrelevant that the cleaner would be unable to pay damages. The cleaner would owe a duty of care.

The cleaner might owe a duty based on the fact that they created the danger by leaving the door unlocked, Stansbie v Troman [1948] 2 KB 48. However, the off licence does not owe the neighbour a duty of care simply because they were responsible for the cleaner. This does not fall into any of the exceptions for a failure to prevent a third-party causing harm.

How well did you know this?
1
Not at all
2
3
4
5
Perfectly
5
Q

A police informant is claiming damages for negligence by the police. It is alleged that the police negligently left the informants contact details unattended in a police car which was stolen, and eventually reached the criminal against whom the informer had given evidence. The criminal broke into the informers’ house and physically assaulted them.

Which of the following statements best describes the principal reason why the police might owe the informer a duty of care?

A duty of care is owed because it is fair, just and reasonable.

A duty of care is owed because the police assumed responsibility for the informant’s welfare.

A duty of care is owed because the informant is an identifiable victim.

A duty of care is owed because the police created the source of danger.

A duty of care is owed because the informant has suffered a personal injury.

A

A duty of care is owed because the police assumed responsibility for the informant’s welfare.

Correct
Correct: This answer best reflects why the Court of Appeal imposed a duty of care in the case of Swinney v Chief Constable of Northumbria (No.2), The Times, 25 May 1999 (same facts as the fact pattern). The police assume responsibility to protect informers against the criminals they give evidence about. Informers should not be considered like other members of the public; they have a special relationship with the police.

While the other answer options might sound plausible, they are each incorrect.

It would be fair, just and reasonable to impose a duty of care but this would not be the principal reason for imposing a duty (rather it would support that conclusion). The focus would be on whether the police had assumed responsibility for the claimant’s welfare (proximity).

Whilst being an identifiable victim certainly supports the argument for imposing a duty of care, recent case law suggests that not only must the claimant be identifiable, but the police must also have clearly assumed responsibility for the claimant’s welfare (Mitchell v Glasgow City Council [2009] 1 AC 874 , CN and GN v Poole Borough Council [2019] UKSC 25).

Whilst creating the source of danger could support the argument for imposing a duty of care, this argument was not emphasised in Swinney and focus seems to be on assumption of responsibility.

It is likely that in this scenario a duty of care would be owed for personal injury, but not for any property damage or economic loss. However, the fact the informer has suffered personal injury is not the reason why a duty would be owed.

How well did you know this?
1
Not at all
2
3
4
5
Perfectly
6
Q

A householder carries out some trivial repair work on her property, including the door handle on her front door. The handle has not been fixed properly. When the householders’ friend comes over, he pulls the handle and it comes off in his hand. He falls backwards and sprains his wrist.

Which of the following statements best describes the standard of care owed by the householder?

The standard of care expected of the householder would be that of the reasonable man.

The standard of care expected of the householder would be that of a reasonably competent carpenter.

The standard of care expected of the householder would be that of a reasonable homeowner.

The standard of care expected of the householder would be that of a reasonably competent professional carpenter.

The standard of care expected of the householder would be that of a reasonably competent amateur carpenter.

A

The standard of care expected of the householder would be that of a reasonably competent amateur carpenter.

Correct
Correct. The facts are very similar to the case of Wells v Cooper [1958] 2 QB 265 in which the defendant fixed his doorknob. When the claimant opened the door, the doorknob came away from the door and the claimant slipped, fell and injured himself. There were two standards to choose from, that of the reasonably competent professional carpenter and that of the reasonably competent amateur carpenter. The Court of Appeal held the defendant to the standard of the reasonably competent amateur carpenter, which was lower than the standard expected of a professional. The job in question was a trivial domestic replacement. Had the job been something more ambitious, the standard might have been different.

While the other answer options might sound plausible, they are each incorrect.

Given that the householder was carrying out trivial repair work, it would not be appropriate to hold her to the professional standard. It is unlikely that special skills and expertise were needed.

It is not incorrect to say that the householder would be held to the standard of the reasonable man, but it is more accurate to say that of the reasonably competent amateur carpenter. The standard of care attaches to the act (Wilsher v Essex Area Health Authority [1987] QB 730).

The same reasoning applies to the inaccuracy of a reasonable householder. This is too broad.

It is important to make it clear whether the standard is amateur or professional, as this will affect whether or not there has been a breach of duty. Less will be expected of the amateur carpenter.

How well did you know this?
1
Not at all
2
3
4
5
Perfectly
7
Q

A patient develops permanent spinal curvature after undergoing routine surgery to his back. The surgeon has used the ‘Slice and Dice’ technique which carries a 15% risk of spinal curvature. The newer ‘Plice’ technique, as very recently highlighted in ‘Cutting Edge Weekly’ medical magazine, carries only a 5% risk of spinal curvature. However, the surgeon was not aware of it.

Which of the following statements best describes breach of duty in relation to the surgeon?

It is reasonable to expect the surgeon to keep up to date and he therefore should have used the ‘Plice’ technique. It was recently featured in ‘Cutting Edge Weekly’ magazine and it carries a 10% lower risk of spinal curvature.

It is reasonable to expect the surgeon to keep up to date and informed about all innovations such as the ‘Plice’ technique, by whatever means necessary.

It is unreasonable to expect the surgeon to have been up to date about the ‘Plice’ technique, when the ‘Splice and Dice’ technique has had an 85% success rate so far.

It would be unreasonable to expect the surgeon to have known about and implemented the new ‘Plice’ technique whilst conducting the routine back surgery. This was only recently featured in Cutting Edge magazine; it is a weekly magazine and is not a mainstream medical journal.

It is reasonable to expect the surgeon to attempt to minimise all risk. As such the surgeon should have been aware and employed the ‘Plice’ technique as it was featured in ‘Cutting Edge Weekly’ magazine.

A

Correct. It would be unreasonable to expect the surgeon to know about every new medical development. The courts recognise that it would be too onerous to expect medics to have to keep up to date with all the very latest treatments and procedures pursuant to Crawford v Governors of Charing Cross Hospital, The Times, December 8, 1953. Nevertheless, they must take reasonable steps to ensure they are up to date with mainstream developments especially with the availability of information online (Gascoine v Ian Sheridan & Co and Latham [1994] LS Law Med 437). In addition, the name of the magazine suggests it is providing the very latest information, insights and innovations in the medical field rather than being a mainstream journal. The fact that it is a weekly journal means that it might be burdensome to expect a medic to keep up with the pace of a weekly journal.

While the other answer options might sound plausible, they are each incorrect.

Medics must take reasonable steps to ensure they are up to date with mainstream developments (Gascoine v Ian Sheridan & Co and Latham [1994] LS Law Med 437) but it is too onerous to expect medics to keep up to date with all the very latest treatments (Crawford v Governors of Charing Cros Hospital, The Times, December 8, 1953).

It would be unreasonable and too onerous to expect a medic to keep up to date with the very latest innovation and to have to implement this new ‘Plice’ technique, as proposed or highlighted in the weekly magazine, in particular, one labelled ‘Cutting Edge’ (Crawford v Governors of Charing Cross Hospital, The Times, December 8, 1953). So although it is correct to state that the surgeon does not have to keep up to date with the new ‘Plice’ technique, the reasoning i.e. that the other has a 85% success rate, is incorrect.

It is correct to state that surgeon should attempt to minimise all risk, however, it would be unreasonable and too onerous a burden to expect the surgeon to keep up to date with the very latest innovation and to have to implement this new ‘Plice’ technique, as proposed or highlighted in the weekly magazine, in particular, one labelled ‘Cutting Edge Weekly’ (Crawford v Governors of Charing Cross Hospital, The Times, December 8, 1953).

The surgeon should keep up to date with all mainstream developments per Gascoine v Ian Sheridan & Co and Latham [1994] LS Law Med 437); but to expect the surgeon to have implemented the latest ‘Plice’ technique, despite it carrying a lower risk, would be unreasonable and too onerous a burden (Crawford v Governors of Charing Cross Hospital, The Times, December 8, 1953). The comment that the surgeon should be up to date by whatever means necessary is not a legal requirement.

How well did you know this?
1
Not at all
2
3
4
5
Perfectly
8
Q

A footballer attends hospital seeking medical treatment for his injured knee. The doctor recommends surgery. The footballer is asked to sign a consent form before the procedure, which he does. The footballer does not ask, and is not told, of the risks of the surgery. In fact, there is a 1% risk of permanent nerve damage. The operation is carried out without error by the doctor. The footballer develops permanent nerve damage and can no longer play football.

Which of the following best explains the legal position in relation to the potential breach of duty by the doctor?

A doctor has a duty to inform a patient of any risk over 10%. As the risk of permanent nerve damage was only 1%, the doctor has not breached his duty of care.

As the footballer freely signed the consent form before the surgery, there can be no breach of duty for failure to advise on the 1% risk of nerve damage.

The doctor performed the surgery without error and so there is no breach of duty. Further the footballer did not ask about the risks of the surgery and there was therefore no duty on the doctor to inform him of such a small (1%) risk.

The footballer will have to show that a reasonable body of medical men carrying out this medical procedure would have informed him of the 1% risk of nerve damage. If this is established the doctor will have breached his duty of care.

The doctor had a duty to inform the footballer of all material risks of the surgery. The 1% risk may be small, but it would still be material to a footballer. The doctor is in breach of duty.

A

The doctor had a duty to inform the footballer of all material risks of the surgery. The 1% risk may be small, but it would still be material to a footballer. The doctor is in breach of duty.

Correct
Correct. A doctor owes a duty to inform a patient of all material risks associated with a procedure and of all reasonable alternatives (Montgomery v Lanarkshire Health Board [2015[ SC). The test of materiality applies and requires a doctor to consider what the particular patient would consider material and what a reasonable person in the patient’s position would consider material. In this case a footballer would clearly consider permanent nerve damage a material risk and he should have been informed of this.

While the other options might sound plausible, they are each incorrect, which should be self-explanatory following the feedback above on why this answer was correct. Montgomery v Lanarkshire Health Board [2015[ SC confirms that the Bolam test has no application to a failure to warn of the risks.

How well did you know this?
1
Not at all
2
3
4
5
Perfectly
9
Q

A receptionist at a car factory would occasionally walk pass other employees whilst they were working with spray paint. Those working with the spray paint were provided with masks as the employer was aware of the risk of lung cancer from prolonged exposure to the paint. The receptionist contracts lung cancer. Since the receptionist was diagnosed, medical science has concluded that even occasional exposure to the paint increases the risk of lung cancer.

Which of the following statements best describes breach of duty in relation to the employer?

A reasonable employer may not have been able to foresee the likelihood of harm, but given there was some knowledge of the risk of lung cancer and given the ease of precautions (provision of face masks for all employees exposed to the paint), the exposure was in breach of duty.

A reasonable employer would not have known that there was a danger to those who did not have prolonged exposure to the paint, and therefore failure to take precautions against such exposure was not a breach of duty.

A reasonable employer would have provided all employees with any level of exposure to the paint with masks, given the magnitude of potential harm and the ease of precautions, and is thus in breach of duty.

A reasonable employer would not have taken precautions against such exposure as the risk of lung cancer was so low. Such exposure was therefore not in breach of duty.

A reasonable employer would not have taken precautions (provision of face masks) as this was disproportionate to the likelihood of harm. The exposure was therefore not in breach of duty.

A

A reasonable employer would not have known that there was a danger to those who did not have prolonged exposure to the paint, and therefore failure to take precautions against such exposure was not a breach of duty.

Correct. The courts must assess the defendant’s actions against the knowledge in the industry at the time of the alleged breach (at the time the receptionist was exposed to the paint). This is known as the ‘state of art’ defence (Roe v Minister of Health [1954] 2 QB 66). The employers did not know about the risk of lung cancer from occasional exposure to the paint at the time of the breach and would not, therefore, be expected to take precautions against an unknown risk.

While the other answer options might sound plausible, they are each incorrect.

Whilst the courts will balance likelihood of harm against practicality of precautions when assessing breach (Wagon Mound (No. 2) [1967] 1 AC 617), there must be some foreseeability of harm in order for there to be a breach (Bolton v Stone [1951] AC 850) and there was none here.

As above, whilst the courts will balance magnitude of harm against practicality of precautions when assessing breach (Wagon Mound (No.2) [1967] 1 AC 617), the magnitude of harm must be foreseeable (Paris v Stepney Borough Council [1951] AC 367) in order for there to be a breach and it was not here.

The risk of lung cancer was not low. It was not known at all (in relation to occasional exposure).

Taking precautions against an unknown risk would be considered disproportionate (Latimer v AEC Ltd [1953] AC 643) but there was no foreseeable likelihood of harm.

How well did you know this?
1
Not at all
2
3
4
5
Perfectly
10
Q

A father has recently become reconciled with his daughter after a long period of estrangement. As a result, he makes an appointment with his experienced solicitor to amend his will to include a legacy of £100,000 to his daughter. He sees a trainee solicitor instead and instructs him accordingly. One month later, the father dies of a sudden heart attack. Due to pressure of work, the trainee solicitor had not amended the will.

Which of the following statements best describes the standard of care owed by the trainee solicitor?

The standard of care is that of the reasonable man as the trainee solicitor was not professing to be a qualified solicitor.

The standard of care is that of a reasonably competent trainee solicitor.

The standard of care is that of an experienced private client solicitor as an experienced solicitor should have been overseeing the trainee solicitor.

The standard of care is that of a reasonably competent solicitor.

The standard of care is that of the ordinary reasonable man exercising and professing to be a private client solicitor.

A

The standard of care is that of the ordinary reasonable man exercising and professing to be a private client solicitor.

Correct
Correct. The standard for a professional is the standard of the ordinary reasonable man exercising and professing to have that special skill, which on the facts is that of a qualified private client solicitor (Bolam v Friern Hospital Management Committee [1957] 1 WLR 582). It is irrelevant that the person conducting the work was a trainee solicitor as the standard of care is objective and attaches to the ‘act not the actor’ (Wilsher v Essex Area Health Authority [1987] QB 730).

While the other answer options might sound plausible, they are each incorrect.

As mentioned above, it is irrelevant that the person conducting the work was a trainee solicitor as the standard of care is objective and attaches to the ‘act not the actor’ (Wilsher v Essex Area Health Authority [1987] QB 730). Here the act was the act of a solicitor (drafting wills).

It is not incorrect to say that the standard would be that of a reasonably competent solicitor, but it is more accurate to say that of a reasonably competent private client solicitor. Solicitors specialise in particular areas of law and the courts will be concerned with what a reasonable body of private client solicitors would have done, in the same way that the courts differentiate between, for example, GPs and surgeons.

It does not matter that a more experienced solicitor should have been overseeing the trainee solicitor. It is the trainee solicitor that has potentially been negligent and he will be judged by the standard of a qualified solicitor (see above). In any event, level of experience is not really relevant to standard of care as the person need not possess the highest expert skill at the risk of being found negligent. It is sufficient if he exercises the ordinary skill of an ordinary competent man exercising that particular profession.

How well did you know this?
1
Not at all
2
3
4
5
Perfectly
11
Q

A patient attends A&E with a severe headache. A doctor briefly examines the patient and tells them to stay in the waiting room, where the patient has a seizure and dies. Medical evidence suggests that if the doctor had treated the patient properly, there was a 40 per cent chance of living.

Which of the following statements best describes whether the patient’s estate can claim damages from the doctor for the patient’s death?

The patient’s estate can claim damages from the doctor because the negligent failure to treat the patient materially increased the risk of the patient dying.

The patient’s estate cannot claim damages from the doctor because the seizure was a natural cause of death.

The patient’s estate cannot claim damages from the doctor because the ‘but for’ test is not satisfied.

The patient’s estate can claim damages from the doctor because the negligent failure to treat the patient materially contributed to the patient’s death.

The patient’s estate can claim damages from the doctor because the negligent failure to treat the patient reduced the claimant’s chances of surviving.

A

The patient’s estate cannot claim damages from the doctor because the ‘but for’ test is not satisfied.
Correct: But for the doctor’s breach (briefly examining the patient), the patient still would have died at that time and in that way on the balance of probabilities (Barnett v Chelsea and Kensington Hospital [1969] 1 QB 428). There was only a 40 per cent chance that the patient would have lived if the doctor had examined the patient properly i.e. there was a 60 per cent chance that the patient would have died even if the doctor had examined the patient properly and not been negligent.

While the other answer options might sound plausible, they are each incorrect.

Factual causation cannot be satisfied for personal injuries based on loss of chance (Hotson v East Berkshire Health Authority [1987] AC 750). Where the risk of damage without the breach is 50 per cent or more, the claimant will fail to establish factual causation.

The material contribution test does not apply here. This is not a case where medical science cannot establish the probability that ‘but for’ the breach the death would not have happened. Medical evidence confirms that there was a 60 per cent chance of death even without the breach.

Just because a seizure can be a natural cause of death this does not mean that factual causation will fail. The question is whether the seizure would have happened but for the breach on the balance of probabilities (Barnett v Chelsea and Kensington Hospital [1969] 1 QB 428).

The material increase in risk test does not apply here as currently this is confined to industrial disease cases, namely mesothelioma and lung cancer caused by exposure to asbestos (Fairchild v Glenhaven Funeral Services Ltd & Others [2003] 1 AC 32).

How well did you know this?
1
Not at all
2
3
4
5
Perfectly
12
Q

A man is in a car accident and suffers a head injury. On arrival at hospital, the man is not seen by a doctor for five hours. After seeing the doctor, he is immediately referred for an MRI (a type of scan producing images of inside the body), but there is a further delay of two hours. He is left brain damaged. Medical evidence suggests that the car accident, the first delay and second delay were all capable of contributing to his brain damage.

Which of the following statements is most accurate regarding factual causation in relation to the man’s brain injury?

Factual causation cannot be established as it cannot be shown that ‘but for’ the breaches (two delays) the man would not have suffered brain injury.

Given the car accident and two delays were all capable of causing or contributing to the brain injury, the material contribution test is satisfied.

Factual causation fails as it could have been the car accident, not the breaches (the two delays) that caused the brain injury.

There is a greater chance that the breaches (two delays) caused the brain injury than the car accident so factual causation is satisfied.

The two delays materially increased the risk of brain injury meaning factual causation is satisfied.

A

Given the car accident and two delays were all capable of causing or contributing to the brain injury, the material contribution test is satisfied.

Correct: This is a case where medical science cannot establish the probability that ‘but for’ an act of negligence (the delays) the brain injury would not have happened, but it can establish that the contribution of the delays was more than negligible (therefore the material contribution test is satisfied) (Bailey v Ministry of Defence [2008] EWCA Civ 883). Medical evidence established that all three causes were capable of contributing to the brain injury.

While the other answer options might sound plausible, they are each incorrect.

The material increase in risk test does not apply here as currently this is confined to industrial disease cases, namely mesothelioma and lung cancer caused by exposure to asbestos (Fairchild v Glenhaven Funeral Services Ltd & Others [2003] 1 AC 32).

Given this is a case where the medical evidence cannot attribute the brain injury to a specific cause, but can say that the car accident and breaches (two delays) contributed to the loss, the courts can depart from the ‘but for’ test and apply the material contribution test (Bonnington Castings Ltd v Wardlaw [1956] AC 613). If the medical evidence had said that it was the car accident OR the breaches which caused the brain injury (not both), then factual causation might have failed on the basis that it could have been the car accident rather than the breaches that caused the brain injury, applying the ‘but for’ test as per Wisher v Essex AHA [1988] AC 1074.

Medical evidence established that all three causes were capable of contributing to the brain injury. It has not been possible to separate out the relative contribution of the three factors, so it is not possible to say that there is a greater chance that the breaches caused the loss.

As explained above, this is a case where the courts might depart from the ‘but for’ test and apply the material contribution test instead (three possible causes and the two delays have made a more than negligible contribution) (Bonnington Castings Ltd v Wardlaw [1956] AC 613).

How well did you know this?
1
Not at all
2
3
4
5
Perfectly
13
Q

A woman contracts mesothelioma as a result of exposure to asbestos. Medical evidence has proved that mesothelioma (a type of lung cancer) can be caused by a single asbestos fibre entering the lung. The woman was negligently exposed to asbestos by a previous employer over 40 years ago. She was also exposed to asbestos in the general atmosphere as a young girl, as she lived in close proximity to a factory which used asbestos.

Which of the following statements is most accurate regarding factual causation in relation to the previous employer?

Factual causation fails as the woman cannot show on the balance of probabilities that the employer caused her mesothelioma. The woman could have been exposed to the single fibre as a young girl.

Factual causation is satisfied as there is a chance that the employer caused the woman’s mesothelioma.

Factual causation is satisfied as the employer materially contributed to the woman’s mesothelioma.

Factual causation is not satisfied as there are two possible independent causes and the ‘but for’ test therefore fails.

Factual causation is satisfied as the woman can show that the employer materially increased the risk of her contracting mesothelioma.

A

Factual causation is satisfied as the woman can show that the employer materially increased the risk of her contracting mesothelioma.

Correct
Correct. It is impossible for medical experts to say which exposure caused the mesothelioma. The ‘but for’ test therefore fails as it cannot be said on the balance of probabilities that the employer caused the woman’s mesothelioma (there is only a 50 per cent chance). However, this is an example of where the courts depart from the ‘but for’ test and apply the material increase in risk test (McGhee v National Coal Board [1973] 1 WLR 1, applied in Fairchild v Glenhaven Funeral Services Ltd & Others [2003] 1 AC 32). The employer might well be held liable for damage that it did not cause but this has to be weighed against the argument that people who suffer harm due to their employers’ breach deserve to be compensated. In mesothelioma cases, science is unable to provide the claimant with the means to ever establish factual causation using the ‘but for’ test (unless the claimant worked for only one employer who exposed them to asbestos, which is rare). The courts have therefore concluded that to achieve fairness the ‘but for’ test should be departed from and the material increase in risk test applied. Here it could be shown that the employer materially increased the risk of the woman contracting mesothelioma (given it only takes one fibre to cause the disease).

While the other answer options might sound plausible, they are each incorrect.

This is a situation (mesothelioma cases) where the courts depart from the ‘but for’ test and apply the material increase in risk test instead(McGhee v National Coal Board [1973] 1 WLR 1, applied in Fairchild v Glenhaven Funeral Services Ltd & Others [2003] 1 AC 32). It therefore does not matter that the woman cannot show on the balance of probabilities that the employer caused the mesothelioma.

The employer is liable for the mesothelioma because there is a chance it did cause the disease, but the correct legal terminology is material increase in risk(McGhee v National Coal Board [1973] 1 WLR 1, applied in Fairchild v Glenhaven Funeral Services Ltd & Others [2003] 1 AC 32).

The material contribution test does not apply here. As mesothelioma can be caused by exposure to a single asbestos fibre, the woman will not be able to prove on the balance of probabilities that the employer materially contributed (made a more than negligible contribution). The employer may have made no contribution at all to the disease.

There are two possible causes and the ‘but for’ test does fail, but this is an example of where the courts depart from the ‘but for’ test and apply the material increase in risk test instead (McGhee v National Coal Board [1973] 1 WLR 1, applied in Fairchild v Glenhaven Funeral Services Ltd & Others [2003] 1 AC 32).

How well did you know this?
1
Not at all
2
3
4
5
Perfectly
14
Q

Larry suffers physical injuries as a result of falling from a theme park ride. The theme park had failed to install appropriate safety measures. Larry is taken to hospital where a surgeon operates on him. Larry has an allergic reaction to the anaesthetic and suffers additional nerve damage.

Which one of the following is correct with regard to legal causation?

The actions of the surgeon will break the chain of causation and the theme park will only be liable for Larry’s original physical injuries and not the nerve damage.

The actions of the surgeon will not break the chain of causation but the theme park will only be liable for Larry’s original physical injuries.

The actions of the surgeon will break the chain of causation and the theme park will not be liable for any of Larry’s injuries.

The actions of the surgeon will break the chain of causation but the theme park will remain liable for all of Larry’s injuries.

The actions of the surgeon will not break the chain of causation and the theme park will be liable for all of Larry’s injuries.

A

The actions of the surgeon will not break the chain of causation and the theme park will be liable for all of Larry’s injuries.

Correct
Correct. The courts are reluctant to hold that medical treatment breaks the chain of causation and will only do so where the treatment is so gross and egregious as to be unforeseeable (Robinson v The Post Office).

While the other answer options might sound plausible, they are each incorrect.

If the actions of the surgeon did break the chain of causation (which they do not on these facts), the theme park would still be liable for any injuries Larry suffered prior to the surgeon’s act.

The actions of the surgeon are highly unlikely to break the chain of causation because they are not so gross and egregious as to be unforeseeable.

If the actions of the surgeon do not break the chain of causation, the theme park will remain liable for any losses flowing from their breach (physical injuries and nerve damage).

How well did you know this?
1
Not at all
2
3
4
5
Perfectly
15
Q

A scaffolder attends a building site where he is working. The day before, his colleague failed to secure some timbers properly. These fall on the scaffolder’s head and cause him to suffer a serious head injury. He had not yet put on his safety hard hat.

What is the scaffolder’s legal position regarding legal causation in his negligence claim against his colleague?

The scaffolder’s failure to wear a hard hat would be considered highly unreasonable and would therefore break the chain of causation. He will not receive any damages for his head injury.

If the scaffolder’s failure to wear a hard hat is considered unforeseeable, this may break the chain of causation.

If the scaffolder’s failure to wear a hard hat is considered unreasonable, this may break the chain of causation, but a court is more likely to deal with this in terms of contributory negligence.

If the scaffolder’s failure to wear a hard hat is considered highly unreasonable, this may break the chain of causation, but a court is more likely to deal with this in terms of contributory negligence.

If the scaffolder’s failure to wear a hard hat is considered unreasonable, this may break the chain of causation.

A

If the scaffolder’s failure to wear a hard hat is considered highly unreasonable, this may break the chain of causation, but a court is more likely to deal with this in terms of contributory negligence.

Correct
Correct. The scaffolder’s failure to wear a hat might be considered highly unreasonable and therefore break the chain of causation (McKew v Holland). However, this answer reflects the approach of the courts in preferring to reduce the damages of a culpable claimant rather than find that the claimant’s actions have broken the chain of causation.

While the other options might sound plausible, they are each incorrect. (unreasonable is not enough; it has to be highly unreasonable)

How well did you know this?
1
Not at all
2
3
4
5
Perfectly
16
Q

A man broke his leg when he was negligently hit by a car. This injury would normally result in a two month absence from work. However, due to a pre-existing condition that he has, his leg does not fully recover and he is also left with a permanent limp.

Which of the following statements best describes what the man can claim from the negligent driver?

The man can claim for personal injury for the broken leg and permanent limp, together with consequential economic loss for his lost earnings from not being able to work for two months.

The man can claim for personal injury for the broken leg and permanent limp.

The man can claim for his lost earnings from not being able to work for two months.

The man can claim for personal injury for the broken leg.

The man can claim for personal injury for the broken leg and consequential economic loss for his lost earnings from not being able to work for two months.

A

The man can claim for personal injury for the broken leg and permanent limp, together with consequential economic loss for his lost earnings from not being able to work for two months.

Correct: This answer applies the principle that the tortfeasor must take the Claimant as they find them as set out in Smith v Leech Brain [1962] 2 QB 405. Consequently, the driver is liable in relation to both the broken leg and the permanent limp. It also correctly identifies the economic loss as consequential economic loss given that the loss (i.e. the loss of earnings) is as a result of a personal injury (i.e. the broken leg).

While the other answer options might sound plausible, they are each incorrect.

The explanation above should explain why the other options are incorrect, namely the other options do not allow recovery for all the losses suffered by the man.

How well did you know this?
1
Not at all
2
3
4
5
Perfectly
17
Q

A decorator negligently left an oil lamp burning in a woman’s house. The woman was unaware of the lamp and bumped into it by accident. The lamp fell and then exploded, causing the woman to suffer severe burns.

Which of the following statements best describes what the woman can claim from the decorator in negligence?

The woman cannot claim for anything as burns from an explosion were not reasonably foreseeable.

The woman cannot claim for anything as she can only claim for losses that are a direct result of the decorator’s breach.

The woman can claim for personal injury for the severe burns as this loss was a direct result of the decorator’s breach.

The woman can claim for personal injury for the severe burns as damage caused by fire was reasonably foreseeable.

The woman can claim for minor burns as damage caused by a fire was reasonably foreseeable but not from an explosion.

A

The woman can claim for personal injury for the severe burns as damage caused by fire was reasonably foreseeable.

Correct: This answer applies the principle that once it has been established that the kind of damage is reasonably foreseeable, there is no need for it to be reasonable for the Defendant to foresee the exact circumstances leading up to the damage, as set out in Hughes v Lord Advocate [1963] AC 837.

While the other options might sound plausible, they are each incorrect.

The explanation above should explain why the other options are incorrect, i.e. they do not apply the legal principle from Hughes v Lord Advocate [1963] AC 837. Two of the options refer to ‘direct result’ of the breach. This is the historic test for remoteness which has been replaced with reasonable foreseeability (Wagon Mound No 1 [1961] AC 388).

How well did you know this?
1
Not at all
2
3
4
5
Perfectly
18
Q

During a premier league football match, a footballer suffers a broken leg after another player executes a dangerous and serious foul tackle against them.

Which of the following statements best describes whether the player who executed the tackle (‘the defendant’) can successfully rely on the defence of volenti?

The defendant can rely on volenti because the injured footballer consented to the risk of being tackled during a football match.

The defendant can rely on volenti because the injured footballer had the capacity to consent and knew the nature and extent of the risk of harm by participating in the football match.

The defendant cannot rely on volenti because the injured footballer did not expressly consent to the risk of injury.

The defendant cannot rely on volenti because the injured footballer did not consent to dangerous play showing reckless disregard for their safety.

The defendant can rely on volenti because the injured footballer impliedly consented to the risk of injury by playing in the football match.

A

The defendant cannot rely on volenti because the injured footballer did not consent to dangerous play showing reckless disregard for their safety.
Correct. This is the correct law for volenti in relation to sport.

While the other options might sound plausible, they are each incorrect.

It is highly unlikely that a participant in sport will be found to have consented to something that happens outside the rules of the game and shows a reckless disregard for their safety (e.g. a dangerous and serious foul tackle).

In order to rely on volenti, the defendant need not prove that the claimant expressly consented to the risk of injury. Consent can be implied. The reason consent has not been implied here is stated in the paragraph above.

Nothing on the facts suggest that the footballer did not have capacity to consent. However, it is highly unlikely that they will be assumed to have known that there was a risk of injury caused by a dangerous and serious foul tackle i.e. by dangerous play demonstrating a reckless disregard for their safety.

How well did you know this?
1
Not at all
2
3
4
5
Perfectly
19
Q

A pedestrian is negligently knocked over by a cyclist who was travelling very fast. The pedestrian suffers a serious head injury and is in hospital for two weeks, during which time they cannot work, but they receive contractual sick pay from their employer. They die from their head injuries two weeks after the accident.

Which of the following statements is most accurate in relation to damages that the pedestrian’s estate might receive if successful in a claim for negligence against the cyclist?

The pedestrian’s estate is likely to be awarded compensatory damages, divided into special and general damages. The contractual sick pay will not be deducted from any special damages awarded. The general damages will include a PSLA award.

The pedestrian’s estate is likely to be awarded compensatory damages, divided into special and general damages. The contractual sick pay will be deducted from any special damages awarded. The general damages will not include a PSLA award.

The pedestrian’s estate is likely to be awarded compensatory damages, divided into special and general damages. The contractual sick pay will not be deducted from any special damages awarded. The general damages will not include a PSLA award.

The pedestrian’s estate is likely to be awarded special damages only. The contractual sick pay will be deducted from any special damages awarded.

The pedestrian’s estate is likely to be awarded compensatory damages, divided into special and general damages. The contractual sick pay will be deducted from any special damages awarded. The general damages will include a PSLA award.

A

The pedestrian’s estate is likely to be awarded compensatory damages, divided into special and general damages. The contractual sick pay will be deducted from any special damages awarded. The general damages will include a PSLA award.

Correct
Correct. Compensatory damages are by far the most common type of damages awarded in the tort of negligence. The aim of such damages is to put the claimant in the position they would have been in but for the defendant’s tortious act, as far as this is possible with an award of money. Even though the pedestrian has died, their estate may bring a claim for any losses suffered by the pedestrian as a result of the accident up to the date of death. This is calculated on the same basis as a normal personal injury award, but no claims can be made for any losses that might have arisen after the date of her death (s.1(2)(a) Law Reform (Miscellaneous Provisions) Act 1934).

The pedestrian’s estate will therefore be entitled to special damages which cover specifically provabale and quantifiable financial losses, for example, loss of earnings. However, deductions will be made for any contractual sick pay the pedestrian received before they died. This is to avoid the claimant receiving two sums of money for the same reason.

The estate will also be entitled to general damages which will include a sum for the PSLA award (the pain, suffering and loss of amenity award). This award attempts to provide some financial compensation for the effect of the injury. The court would therefore assess the PSLA suffered by the pedestrian from the time of the accident until the date of death.

While the other options might sound plausible, they are each incorrect.

The explanation given above for why this answer is correct should explain why the other options are incorrect.

How well did you know this?
1
Not at all
2
3
4
5
Perfectly
20
Q

A teenage boy is cycling on a narrow country road at a fast pace as he is training for a race. A woman is walking along the road with her dog, which is not on the lead. The dog barks furiously at the cyclist and jumps in front of the bicycle, causing the boy to fall and suffer a broken shoulder.

Which of the following represents the best advice to the woman in relation to defences to a negligence claim by the boy?

The defence of volenti will apply as the boy has chosen to run an obvious and serious risk of harm in cycling at a fast pace on a country lane.

There will be no defence available to the woman as the boy has done nothing wrong.

The defences of volenti and contributory negligence will apply as the obvious and serious risk of harm in cycling at a fast pace has contributed to the harm suffered by the boy.

The defence of contributory negligence will apply as the boy’s actions have contributed to the harm he has suffered.

The defence of contributory negligence will apply to negate any liability of the woman.

A

The defence of contributory negligence will apply as the boy’s actions have contributed to the harm he has suffered.

Correct. Contributory negligence would probably be argued successfully here as the facts appear to satisfy the two-stage test from Jones v Livox Quarries: (1) the boy has failed to take reasonable care for his own safety by cycling at a fast pace on a narrow country road, and (2) this has contributed, at least to some extent, to the injuries he has suffered. If the court were satisfied that this test was satisfied, they would have the power to reduce the boy’s damages by a ‘just and equitable’ amount: s.1 Law Reform (Contributory Negligence) Act 1945

While the other options might sound plausible, they are each incorrect.

The given facts are unlikely to satisfy the four-stage test for volenti. Even if the boy has capacity to consent and is taken to have understood that there is a risk of accident when cycling fast on a narrow country road, he will not be found to have consented to the negligence of the woman in failing to control her dog on a public road, ie. the nature and extent of the risk has not been understood. Furthermore, although he has voluntarily run the risk of injury, the risk itself is unlikely to be regarded as such an obvious and serious risk as to satisfy the threshold set for implied agreement to harm: see Morris v Murray.

The speed at which the boy is cycling, given that this is a narrow country road, may be found to show a lack of reasonable care for his own safety, which has contributed to the harm he has suffered. It is not therefore correct to say that the boy has done nothing wrong.

The defence of contributory negligence is a partial defence which operates to reduce the level of damages payable by a defendant but does not negate liability.

How well did you know this?
1
Not at all
2
3
4
5
Perfectly
21
Q

A supply music teacher (‘the Teacher’) is sent by their agency, to work at a private school for one week. The Teacher is paid by their agency. The school stipulates that the Teacher must arrive at the school before 9am every weekday. They are free to work elsewhere at the weekends. While the Teacher is setting up the school stage for a music concert, they accidentally drop a music stand on the foot of one of the pupils. The pupil suffers swelling and bruising to their foot.

Which of the following statements is most accurate?

The pupil can bring a claim against the Teacher in negligence and the school in employers’ primary liability as they have failed to provide the pupil with competent staff.

The pupil can bring a claim against the Teacher in negligence and against the agency vicariously.

The pupil can bring a claim against the Teacher directly in negligence and vicariously against the school.

The pupil can bring a claim against the Teacher directly in negligence and vicariously.

The pupil can only bring a vicarious claim against the school if they can establish that the Teacher is an employee of the school

A

The pupil can bring a claim against the Teacher directly in negligence and vicariously against the school.

Correct: The pupil can bring a claim vicariously against the school because even though the Teacher is unlikely to be deemed an employee of the school as they will not satisfy the test from Ready Mixed Concrete v Ministry of Pensions, they may be deemed to be in a relationship akin to employment with the school (Barclays Bank v Various Claimants). They committed the tort as a result of an activity (setting up the music stage) which they performed on behalf of the school and the school exercised a significant degree of control over what the Teacher did.

While the other answer options might sound plausible, they are each incorrect.

The pupil would not be able to bring a claim against the school in employers’ primary liability as they are not an employee of the school.

The pupil can bring a vicarious claim against the school if the Teacher is in a relationship akin to employment with the school (not just if the Teacher is an employee of the school).

The pupil cannot sue the agency vicariously. The Teacher is not the agency’s employee nor are they in a relationship akin to employment with the agency. Applying the economic reality test from Ready Mixed Concrete v Minister of Pensions, it is unlikely that the Teacher will be deemed to be the agency’s employee. The agency is unlikely to exercise the requisite day to day control over the Teacher to render them an employer. In addition, it is unlikely that the Teacher is in a relationship akin to employment with the agency given the five-stage test approved in Barclays Bank.

The pupil would not be bringing a claim against the Teacher vicariously. Vicarious liability is relevant when the claimant sues the defendant for a tort committed by someone else. Here, the Teacher has committed the tort so any claim against the Teacher will be direct.

How well did you know this?
1
Not at all
2
3
4
5
Perfectly
22
Q

A yoga instructor employed at a dance academy is asked to cover a Zumba class. The yoga instructor is not supposed to teach Zumba because of a pre-existing knee injury, which their employer knows about. Nevertheless, they agree feeling concerned they may lose their job if they refuse. Halfway through the class the yoga instructor’s knee gives way and their knee has to be operated on. When the yoga instructor returns to work, a colleague laughs at their injury. The yoga instructor gets verbally aggressive and kicks their colleagues’ leg. The yoga instructor has a history of violent mood swings.

The colleague could bring a claim against the yoga instructor in the tort of battery. What other claims arise from these set of facts?

The yoga instructor and the colleague can bring vicarious liability claims against the dance academy.

The yoga instructor and the colleague can both bring direct employers’ primary liability claims against the dance academy.

The yoga instructor can bring a claim in employers’ primary liability against the dance academy. The colleague can bring both a vicarious claim against the dance academy, and a direct claim in employers’ primary liability.

The yoga instructor can bring both a vicarious claim against the dance academy, and a direct claim in employers’ primary liability. The colleague can bring an employers’ primary liability claim against the dance academy.

The yoga instructor and the colleague can both bring vicarious liability and direct employers’ primary liability claims against the dance academy.

A

The yoga instructor can bring a claim in employers’ primary liability against the dance academy. The colleague can bring both a vicarious claim against the dance academy, and a direct claim in employers’ primary liability.

Correct: the yoga instructor is employed by the dance academy and can therefore bring a claim against them in employers’ primary liability for breach of duty (the duty to take reasonable precautions to ensure an employee’s safety, including safe systems of work) (Wilsons and Clyde Coal Co Ltd v English [1938] AC 57).

The colleague can bring a vicarious claim against the dance academy for the yoga instructors’ assault. A tort has been committed, by an employee and during the course of employment. In addition, the colleague can bring an employers’ primary liability claim against the dance academy. They are employed by the dance academy and can therefore bring a claim against them in employers’ primary liability for breach of duty (the duty to take reasonable precautions to ensure an employee’s safety, including the provision of safe and competent employees) (Wilsons and Clyde Coal Co Ltd v English [1938] AC 57).

While the other options might sound plausible, they are each incorrect.

The yoga instructor cannot bring a vicarious claim against the dance academy. It is the dance academy that has committed the tort and so the yoga instructor must sue them directly.

How well did you know this?
1
Not at all
2
3
4
5
Perfectly
23
Q

A crane driver negligently caused personal injury to a woman through their operating of the crane. The crane driver has a contract with the company they work for and this contract describes them as self-employed. The contract stipulates that their working hours are 9am to 6pm and that they must work for the company at least three days a week. They are paid at the end of the month for the number of hours they have worked. They use the company’s crane when working and must wear the company’s uniform. The woman wants to sue this company vicariously for the crane driver’s negligence.

Which of the following statements most accurately describes whether the woman can sue the company vicariously?

It is highly likely the crane driver is the company’s employee, meaning the woman can sue the company vicariously for the crane driver’s negligence. They are likely to be an employee because they have entered into a contract with the company.

It is highly likely the crane driver is the company’s employee, meaning the woman can sue the company vicariously for the crane driver’s negligence. The crane driver is likely to be an employee because the company exerts a high degree of control over the crane driver and the crane driver is paid in exchange for their service.

It is highly likely the crane driver is the company’s employee, meaning the woman can sue the company vicariously for the crane driver’s negligence. The crane driver is likely to be an employee because the company pays the crane driver regularly for the number of hours they work each month.

It is unlikley that the crane driver is the company’s employee, meaning the woman cannot sue the company vicariously for the crane driver’s negligence. They are unlikely to be an employee because the contract states that the crane driver is self-employed.

It is unlikely that the crane driver is the company’s employee, meaning the woman cannot sue the company vicariously for the crane driver’s negligence. They are unlikely to be an employee because the company only pays the crane driver for the number of hours worked rather than a fixed salary.

A

It is highly likely the crane driver is the company’s employee, meaning the woman can sue the company vicariously for the crane driver’s negligence. The crane driver is likely to be an employee because the company exerts a high degree of control over the crane driver and the crane driver is paid in exchange for their service.

Correct. Applying the multiple factors test from Ready Mixed Concrete v Minister of Pensions [1968] 2 QB it is likely that the crane driver is an employee of the company despite being described as self-employed. The crane driver receives remuneration in exchange for their personal service, and the company exert a high degree of control over them, requiring them to work for the company three days a week between 9-6 and to wear their uniform. We are not told of any contractual factors (except the label) that are inconsistent with employment, in fact, we are told that the crane driver must use the company’s tools at work. Overall, this suggests the crane driver is the company’s employee and as their employer, the company can be vicariously liable for their negligence committed during the course of employment.

While the other options might sound plausible, they are each incorrect.

It is correct that the fact the crane driver is paid regularly by the company, this is an indicator that the crane driver is their employee. However, there are additional indicators of the crane driver’s employment status (as discussed above).

The label in a contract as to whether someone is employed or self-employed is taken into consideration when determining employment status, but it is not conclusive. All relevant contractual terms are considered to determine the true nature of someone’s employment status.

The first stage of the test from Ready Mixed focuses on remuneration in exchange for personal service – regular remuneration (not just a fixed salary) is evidence of being an employee. The crane driver must work for the company at least three days a week, so will be receiving regular pay in exchange for their personal service.

Just because the crane driver has a contract with the company, this does not mean that they are an employee. It will be the terms of the contract that determine this, subject to the test from Ready Mixed.

How well did you know this?
1
Not at all
2
3
4
5
Perfectly
24
Q

A construction employee is told by their employer that they must wear a safety hat at all times whilst on the construction site. One day they arrive for their shift but have forgotten to bring their safety hat. The site foreman gives the employee a safety hat, but the employee refuses to wear it. The site foreman allows the employee to work anyway. During the employee’s shift they suffer a serious head injury which would have been avoided had they been wearing a safety hat.

Which of the following best explains whether or not the employer has breached its duty of care to the construction employee?

The employer is not in breach of its duty as the construction employee refused to wear the safety hat provided.

The employer is in breach of duty as it has failed to provide safety equipment and safe and competent employees.

The employer is not in breach of its duty as it employed a foreman to try and ensure that a safe system of work was operated.

The employer is not in breach of its duty as it told the construction employee to wear a safety hat at all times.

The employer is in breach of duty as it has failed to operate a safe system of work and to provide safe and competent employees.

A

The employer is in breach of duty as it has failed to operate a safe system of work and to provide safe and competent employees.

Correct. An employer owes a non-delegable duty of care to its employees both to provide a safe system of work and to operate that system. The foreman is acting on behalf of the company when they fail to operate the safe system in place. The employer can therefore be directly liable for the foreman’s failure to operate the safe system. The question of whether insistence on using safety equipment provided should be resorted to depends on the nature and risk of harm liable to occur if the equipment is not worn. Here the nature and risk of harm is serious and it is probable that insistence on wearing the safety hat was appropriate (Bux v Slough Metals [1974] 1 Lloyd’s Rep 155). In addition, given the foreman allowed the employee to work without a safety hat, the employer has arguably failed to provide safe and competent employees (Black v Fife Coal Ltd [1912] AC 149).

While the other options might sound plausible, they are each incorrect.

The employer has provided safety equipment. The issue is whether the foreman should have insisted on the construction employee wearing the safety hat.

There are circumstances in which it may be appropriate for an employer to insist that an employee wear/use safety equipment. It is often not enough to just provide and tell employees to use safety equipment. Not only must a safe system of work be provided, it must also be operated.

How well did you know this?
1
Not at all
2
3
4
5
Perfectly
25
Q

Company A lends their employee bus driver to Company B for the duration of one week. Company A trained the bus driver how to drive their buses. During this week Company A continues to pay the bus driver’s wages and retains the power of dismissal. However, the bus driver drives Company B’s bus which is a similar model to the buses used by Company A. The bus driver negligently opens the door too fast hititng a passenger waiting to get on the bus.

Which of the following statements is most accurate in relation to vicarious liability?

The injured passenger should sue both Company A and Company B vicariously for the bus driver’s negligence.

The injured passenger should sue Company B vicariously for the bus driver’s negligence.

The injured passenger should sue Company A vicariously for the bus driver’s negligence.

The injured passenger cannot sue the bus driver, Company A or Company B vicariously for the bus driver’s negligence.

The injured passenger should sue the bus driver vicariously for his/her negligence.

A

The injured passenger should sue both Company A and Company B vicariously for the bus driver’s negligence.

Correct. This seems to be a rare occasion where there is dual liability. In Mersey Docks and Harbour Board v Coggins and Griffiths [1947] AC 1 the House of Lords stated that, as a general rule, there was a rebuttable presumption that the original employer would remain vicariously liable. However, in Viasystems Ltd v Thermal Transfer Ltd and Others [2005] EWCA Civ 1151 the court found that, in certain circumstances, it is possible for two parties to be vicariously liable for the actions of the same, negligent employee. Dual liability might occur where an employee is lent to another employer and both employers are entitled, and obliged, to control the employee’s actions so as to prevent the negligent act. This seems to be the case here – the duration of the lending was relatively short, Company A trained the bus driver to drive, they continued to pay the bus driver’s wages and retained the power of dismissal. However, the bus driver was driving Company B’s bus at time of the negligence and Company B had a responsibility to make sure the bus driver was a competent driver before allowing the bus driver to drive one of their buses.

While the other options might sound plausible, they are each incorrect.

The explanation above should explain why the other options are incorrect. The injured passenger would not be suing the bus driver vicariously. The bus driver committed the negligence so the injured passenger can sue him/her directly in the tort of negligence.

How well did you know this?
1
Not at all
2
3
4
5
Perfectly
26
Q

Newlyweds leaving for their honeymoon are dropped off at the airport by their best man. Just after their arrival at the airport, it catches fire due to poor electrical maintenance. The best man hears about the fire on the radio in his car and is extremely distressed. The newlyweds escape unharmed, but three months later their best man is still suffering from nightmares.

Which of the reasons below is the best reason why a negligence claim brought by the best man against the airport would fail?

He was not related to the newlyweds.

He did not fear for his own safety.

He did not suffer from a medically recognised psychiatric illness.

He did not witness the incident.

His distress was not reasonably foreseeable.

A

He did not suffer from a medically recognised psychiatric illness.

Correct
Correct. In all claims for psychiatric harm a key criterion is that the claimant has suffered a medically recognised psychiatric illness; distress and nightmares are not medically recognised psychiatric illnesses.

How well did you know this?
1
Not at all
2
3
4
5
Perfectly
27
Q

A married couple are at a festival. The wife stays at the back of the crowd and the husband pushes to the front. Due to poor crowd control arrangements, when the crowd unexpectedly surges forward a number of people at the front are severely injured. The couple are both very frightened. The husband helps to get the injured people into ambulances, but the wife is too worried about her husband to help. Neither of them are physically hurt, but both suffer from a recognised psychiatric illness afterwards. If the couple were to bring a negligence claim against the festival organiser for their psychiatric illness, which of the following best describes how they will be viewed for the purposes of duty of care?

Two primary victims.

Two secondary victims.

A primary victim and a secondary victim.

An actual victim and a primary victim.

An actual victim and a secondary victim.

A

A primary victim and a secondary victim.

Correct. The husband was in the danger zone and feared for his own safety – Page v Smith (1996) 1 AC 155. The wife was not in the danger zone but feared for her husband’s safety – Alcock v Chief Constable of South Yorkshire Police (1992) 1 AC 310.

How well did you know this?
1
Not at all
2
3
4
5
Perfectly
28
Q

A man is watching a cricket match on television when he sees part of the spectator area at the cricket ground collapse. The man knows that his son is at the match but does not know where he was sitting. He goes straight to the cricket ground where he is told by the police that his son has been taken to hospital. When he arrives at the hospital an hour later, he is told that his son has died from crush injuries but several hours pass before he is able to identify him in the morgue. The man develops clinical depression.

Which of the following provides the best advice to the cricket ground?

The cricket ground will owe a duty of care to the man as he witnessed the immediate aftermath of the accident.

The cricket ground will owe a duty of care to the man as he has suffered a recognised psychiatric injury foreseeable in a person of ordinary fortitude.

The cricket ground will owe a duty of care as the man’s clinical depression is a sudden reaction to witnessing the accident.

The cricket ground will not owe a duty of care to the man as he saw the accident happen on television.

The cricket ground will owe a duty of care to the man as the father/son relationship is a sufficiently close tie of love and affection to provide proximity.

A

The cricket ground will not owe a duty of care to the man as he saw the accident happen on television.

The man is a secondary victim as he was not in the danger zone but was in reasonable fear for the safety of his son (Alcock; Page v Smith). He will only be able to establish that he is owed a duty of care, by the cricket ground, if he can meet the criteria in Alcock. One criterion is that the manner of perception must be with one’s own unaided senses. Here he sees the accident happen on television, thus not with his own senses unaided. An exception to this has been made where the claimant has witnessed the ‘immediate aftermath’ of the accident (McLoughlin v O’Brian). However, on the facts the man did not ‘witness’ the injuries to his son for several hours and so would be highly unlikely to succeed on this basis as a time lapse of 7-8 hours was found to be too long in Alcock. Additionally, the man did not know where, at the cricket ground, his son was in the ground. In Alcock, the equivalent manner of perception was found to be insufficient, partly because broadcasting rules prevented the broadcast of recognisable individuals affected. The same would probably apply here and so the man would not be owed a duty of care by the cricket ground.

How well did you know this?
1
Not at all
2
3
4
5
Perfectly
29
Q

A niece and her aunt go for a haircut and colour. The hairdresser uses the wrong hair dye which causes the niece a severe burning rash and her skin starts to peel away immediately. The niece becomes clinically depressed. The aunt is so upset when she sees her niece’s skin peeling away that she cannot stop thinking about it several weeks later. The aunt is later diagnosed with post-traumatic stress disorder.

Which one of the following statements best explains the legal position in the tort of negligence where the loss is psychiatric harm?

The niece is an actual victim. The aunt is a primary victim; her claim will likely fail as she has not suffered any physical injury.

The niece is an actual victim. The aunt is a secondary victim; her claim will likely fail because she did not witness the event or the immediate aftermath.

The niece is an actual victim. The aunt is a primary victim; her claim will likely fail due to insufficient ties of love and affection with her niece.

The niece is a primary victim. The aunt is a secondary victim; her claim will likely fail due to insufficient ties of love and affection with her niece.

The niece is an actual victim. The aunt is a secondary victim; her claim will likely fail due to insufficient ties of love and affection with her niece.

A

The niece is an actual victim. The aunt is a secondary victim; her claim will likely fail due to insufficient ties of love and affection with her niece.

Correct
The niece is an actual victim as she has suffered physical injury as well as psychiatric harm. The aunt is a secondary victim as she has suffered psychiatric harm as a result of fearing for someone else’s safety. It is likely that her claim would fail as she may not have close ties of love and affection with her niece (the second stage of the Alcock criteria when establishing duty of care). However, of course, we would need to know more about their relationship to be sure but the aunt/niece relationship does not fall within one of the relationships where there is a rebuttable presumption of close ties of love and affection. While the other answer options might sound plausible, they are incorrect. The niece is not a primary victim as she has suffered physical injury as well as psychiatric harm. The aunt is a secondary victim, but she has witnessed the event. The aunt is not a primary victim as she was never in reasonable fear for her own safety.

How well did you know this?
1
Not at all
2
3
4
5
Perfectly
30
Q

A contractor, working on the Council’s electricity cables, accidentally cuts through one of the cables. As a result, the electricity supply to a nearby bakery is cut off for 24 hours. The industrial oven in the bakery is no longer working. The batch of 100 cakes in the oven have failed to rise and are ruined. The ovens ordinarily produce 500 cakes per day. The bakery would like to sue the contractors in negligence.

Which of the following statements is the best advice to the bakery as to what losses are claimable?

The bakery will be able to recover the cost of the ruined batch of cakes in the oven and the profit from the additional 400 cakes they would have made that day but they will not recover for the loss of profit on the ruined cakes in the oven.

The bakery will be able to recover the cost of the ruined batch of cakes in the oven, the lost profit from those cakes and the profit from the additional 400 cakes they would have made that day.

The bakery will be able to recover the lost profit from the ruined batch of cakes in the oven and the other 400 cakes that would have been made that day but they will not recover the costs of the ruined batch of 100 cakes in the oven.

The bakery will be able to recover the cost of the ruined batch of cakes in the oven but they will not recover for the lost profit from these cakes or the 400 additional cakes they would have made that day.

The bakery will be able to recover the cost of the ruined batch of cakes in the oven and the lost profit from selling that batch. They will not be able to recover the profit for the other 400 cakes they would have made that day.

A

The bakery will be able to recover the cost of the ruined batch of cakes in the oven and the lost profit from selling that batch. They will not be able to recover the profit for the other 400 cakes they would have made that day.

This is correct. Pursuant to Spartan Steel v Martin, the bakery will be able to recover the cost of the batch of 100 cakes in the oven (property damage), the lost profits from that batch (consequential economic loss). They will not be able to recover for the lost profits on the 400 additional cakes they would have made that day but for the interruption to the electricity supply. This loss constitutes pure economic loss on the basis that the cables are not owned by the bakery and it is therefore not claimable in tort law.

How well did you know this?
1
Not at all
2
3
4
5
Perfectly
31
Q

An ex-employee asks their previous employer to give a reference to a potential future employer. The previous employer carelessly confuses its own records, and as a consequence, advises the potential future employer that the ex-employee had many unexplained days off work. This is inaccurate. As a result of this inaccuracy, the potential future employer decides not to employ the ex-employee, causing them (the ex-employee) financial loss.

Which of the following best summarises whether or not the ex-employee’s previous employer owes them a duty of care in relation to this loss, and why.

There could be a duty of care owed to the ex-employee on the basis that the previous employer assumed responsibility to the ex-employee to exercise reasonable skill and care in the preparation of the reference when agreeing to provide it.

There could be a duty of care owed to the ex-employee on the basis that harm is clearly foreseeable.

There could be a duty of care owed to the ex-employee on the basis of the proximity between the previous employer and the ex-employer.

There is no duty of care owed to the ex-employee as the reference was provided to the potential future employer, not the ex-employee.

No duty of care is owed because the loss is pure economic loss.

A

There could be a duty of care owed to the ex-employee on the basis that the previous employer assumed responsibility to the ex-employee to exercise reasonable skill and care in the preparation of the reference when agreeing to provide it.

Correct. This reflects Spring v Guardian Assurance.

How well did you know this?
1
Not at all
2
3
4
5
Perfectly
32
Q

A professional golfer is being driven to a golf tournament by a friend. The golfer knows that their friend only passed their driving test one week ago and so is an inexperienced driver. On the way to the tournament the golfer and driver are in a car crash caused by the driver’s negligent driving. The golfer breaks their arm in the crash. As a result of this injury the golfer is unable to golf for several months.

Which one of the following correctly describes the forms of loss that the golfer would be able to claim for?

Pure economic loss.

Personal injury and property damage.

Personal injury only.

Personal injury and pure economic loss.

Personal injury and consequential economic loss.

A

Personal injury and consequential economic loss.

Correct
Correct. The golfer has suffered personal injury (a broken arm) and consequential economic loss (unable to play golf as a result of their injuries).

How well did you know this?
1
Not at all
2
3
4
5
Perfectly
33
Q

At the gym one weekend, a wealthy individual who has enjoyed investing in the stock market for many years bumps into a friend who is a stockbroker. The stockbroker gives the wealthy individual the name of a company whose share price they are sure is about to rise sharply and suggests that the individual might like to invest in the company. The wealthy individual invests immediately but the company goes bankrupt the following week and they lose £50,000.

Which of the following is the best explanation of whether or not the wealthy individual may be able to successfully sue their friend for their negligent advice?

The wealthy individual may be unsuccessful because it was unreasonable for them to rely on the stockbroker’s advice as it was given in an informal setting,

The wealthy individual may be unsuccessful because it was unreasonable for them to rely on their friend’s advice; the wealthy individual was an experienced investor on the stock market.

The wealthy individual may be unsuccessful because it was unreasonable for them to rely on the stockbroker’s advice as it was given for free.

The wealthy individual may be successful because there is a special relationship between themselves and the stockbroker, as the stockbroker is their friend.

The wealthy individual may be successful because it was reasonable for them to rely on the stockbroker’s advice given the friend was a stockbroker.

A

Correct. For the stockbroker to owe the individual a duty of care the courts might consider the reasonable reliance test; Hedley Byrne v Heller (1964) AC 465. The individual must have relied on the stockbroker’s advice, which they did. The stockbroker must have known that they would rely on their advice, which they did. However, it must have been reasonable for the individual to rely on the stockbroker’s advice, which it arguably was not, given their experience in investing in the stock market; Stevenson v Nationwide Building Society (1984) 272 EG 663.

How well did you know this?
1
Not at all
2
3
4
5
Perfectly
34
Q

A university researcher prepares a report on fast growing new businesses. The report is prepared for the university’s own internal use and for the local Chamber of Commerce. An investor visiting the Chamber of Commerce picks up a copy of the report and decides to invest in one of the new businesses, which is described as ‘solvent, fast growing, and a risk-free investment’. The business fails miserably, and the investor loses all their money.

The investor is furious and takes advice from a solicitor as to a potential action against the university researcher for negligence.

Which of these statements is the best legal advice about whether or not the university researcher will owe the investor a duty of care?

The investor is unlikely to be owed a duty of care because they do not meet the criteria for an assumption of responsibility set out in Caparo.

The investor is unlikely to be owed a duty of care because they cannot show that they paid for the report in order to demonstrate that they relied on it.

The investor is likely to be owed a duty of care because they will be able to show that it was reasonable for them to rely on the report, and that the university researcher knew or ought to have known that an investor would rely on that report.

The investor is likely to be owed a duty of care because they will be able to show reasonable reliance on the report, supported by the university lecturer’s special expertise in this area.

The investor is likely to be owed a duty of care because the report was made for a similar purpose to that for which it was used.

A

The investor is unlikely to be owed a duty of care because they do not meet the criteria for an assumption of responsibility set out in Caparo.

Correct. This is a third-party case like Caparo, so it is likely the courts would apply the four criteria from this case for assumption of responsibility.

How well did you know this?
1
Not at all
2
3
4
5
Perfectly
35
Q

Which of the following is the best summary of the ‘defensive practices’ policy concern?

Liability should not be imposed where the alleged negligence is actually an established practice.

Liability should be imposed where it would defend the rights of victims.

Liability should be imposed where it discourages practices which include inappropriate risks.

Liability should not be imposed where it would lead to people taking undesirable ‘cautious’ / risk averse action.

A

Liability should not be imposed where it would lead to people taking undesirable ‘cautious’ / risk averse action.

Correct
Well done. This is what the ‘defensive practices’ policy concern is about. The idea of imposing liability where it discourages practices which include inappropriate risks is more akin to the ‘maintaining high standards’ policy concern. The other two answers suggest a misunderstanding of this area.

How well did you know this?
1
Not at all
2
3
4
5
Perfectly
36
Q

A police informant is claiming damages for negligence by the police. It is alleged that the police negligently left the informant’s contact details unattended in a police car which was stolen, and eventually reached the criminal against whom the informant had given evidence. The criminal broke into the informant’s house and physically assaulted them. Which of the following statements best describes the principal reason why the police might owe the informant a duty of care?

A duty of care is owed because the police created the source of danger.

A duty of care is owed because the informant is an identifiable victim.

A duty of care is owed because the police assumed responsibility for the informant’s welfare.

A duty of care is owed because it is fair, just and reasonable

A

A duty of care is owed because the police assumed responsibility for the informant’s welfare.

Correct
This answer best reflects why the Court of Appeal imposed a duty of care in the case of Swinney v Chief Constable of Northumbria (No.2), The Times, 25 May 1999 (same facts as the fact pattern). The police assume responsibility to protect informers against the criminals they give evidence about. Informers should not be considered like other members of the public; they have a special relationship with the police. Whilst being an identifiable victim certainly supports the argument for imposing a duty of care, recent case law suggests that this factor alone is not sufficient to a duty being imposed, so this is not the principal reason why the policy might owe a duty. Whilst creating the source of danger could support the argument for imposing a duty of care, this argument was not emphasised in Swinney and focus seems to be on assumption of responsibility.

How well did you know this?
1
Not at all
2
3
4
5
Perfectly
37
Q

Which of the following cases is the best authority for the proposition that medical professionals owe a duty of care to patients which they have accepted for treatment?

Robinson v CC of West Yorkshire Police

Baker v TE Hopkins & Son Ltd

Cassidy v Ministry of Health

Caparo Industries v Dickman

A

Cassidy v Ministry of Health

Well done. The other cases are all relevant to duty of care, but not this particular proposition.

How well did you know this?
1
Not at all
2
3
4
5
Perfectly
38
Q

A cyclist moves to overtake a slow-moving car, after giving the appropriate signals. At the same time, the driver of the car turns right without checking his mirrors and collides with the cyclist, causing injuries to the cyclist. Which of the following best summarises how the court will approach the question of whether the motorist owed a duty of care to the cyclist?

The court will consider foreseeability, proximity between the parties and whether imposing a duty of care would be fair, just and reasonable. Assuming these are satisfied, a duty of care was owed.

Foreseeability of harm is normally required before a duty can be imposed, but foreseeability alone does not normally justify imposing a duty. In any event, this is not how the court is likely to approach duty in this case. Revisit this element.

The court will rely on clear precedent to conclude that a duty of care was owed.

No duty of care was owed.

A

The court will rely on clear precedent to conclude that a duty of care was owed.

Well done. Precedent makes it a clear that a motorist owes a duty to other road users. See (eg) Nettleship v Weston. So it would be wrong to say ‘no duty of care was owed’. The other answers are wrong because the court will not need to engage in an original and/or complex analysis in relation to duty if there is a clear precedent. Even if it did, foreseeability of harm alone does not normally justify imposing a duty.

How well did you know this?
1
Not at all
2
3
4
5
Perfectly
39
Q

A farmer runs a sheep farm at the edge of a small rural village. She has owned and run the farm for 25 years. The sound of the sheep and the shearing is very loud prevents the owner of neighbouring house from using his garden for most of the summer. He has lived in the house for 22 years and has not been too bothered by the noise, but this year the farmer has increased the number of sheep by 50% and the neighbour decides he must do something about it now.

The neighbour visits a local solicitor for advice.

Which of these represents the most accurate legal advice?

The farmer has been causing an actionable nuisance to the neighbour, and the neighbour will not have an actionable claim because they moved to the nuisance.

The farmer has been causing an actionable nuisance to the neighbour, and the farmer is unlikely to have a defence of prescription if there has been significant increase in the nuisance.

The farmer has been causing an actionable nuisance for 25 years so may have the defence of prescription.

The farmer has been causing an actionable nuisance to the neighbour for 22 years and the neighbour has not complained until now, so the farmer will have the defence of prescription.

The farmer has probably not been causing an actionable nuisance due to the rural neighbourhood in which the farm is located.

A

The farmer has been causing an actionable nuisance to the neighbour, and the farmer is unlikely to have a defence of prescription if there has been significant increase in the nuisance

This answer best reflects the law in this area. There is an actionable nuisance on the facts and the change in the level of nuisance prevents the defendant from successfully relying on the defence of Prescription. For prescription defence to apply there must be 20 years or more of the actionable nuisance. In these facts the nuisance has only arisen recently. The fact the claimant moved to a nuisance is irrelevant. See Coventry v Lawrence [2014] UKSC 13 for example on this point. Character of the neighbourhood is a factor to be considered, though we don’t have sufficient information here.

How well did you know this?
1
Not at all
2
3
4
5
Perfectly
40
Q

A 5 year old boy is attending a funfair with his mother. While she is having their tickets checked, he wanders off and finds a large tent with cheering coming from inside it. At the entrance to the tent there is a sign stating: ‘Children must be accompanied by an adult.’ He walks into the tent and sees a tiger in a cage. He pushes his arm through the gap in the bars to pat the tiger. The tiger bites his arm, causing serious injury.

Which one of the following statements best explains whether or not the funfair owner owes the boy a duty of care?

The boy could be either a visitor or a trespasser. A duty of care will be therefore automatically imposed on the funfair owner.

The boy is a visitor because he had implied permission to be on the premises. The funfair owner therefore owes him an automatic duty of care.

The boy is a visitor because he had contractual permission to be on the premises once his mother bought the tickets. The funfair owner therefore owes him an automatic duty of care.

The boy is a trespasser because he should have been accompanied by an adult when he walked into the tent. The funfair owner therefore may owe him a duty of care.

The boy is a trespasser because he should not have put his arm into the tiger’s cage. The funfair owner will not therefore owe him a duty of care.

A

The boy is a visitor because he had contractual permission to be on the premises once his mother bought the tickets. The funfair owner therefore owes him an automatic duty of care.

Correct. The boy has contractual permission to visit the funfair as a ticket holder and so is a visitor for the purposes of s1(2) OLA 1957. His permission has not been limited by the sign that he, aged 5, will not have been able to read or understand. Following Pearson v Coleman Bros (1948) 2 KB 359 an occupier must be very clear as to the areas where visitors are denied access if he is seeking to limit their permission by area. The funfair is ‘premises’ for the purposes of s1(3) OLA 1957 and is controlled by the funfair owner, who is the ‘occupier’ for the purposes of s1(2) OLA 1957. The funfair owner therefore owes the boy an automatic duty of care under s 2(1) OLA 1957.

How well did you know this?
1
Not at all
2
3
4
5
Perfectly
41
Q

A family has owned a large country estate for 22 years, which they allow members of the public to visit upon paying an entrance fee. A woman pays the entrance fee and visits the country estate. As she is walking around the estate, she becomes distracted by her mobile phone ringing, so she does not look where she is going and she trips over a paving slab that has been left in the middle of the pathway. She suffers a broken ankle as a result.

Which statement best describes the defence available to the family in relation to the woman’s claim for her broken ankle?

The family can raise the defence of an act of a third party.

There are no defences that the family can raise.

The family can raise the defence of volenti.

The family can raise the defence of prescription.

The family can raise the defence of contributory negligence.

A

The family can raise the defence of contributory negligence.

Correct. The defence of contributory negligence is available against a visitor (section 2(3) Occupiers’ Liability Act 1957) and, on the facts, the woman has satisfied the test set out in Jones v Livox Quarries [1952] 2 QB 608.

How well did you know this?
1
Not at all
2
3
4
5
Perfectly
42
Q

An empty hospital ward is due to be refurbished and the hospital has put a sign on the door of the ward that says ‘Work in Progress - No entry under any circumstances’ and has placed a table across the doorway to prevent access. Some patients have been moving the table and opening the door to use the ward as a waiting room, as there is never enough space in the general waiting area. The hospital is aware a couple of patients have done this. A patient trips over a loose floor tile while waiting in the empty ward and breaks his arm.

Which of the following best describes whether or not the hospital is in breach of a duty of care to the patient under the Occupiers’ Liability Act 1984?

The hospital is unlikely to owe a duty of care to the patient as the risk is not one which the hospital would reasonably be expected to protect against.

The hospital is unlikely to owe a duty of care to the patient as it was not aware that anyone would be using the ward because it had prevented access.

The hospital is likely to owe a duty of care to the patient, but it is not in breach because it has taken reasonable steps to warn visitors of the danger.

The hospital is unlikely to owe a duty of care to the patient as he is not supposed to be in the empty ward.

The hospital is likely to owe a duty of care to the patient and it is in breach as it has failed to put up a sign that warns visitors of the actual danger.

Correct

A

The hospital is likely to owe a duty of care to the patient, but it is not in breach because it has taken reasonable steps to warn visitors of the danger.

Correct. The patient is a trespasser in the ward as the sign makes it clear that his presence in the ward would be objected to by the hospital - Robert Addie & Son (Collieries) Ltd v Dumbreck (1929) AC 358. The hospital will only owe a duty of care to the patient if each of the elements of s.1(3)(a)-(c) OLA 1984 are satisfied. If owed, the duty will be to take such care as is ‘reasonable in all the circumstances to see that he does not suffer injury by reason of the danger concerned’: s.1(4) OLA 1984. The hospital is aware of the danger, patients are known to be using the ward and the danger of the loose floor tile is likely to be a risk reasonable for the hospital to guard against. However, the hospital, in putting the sign on the ward door and a table in the doorway to prevent access has taken reasonable steps to warn of the danger. See s.1(5) OLA 84 and Titchener v BRB.

How well did you know this?
1
Not at all
2
3
4
5
Perfectly
43
Q

A department store needs to repaint its ladies’ toilets. They contract a decorator to do the work overnight. The next morning, although the decorator is still finishing off the work and the paint is still wet, the department store opens the ladies’ toilets as usual. A lady enters and her Chanel handbag gets covered in paint.

Which of the following best describes against whom the lady can bring a claim in occupiers’ liability under the OLA 1957?

The lady can bring a claim against the department store only, as they alone have a sufficient degree of control over the toilets.

The lady cannot bring a claim against anyone in occupiers’ liability as the toilets are not premises.

The lady can bring a claim against the department store and/or decorator, as both have a sufficient degree of control over the toilets.

The lady can bring a claim against the decorator only, as the decorator is the only person with a sufficient degree of control over the toilets.

The lady cannot bring a claim against anyone in occupiers’ liability as she has not suffered any personal injury

A

The lady can bring a claim against the department store and/or decorator, as both have a sufficient degree of control over the toilets.

Correct. The department store and decorator are multiple occupiers of the toilets (s 1(2) OLA 1957/Wheat v Lacon (1966) AC 552). The ladies’ toilets are ‘premises’ for the purposes of s 1(3) OLA 1957.

How well did you know this?
1
Not at all
2
3
4
5
Perfectly
44
Q

A client owns and runs a cafe. One of their customers sat on a broken chair which collapsed. They hit their head against the wall and suffered a serious head injury. There was a notice in the cafe reading “The owner of the cafe accepts no liability for any injury caused on the cafe premises howsoever caused.” The customer is suing the client for breach of duty under the Occupiers’ Liability Act 1957.

Which of the following statements is the most accurate advice to give the client?

The notice is an exclusion notice and will operate as an effective defence to any claim brought by the customer, as under the Unfair Contract Terms Act 1977, the notice will be deemed fair and reasonable.

The notice is an exclusion notice but is ineffective as under the Unfair Contract Terms Act 1977, a business occupier cannot exclude liability for personal injury resulting from their negligence.

The notice is an exclusion notice and will operate as an effective defence to any claim brought by the customer, as under the Consumer Rights Act 2015, the notice will be deemed fair.

The notice is an exclusion notice and will operate as an effective defence to any claim brought by the customer.

The notice is an exclusion notice but is ineffective as under the Consumer Rights Act 2015, a trader cannot exclude liability for personal injury resulting from their negligence.

A

The notice is an exclusion notice but is ineffective as under the Consumer Rights Act 2015, a trader cannot exclude liability for personal injury resulting from their negligence.

Correct
This is correct. Under s.65(1) CRA 2015 a trader (someone acting for the purposes relating to their trade, business, craft or profession) cannot exclude their liability for personal injury resulting from their negligence where the claimant is a consumer (someone acting wholly or mainly outside their trade, business, carft or profession).

How well did you know this?
1
Not at all
2
3
4
5
Perfectly
45
Q

A man buys some moisturiser. The packaging states ‘test the product on a small area of skin and wait 15 minutes before use all over the body. Do not use if a rash occurs’. The man tests the moisturiser on his arm and waits 5 minutes without any reaction. He then applies it to the rest of his body and later experiences a severe rash.

Which of the following statements most accurately reflects whether the man can successfully bring a claim in the tort of product liability under the Consumer Protection Act 1987?

The claim will likely succeed because the warning clearly explains what consequences may occur after the skin test.

The claim will likely succeed. The warning is not sufficiently clear about how long to wait and it was reasonable for the man to wait for only 5 minutes.

The claim will likely succeed because the manufacturer produced a defective product.

The claim will not likely succeed. The warning is sufficiently clear about how long to wait and it was not reasonable for the man to wait for only 5 minutes.

The claim will likely succeed because the warning is on the packaging not the product itself.

A

The claim will not likely succeed. The warning is sufficiently clear about how long to wait and it was not reasonable for the man to wait for only 5 minutes.

Correct. It seems unreasonable to only wait 5 minutes. If the manufacturer has given an adequate warning which the claimant has not followed then the manufacturer will likely successfully be able to argue that the product was not defective. The warning would be a reason for the man’s claim to fail because he was given adequate warning of what to do and what consequences to watch out for. A claim will not necessarily succeed simply because the warning was not on the product itself.

How well did you know this?
1
Not at all
2
3
4
5
Perfectly
46
Q

A woman buys a new steam cleaner. Two weeks later, while she is using it, it overheats and explodes causing irreparable damage to her wedding dress which had been hanging in the room. The dress is ruined and the wedding, due to be held the next day, has to be postponed. She purchases a replacement steam cleaner. Which of the woman’s losses are in principle recoverable if she brings a successful negligence claim against the manufacturer of the steam cleaner?

The woman will be able to claim only for the cost of replacement of the steam cleaner.

The woman will be able to claim only for the costs of postponing the wedding.

The woman will not be able to claim for the cost of replacement of the steam cleaner, the wedding dress nor the costs of postponing the wedding.

The woman will be able to claim for the damage to the dress and the costs of postponing the wedding, but not for the cost of replacement of the steam cleaner.

The woman will be able to claim for the damage to the dress, the costs of postponing the wedding and the cost of replacement of the steam cleaner

A

The woman will be able to claim for the damage to the dress and the costs of postponing the wedding, but not for the cost of replacement of the steam cleaner.

Correct. The cost of repair or replacement of an inherently defective product itself is not claimable in general negligence; this is regarded as pure economic loss. A claim can also be brought in relation to the ruined dress. It may be arguable that the wedding did not have to be postponed, just because the dress was ruined, in which case the postponement costs may not be recoverable, but the woman would be best advised to make the claim.

How well did you know this?
1
Not at all
2
3
4
5
Perfectly
47
Q

A manufacturer makes a defective car and sells it to a car dealership. The car dealership sells the car to a woman. The brakes on the car malfunction due to the defect and the woman suffers serious injuries in a car crash from which she dies several weeks later. The woman’s estate asks the car dealership for the name of the manufacturer because they have no other way of ascertaining it. The car dealership refuses to disclose the name of the manufacturer.

Which of the following statements most accurately reflects the claim(s) that the woman’s estate can currently bring in the tort of product liability under the Consumer Protection Act 1987?

The woman’s estate cannot bring any claims because claims cannot be brought on behalf of a dead person’s estate.

The woman’s estate can bring a claim against the manufacturer and the car dealership.

The woman’s estate cannot bring any claims because whilst claims can in principle be brought on behalf of a dead person’s estate, in this particular case, there is no cause of action.

The woman’s estate can only bring a claim against the manufacturer.

The woman’s estate can only bring a claim against the car dealership.

A

The woman’s estate can only bring a claim against the car dealership.

Correct
Correct. A claim can be brought against the car dealership as a supplier of the defective product that refuses to identify the manufacturer under s 2 (3) of the Consumer Protection Act 1987. A claim cannot currently be brought against the manufacturer because the identity of the manufacturer is not known. Claims can be brought on behalf of a deceased person’s estate.

How well did you know this?
1
Not at all
2
3
4
5
Perfectly
48
Q

An NHS Trust is being sued by a number of patients under the Consumer Protection Act 1987. The patients claim to have been transfused with blood infected by hepatitis in one of the Trust’s hospitals. The Trust wishes to say in its defence that it took all reasonable steps to prevent such a thing occurring, and that in any event the risk of infection was very small. The Trust wishes to know whether the blood was a defective product. Which one of the following is the best advice to give as to the question the court will ask itself when determining whether the blood was defective?

In deciding whether or not the blood was defective, the court will ask itself whether the Trust took as much care as its resources would allow to guard against infection.

In deciding whether or not the blood was defective, the court will consider that the state of scientific knowledge was not such as could have allowed the risk of infection to be discovered.

In deciding whether or not the blood was defective, the court will have regard to the expectations patients generally have a right to have.

In deciding whether or not the blood was defective, the court will examine the expectations of the individual claimants in this litigation.

In deciding whether or not the blood was defective, the court will judge the Trust against the standard of the reasonably competent NHS Trust.

A

In deciding whether or not the blood was defective, the court will have regard to the expectations patients generally have a right to have.

Correct. Under Section 3 of the Consumer Protection Act 1987 we find the test for whether a product is defective and this option is the closest to the wording of that test. Considering the other answers, it is the expectations of ‘persons generally’ which are crucial, not the expectations of the particular claimants. Neither the concept of ‘the standard of the reasonably competent NHS Trust’ nor an assessment of the Trusts resources are part of the test. The ‘state of scientific knowledge’ may be relevant to a defence, but not to considering liability in the first instance.

How well did you know this?
1
Not at all
2
3
4
5
Perfectly
49
Q

A chemist is being sued by several customers under the Consumer Protection Act 1987 who claim that a brand of the chemist’s own cough mixture has caused allergic reactions leading to fever and a painful rash. The chemist will say it has taken all reasonable care in the manufacturing process, and the medical evidence is virtually unanimous in saying that there is no known risk of such a reaction through taking this cough mixture. Only one article, published in Mandarin by a final year biochemistry undergraduate on his personal blog (which has 10,000 regular visitors) sets out research which indicates a 2% risk of such a reaction. This article was published one week ago. The chemist wishes to rely on the ‘development risks’ defence. Which one of these is the most accurate advice to give?

The defence will apply. It does not matter whether or not the blog article is considered mainstream in this case.

The defence will apply. The risk has been published and is therefore within the state of scientific knowledge.

The defence will not apply as the risk has now been discovered.

The defence might apply. The court will have to consider whether the blog article is sufficiently mainstream to bring the matter within the state of scientific knowledge.

The defence will apply as a reasonable body of medical opinion says there is no risk.

A

The defence will apply. It does not matter whether or not the blog article is considered mainstream in this case.

This is correct. The development risks defence judges the state of scientific knowledge at the time the product was put into circulation, so this article is not included in it. The discussion about whether the blog article is sufficiently mainstream to prevent the defence is therefore not relevant. Considering the other answers. The ‘reasonable body of medical opinion’ sounds more like negligence and the Bolam test, but the Consumer Protection Act is a different regime. ‘Discovering’ the risk or something being published about it is not enough to prevent the defence from applying.

How well did you know this?
1
Not at all
2
3
4
5
Perfectly
50
Q

A man wakes up with a headache and loss of sight in one eye. In order to seek treatment, he decides to drive himself to the hospital. Whilst driving, he loses sight in the other eye and drives into a pedestrian, causing them to suffer a broken leg.

What standard of care will the driver be held to?

He should be judged according to the standard of a reasonably competent driver.

He should be judged according to the standard of a reasonably competent ambulance driver, using the ‘act, not actor’ principle.

He should be judged according to the standard of a reasonably competent driver who is suffering from a condition that impairs his ability to drive.

A

He should be judged according to the standard of a reasonably competent driver.

Well done! This answer correct applies Roberts v Ramsbottom [1980] 1 All ER 7. The driver knew that he was ill when he started to drive and so he would be judged according to the standard of a reasonably competent driver.

How well did you know this?
1
Not at all
2
3
4
5
Perfectly
51
Q

A child is playing a game of football on the school playground. She runs for the ball and negligently collides with a lunchtime supervisor. What standard of care will the child be held to?

She should be judged according to the standard of a reasonably competent adult.

She should be judged according to the standard of a reasonably competent footballer, using the ‘act, not actor’ principle.

Standard of care is not relevant here because a duty of care is not imposed on a child.

She should be judged according to the standard of a reasonable child of her age.

A

Well done! This answer correctly applies Mullin v Richards [1998] 1 All ER 920 and Orchard v Lee [2009] EWCA Civ 295. Children can owe a duty of care but the standard expected of them is lower than that of an adult.

How well did you know this?
1
Not at all
2
3
4
5
Perfectly
52
Q

Which factors will the courts consider when deciding if a defendant has fallen below the standard of care?

The usual or common practice, the likelihood of harm, the magnitude of harm, the practicality of precautions, any benefit of the defendant’s conduct, special rules in relation to sport, what the ‘state of the art’ was at the time of breach, and whether the error was negligent or simply an error of judgment.

The usual or common practice, the magnitude of harm, the practicality of precautions, any benefit of the defendant’s conduct, special rules in relation to sport, and whether the error was negligent or simply an error of judgment.

The usual or common practice, the likelihood of harm, the magnitude of harm, the practicality of precautions, any benefit of the defendant’s conduct, what the ‘state of the art’ was at the time of breach, and whether the error was negligent or simply an error of judgment.

The likelihood of harm, the magnitude of harm, the practicality of precautions, any benefit of the defendant’s conduct, and special rules in relation to sport.

A

The usual or common practice, the likelihood of harm, the magnitude of harm, the practicality of precautions, any benefit of the defendant’s conduct, special rules in relation to sport, what the ‘state of the art’ was at the time of breach, and whether the error was negligent or simply an error of judgment.

Well done! The courts can take into account any of these factors if relevant.

How well did you know this?
1
Not at all
2
3
4
5
Perfectly
53
Q

Knightley v Johns [1982] 1 WLR 349

A

This case held that the act of a third party must be highly unforeseeable (i.e. something that was very unlikely to happen as a result of the defendant’s negligence) to break the chain of causation.

How well did you know this?
1
Not at all
2
3
4
5
Perfectly
54
Q

McKew v Holland & Hanmen & Cubitts (Scotland) Ltd [1969] 3 All ER 1621

A

Incorrect. This case held that an act of a claimant will break the chain of causation if it is highly unreasonable.

How well did you know this?
1
Not at all
2
3
4
5
Perfectly
55
Q

Which of the following cases held that medical negligence needs to be ‘palpably wrong’ in order to break the chain of causation?

McKew v Holland & Hanmen & Cubitts (Scotland) Ltd [1969] 3 All ER 1621

Robinson v Post Office [1974] 2 All ER 737

Carslogie Steamship Co Ltd v Royal Norwegian Government [1952] AC 292

Knightley v Johns [1982] 1 WLR 349

A

Robinson v Post Office [1974] 2 All ER 737

Correct
Well done! As a matter of policy, the courts are reluctant to hold that medical treatment breaks the chain of causation. When a defendant causes injury, he takes the risk that the claimant may not respond well to medical treatment or that the medical treatment may not be perfect.

How well did you know this?
1
Not at all
2
3
4
5
Perfectly
56
Q

Which of the following statements best describes general damages?

Damages that cover specifically provable and quantifiable financial losses.

Damages that are compensatory in nature.

Damages that cover non-quantifiable losses and future financial losses.

Damages that put the claimant in the position they would have been in but for the defendant’s tortious act.

A

Damages that cover non-quantifiable losses and future financial losses.

Well done! This is the correct description of general damages. In contrast, special damages cover specifically provable and quantifiable financial losses.

How well did you know this?
1
Not at all
2
3
4
5
Perfectly
57
Q

A passenger in a car refuses to wear a seatbelt. The driver negligently drives the car into a wall, causing the passenger to suffer a head injury. If the passenger had been wearing a seatbelt, then he would not have suffered the head injury. Which one of the following statements describes the position in relation to the defence of contributory negligence?

The defence of contributory negligence is available because the passenger failed to take reasonable care of himself and that failure contributed to his loss. His damages will be reduced.

The defence of contributory negligence is available because the passenger failed to take reasonable care of himself. His damages will be reduced.

The defence of contributory negligence is available because the passenger failed to take reasonable care of himself and that failure contributed to his loss. The passenger will therefore get no damages.

The defence of contributory negligence is not available because a driver cannot raise a defence of contributory negligence against a passenger.

The defence of contributory negligence is not available because the driver should have insisted that the passenger wear the seatbelt and so the passenger has not failed to take reasonable care of himself.

A

The defence of contributory negligence is available because the passenger failed to take reasonable care of himself and that failure contributed to his loss. His damages will be reduced.

Well done! This answer correctly applies the test of contributory negligence as set out in Jones v Livox [1952] 2 QB 608 and identifies that the passenger’s damages will be reduced as a result.

How well did you know this?
1
Not at all
2
3
4
5
Perfectly
58
Q

Lagden v O’Connor [2004] 1 AC 1067.

A

You need to apply the principle that the tortfeasor must take their victim as they find them, which applies equally to the claimant’s financial health as to their physical or mental state.

How well did you know this?
1
Not at all
2
3
4
5
Perfectly
59
Q

Lagden v O’Connor [2004] 1 AC 1067.

A

You need to apply the principle that the tortfeasor must take their victim as they find them, which applies equally to the claimant’s financial health as to their physical or mental state.

How well did you know this?
1
Not at all
2
3
4
5
Perfectly
60
Q

A driver negligently crashes his car into a woman’s front garden wall, causing significant damage. The woman could not afford to carry out the repairs to the wall immediately and as a result the wall collapses four months later. If the repairs had been carried out immediately, the cost would have been £500. The cost of rebuilding the wall will now be £1500. Which of the following statements describes what the woman can claim as her loss?

The woman can claim for property damage of £500.

The woman can claim for property damage of £1500.

The woman can claim for property damage, which the court will quantify at trial.

The woman cannot claim for anything as the loss is now due to her own lack of money rather than the driver’s tort.

A

The woman can claim for property damage of £1500.

Correct. This answer applies the principle that the tortfeasor must take their victim as they find them, which applies equally to the claimant’s financial health as to their physical or mental state. This is set out in Lagden v O’Connor [2004] 1 AC 1067.

How well did you know this?
1
Not at all
2
3
4
5
Perfectly
61
Q

Which case first established the close connection test for identifying whether an employee has committed a tort within the course of employment?

Fletcher v Chancery Lane

Smith v Stages

Mohamud v WM Morrison Supermarkets plc

Lister v Hesley Hall Ltd

A

Lister v Hesley Hall Ltd

Correct
Well done! Lord Steyn stated in this case that the correct approach “is to concentrate on the relative closeness of the connection between the nature of the employment and the particular tort”.

How well did you know this?
1
Not at all
2
3
4
5
Perfectly
62
Q

Mohamud v WM Morrison Supermarkets plc

A

Whilst this case did involve the Supreme Court applying the close connection test and it held that it is actually a two fold test, this case came after Lister v Hesley Hall (which established the test).

How well did you know this?
1
Not at all
2
3
4
5
Perfectly
63
Q

A hairdresser is employed at a hair salon. A fellow employee hairdresser at the salon has a habit of playing practical jokes on their fellow employees. The employer is aware of this but has never told them to stop. One day, the hairdresser who plays practical jokes jumps out on the other hairdresser, scaring them and causing them to fall. They injure their ankle as a result.

Which of the following statements is correct in relation to whether the employer has breached the duty it owes to the injured hairdresser?

The employer has not breached its duty towards the injured hairdresser because the injured hairdresser should bring a claim against the other hairdresser instead of the employer.

The employer has breached its duty towards the injured hairdresser because they have employed someone known to be in the habit of playing practical jokes on fellow employees.

The employer has breached its duty towards the injured woman because they have employed the other hairdresser who is incompetent.

The employer has not breached its duty towards the injured hairdresser because the injured hairdresser should have told the employer about the other hairdresser playing practical jokes.

A

The employer has breached its duty towards the injured hairdresser because they have employed someone known to be in the habit of playing practical jokes on fellow employees.

Well done! This answer correctly applies the case of Hudson v Ridge Manufacturing Company Ltd [1957] 2 QB 348 to the facts of the question.

How well did you know this?
1
Not at all
2
3
4
5
Perfectly
64
Q

A person is employed in a chocolate factory, working with melted chocolate which is incredibly hot. They have never done this sort of work before. Their employer provides them with safety gloves to wear whilst working but tells them that they only need to wear them if they want to. The person decides not to wear the safety gloves and, one day, they seriously burn their hands on the chocolate.

Which of the following statements is correct in relation to whether the employer has breached the duty it owes the injured person?

The employer has breached its duty towards the injured person because they did not encourage or even insist that they wear the safety gloves.

The employer has not breached its duty towards the injured person because the injured person decided not to wear the safety gloves.

The employer has breached its duty towards the injured person because it has not provided a safe place to work.

The employer has not breached its duty towards the injured person because the injured person knew all the risks of the job.

A

The employer has breached its duty towards the injured person because they did not encourage or even insist that they wear the safety gloves.

Well done! Given the high degree of risk of serious harm if the person did not wear the gloves because they were working with hot chocolate, the employer should have encouraged or even insisted that they wear the safety gloves (Bux v Slough Metals [1974] 1 Lloyd’s Rep 155).

How well did you know this?
1
Not at all
2
3
4
5
Perfectly
65
Q

As a result of a negligent driver, a passenger is physically unharmed but experiences a recurrence of a depressive illness. Which factor should their solicitor focus on when considering whether to make a claim for psychiatric harm?

The solicitor should consider whether a claim for psychiatric harm will fail for policy reasons

The solicitor should consider whether physical harm was reasonably foreseeable

The solicitor should consider whether the psychiatric harm was the result of the passenger witnessing a horrific event

The solicitor should make separate claims for negligence and psychiatric harm

A

The solicitor should consider whether physical harm was reasonably foreseeable

The psychiatric harm is a type of loss, but in order to decide whether the passenger can claim the solicitor has to consider whether the driver owed their client a duty of care. The first question to ask is whether physical harm is reasonably foreseeable which it would be in a road traffic accident. There is precedent for drivers owing passengers a duty of care (Nettleship v Weston).

How well did you know this?
1
Not at all
2
3
4
5
Perfectly
66
Q

A man comes across a train crash in which several passengers have been killed and injured, and assists by entering carriages and pulling surviving passengers from the burning wreck of the train. Afterwards he suffers from neurotic anxiety. Which statement explains whether he can recover damages for his neurotic anxiety (psychiatric harm)?

The rescuer cannot recover damages because psychiatric harm was not reasonably foreseeable.

There is no duty owed to rescuers when they are strangers to the victims of accidents because they do not share close ties of love and affection

The rescuer can recover damages if he can show that he was in danger himself in carrying out the rescue.

There is no duty owed to rescuers because of policy reasons to prevent the floodgates opening and to ensure that unmeritorious claims are kept away from trial

A

The rescuer can recover damages if he can show that he was in danger himself in carrying out the rescue.

The rescuer will be classed as a primary victim if he was in danger himself when carrying out the rescue. He would therefore be owed a duty of care as physical injury was reasonably foreseeable.

How well did you know this?
1
Not at all
2
3
4
5
Perfectly
67
Q

The Alcock criteria are sometimes cited as requiring you to consider: a) was the psychiatric harm reasonably foreseeable? b) Is there proximity of relationship between the claimant and the victim? c) Is there proximity in time and space? and d) Was the psychiatric harm shock-induced? In relation to (b), proximity of relationship between the claimant and the victim requires us to consider in particular which of the following?

How close (physically) the claimant and victim were.

The familial proximity of the parties ie how close they are in family terms (parent / child being closer than grandparent / grandchild, for example).

Whether the claimant was involved with / interacted with the victim in the course of the events which amounted to, or flowed from, the ‘shocking’ event.

The quality of the relationship between claimant and victim by reference to the bonds of love and affection between them.

A

The quality of the relationship between claimant and victim by reference to the bonds of love and affection between them.

Correct. This is more important than whether they are parent / child or grandparent/ grandchild etc. Physical closeness is important, but this is what the next criteria, ‘proximity in time and space’ is about.

How well did you know this?
1
Not at all
2
3
4
5
Perfectly
68
Q

Hedley Byrne v Heller & Partners provides an exception for when the courts will allow a claim for pure economic loss caused by a negligent misstatement. In this case the claim failed – what was the reason?

The claim failed because the reliance was not reasonable

The claim failed because the loss was not foreseeable.

The claim failed because the claimant was unable to show there was a special relationship

The claim failed because there was a disclaimer, so there was no voluntary assumption of responsibility

A

The claim failed because there was a disclaimer, so there was no voluntary assumption of responsibility

Correct
Well done. There was a disclaimer - this negated liability. Putting a disclaimer would probably mean that there was no voluntary assumption of responsibility, although note that now it would be subject to the UCTA 1977 and only allowed if it passed the test of reasonableness

How well did you know this?
1
Not at all
2
3
4
5
Perfectly
69
Q

Javaria sets up a face-painting stall at her garden party and provides numerous large barrels of paint for her guests to use. During the party the paint barrels are knocked over and paint leaks into Mark’s garden, ruining his organic vegetable patch. Which of the following is correct?

Mark could not bring a claim against Javaria because this was a one-off event.

Mark could bring a claim against Javaria on the grounds of private nuisance.

Mark could bring a claim against Javaria on the grounds of public nuisance.

Mark could bring a claim against Javaria under the rule in Rylands v Fletcher

A

Mark could bring a claim against Javaria under the rule in Rylands v Fletcher

Correct. The fact that Javaria brought numerous large barrels of paint onto her property means that harm to neighbouring gardens was foreseeable if the paint leaked

How well did you know this?
1
Not at all
2
3
4
5
Perfectly
70
Q

A student, at a university, uses the library facilities to revise for his PGDL exams. In previous months the library was a quiet and peaceful place in which to work. In recent weeks, the university has formally permitted the rugby society to use the room adjacent to the library to conduct fund raising activities. The fundraising events are considerably noisy, and the student can no longer revise in the library. As a result of his inability to revise adequately, the student fails his exams. The university does not offer the option to resit the exams. The student cannot qualify as a lawyer in this way. Which of the following statements represents the most appropriate advice to the student?

The student should be advised that he would have a claim in public nuisance but not in either private nuisance or under the rule in Rylands v Fletcher

The student should be advised that he would not have a valid claim in private nuisance or public nuisance but should bring the claim under the rule in Rylands v Fletcher

The student should be advised that he would have a valid claim in private nuisance and Rylands v Fletcher but not in public nuisance

The student should be advised that he would have a valid claim in private nuisance but not in public nuisance or under the rule in Rylands v Fletcher

The student should be advised that he would not have a valid claim in either private nuisance, public nuisance or under the rule in Rylands v Fletcher

A

The student should be advised that he would not have a valid claim in either private nuisance, public nuisance or under the rule in Rylands v Fletcher

Yes. This is correct. The student, as a bare licensee, does not have the requisite legal interest in land to enable him to bring a claim in either private nuisance or in Rylands v Fletcher (Hunter v Canary Wharf). As a student, he would be a bare licencee. Additionally, the student would not have a valid claim in public nuisance as there is no evidence on the facts that a class of people (AG v PYA Quarries) have suffered any discomfort or inconvenience. The noise from the fundraising activity appears to only affect the student in question.

How well did you know this?
1
Not at all
2
3
4
5
Perfectly
71
Q

Which case held that planning permission does not determine private rights and therefore planning permission is normally of no assistance to the defendant in private nuisance claims?

Ryland v Fletcher

Coventry v Lawrence

Network Rail Infrastructure Ltd v CJ Morris

Colour Quest Ltd v Total Downstream UK Plc

A

Coventry v Lawrence

Well done! That is the correct authority. Network Rail relates to the concept of ‘abnormal sensitivity’. Rylands v Fletcher established the rule that uses the same name. Colour Quest relates to public nuisance.

How well did you know this?
1
Not at all
2
3
4
5
Perfectly
72
Q

Javaria decides to hold a party in her garden to celebrate her 40th birthday. She invites 100 guests and has a live band playing all night. Many of Javaria’s neighbours complain that they could not sleep because of the noise, which could be heard up to 3 miles away. Mark, who runs a bed and breakfast next door, finds that his guests refuse to pay because of the disruption. Which one of the following correctly describes the legal position?

The neighbours will have a claim in public nuisance. Mark will have an individual claim in public nuisance as he has been impacted above and beyond the rest of the class. He would not have a claim in private nuisance

The neighbours will have a claim in public nuisance. Mark will have an individual claim in private nuisance

As this is a one-off event neither the neighbours nor Mark can bring a claim in nuisance

The neighbours will have a claim in public nuisance. Mark will have no claim

A

The neighbours will have a claim in public nuisance. Mark will have an individual claim in public nuisance as he has been impacted above and beyond the rest of the class. He would not have a claim in private nuisance

Correct. Mark will not have a claim in private nuisance here as this is a one-off event rather than a continuous state of affairs

How well did you know this?
1
Not at all
2
3
4
5
Perfectly
73
Q

Which one of the following correctly describes an occupier for the purposes of Occupiers’ Liability?

The occupier is the person who has a sufficient degree of control over the property

The occupier is the person in physical occupation of the property

The occupier is the tenant of the property

The occupier is the owner of the property

A

The occupier is the person who has a sufficient degree of control over the property

Correct, this is the key principle set out in Wheat v Lacon. The rationale is that those who have a sufficient degree of control over premises should also have a duty of care placed upon them in relation to the safety of those on the premises. The occupier is not necessarily the owner or tenant, and ‘physical occupation’ is not a useful definition.

How well did you know this?
1
Not at all
2
3
4
5
Perfectly
74
Q

Tom is having a beer at a pub with a friend. He goes to find the toilet facilities and walks through a door marked ‘Private: Staff Only’. Tom goes on to walk down the staircase and falls due to a faulty hand rail. He is seriously injured in the fall. Which of the following best explains whether Tom is protected by either Occupiers’ Liability Act, and why?

Tom has no protection under either of the occupiers’ liability acts because he has been consuming alcohol

Tom has express permission to be on the premises as a customer of the pub and is therefore a visitor, protected by the Occupiers’ Liability Act 1957

Tom has express permission to be on the premises as a customer of the pub and is therefore a visitor, protected by the Occupiers’ Liability Act 1984

By walking into the staff-only area of the pub Tom has become a trespasser, protected under the Occupiers’ Liability act 1984

A

By walking into the staff-only area of the pub Tom has become a trespasser, protected under the Occupiers’ Liability act 1984

Correct. The sign on the door means that, beyond the door, Tom is a trespasser, but Tom may still be protected as a trespasser under the Occupiers’ Liability Act 1984.

How well did you know this?
1
Not at all
2
3
4
5
Perfectly
75
Q

Lila, a twelve-year old girl, is having Sunday lunch at a pub with her family. At the back of the pub there is a playground area, including an old, rusty slide. Lila sees the slide and begins to climb it. The slide gives way beneath her and she falls, breaking her arm. Which one of the following best explains the duty of care that the pub owed to Lila?

A duty to take such care as is reasonable to see that she will be safe whilst on the premises, as applies to all visitors whether children or adults

No duty of care is owed as the slide was old and rusty and Lila should therefore have known not to use it

A duty to take such care as is reasonable to see that she will be safe whilst on the premises, bearing in mind that children are less careful than adults

A duty akin to a parental role, ensuring that Lila is safe whilst on the premises

Correct
Correct. Under s 2(3)(a) of the Occupiers’ Liability Act 1957 a higher standard of care is owed to children as occupiers must be prepared for children to be less careful than adults. It would be wrong, however, to equate the occupier’s duty to that of a parent. The fact that the slide was old an rusty would not mean that no duty of care was owed, nor would it necessarily make the danger obvious to a child.

A

A duty to take such care as is reasonable to see that she will be safe whilst on the premises, bearing in mind that children are less careful than adults

Correct. Under s 2(3)(a) of the Occupiers’ Liability Act 1957 a higher standard of care is owed to children as occupiers must be prepared for children to be less careful than adults. It would be wrong, however, to equate the occupier’s duty to that of a parent. The fact that the slide was old an rusty would not mean that no duty of care was owed, nor would it necessarily make the danger obvious to a child.

How well did you know this?
1
Not at all
2
3
4
5
Perfectly
76
Q

Which of the following correctly summarises the link between a claim in negligence and a claim under the Consumer Protection Act 1987?

A claim under the Consumer Protection Act 1987 is a type of negligence claim. The elements of a claim in negligence should be considered first, followed by the requirements under the act.

Negligence provides protection in relation to financial losses (other than damage to property). The Consumer Protection act 1987 provides protection in relation to property damage and personal injury.

If a duty is owed under the Consumer Protection Act 1987, a duty will be owed in negligence, but to succeed in negligence, breach must also be shown.

These are two separate causes of action. Both, one or neither may be available. The elements of each cause of action should be considered separately.

Where a party is liable in negligence in relation to a defective product, then that party will also be liable under the Consumer Protection Act 1987

A

These are two separate causes of action. Both, one or neither may be available. The elements of each cause of action should be considered separately.

Correct. The other answers are incorrect, or gross simplifications.

How well did you know this?
1
Not at all
2
3
4
5
Perfectly
77
Q

Roe v Minister of Health [1954] 2 QB 66.

A

‘state of art’ defence: The courts must assess the doctor’s actions against the knowledge in the profession and/or accepted practice at the time of the alleged breach.

How well did you know this?
1
Not at all
2
3
4
5
Perfectly
78
Q

Which one of the following cases held that the court can “reach the conclusion that views held by a competent medical expert are unreasonable”?

Maynard v West Midlands RHA [1985] 1 WLR 634

Bolitho v City and Hackney Health Authority [1997] 4 All ER 771

De Freitas v O’Brien and Connolly [1995] 6 Med LR 108

Bolam v Friern Hospital Management Committee [1957] 2 All ER 118

A

Bolitho v City and Hackney Health Authority [1997] 4 All ER 771

Well done! It should be noted though that Lord Browne-Wilkinson acknowledged that it would only be in ‘rare’ or exceptional cases that judicial intervention could be justified.

How well did you know this?
1
Not at all
2
3
4
5
Perfectly
79
Q

Which factors will the courts consider when deciding if a defendant has fallen below the standard of care?

The usual or common practice, the likelihood of harm, the magnitude of harm, the practicality of precautions, any benefit of the defendant’s conduct, what the ‘state of the art’ was at the time of breach, and whether the error was negligent or simply an error of judgment.

The likelihood of harm, the magnitude of harm, the practicality of precautions, any benefit of the defendant’s conduct, and special rules in relation to sport.

The usual or common practice, the magnitude of harm, the practicality of precautions, any benefit of the defendant’s conduct, special rules in relation to sport, and whether the error was negligent or simply an error of judgment.

The usual or common practice, the likelihood of harm, the magnitude of harm, the practicality of precautions, any benefit of the defendant’s conduct, special rules in relation to sport, what the ‘state of the art’ was at the time of breach, and whether the error was negligent or simply an error of judgment.

A

The usual or common practice, the likelihood of harm, the magnitude of harm, the practicality of precautions, any benefit of the defendant’s conduct, special rules in relation to sport, what the ‘state of the art’ was at the time of breach, and whether the error was negligent or simply an error of judgment.

Well done! The courts can take into account any of these factors if relevant.

How well did you know this?
1
Not at all
2
3
4
5
Perfectly
80
Q

Whitehouse v Jordan [1980] 1 All ER 650 sets out which principle relevant to breach?

The more likely someone is to get injured, the more likely it is that the court will find a breach.

If it would be unreasonable to require a defendant to take necessary precautions, the court will not impose liability.

The court must make a distinction between an error of judgment and actual negligence.

The courts must assess the doctor’s actions against the knowledge in the profession and/or accepted practice at the time of the alleged breach.

A

The court must make a distinction between an error of judgment and actual negligence.

Well done! This principle was discussed by Lord Fraser.

How well did you know this?
1
Not at all
2
3
4
5
Perfectly
81
Q

What is the burden of proof on the claimant in relation to the ‘but for’ test?

The claimant needs to prove their case on the balance of probabilities, i.e. that there is a more than 50 per cent chance that the defendant’s breach caused the loss.

The claimant needs to prove that there is a more than 40% chance that the defendant’s breach caused the loss.

The claimant needs to prove their case beyond reasonable doubt, i.e. that it is beyond reasonable doubt that the defendant’s beach caused the loss.

The claimant needs to fully prove their case, i.e. that there is a 100% chance that the defendant’s breach caused the loss.

A

The claimant needs to prove their case on the balance of probabilities, i.e. that there is a more than 50 per cent chance that the defendant’s breach caused the loss.

Well done! This is the correct burden of proof given that negligence is part of the civil law.

How well did you know this?
1
Not at all
2
3
4
5
Perfectly
82
Q

Carslogie Steamship Co Ltd v Royal Norwegian Government [1952] AC 292

A

This case held that an act of God breaks the chain of causation if it is some exceptional natural event. Natural events will not break the chain of causation if they could have been foreseen and the claimant should have taken them into account as events that were likely to happen.

How well did you know this?
1
Not at all
2
3
4
5
Perfectly
83
Q

Wagon Mound (No.1) [1961] AC 388

A

This case sets out the test for remoteness, i.e. a claimant can only recover if they can show that the damage that they have suffered was reasonably foreseeable at the time that the defendant breached the duty of care owed to the claimant.

How well did you know this?
1
Not at all
2
3
4
5
Perfectly
84
Q

A passenger in a car was injured due to the driver’s negligent driving. The car was found to contain a large packet of cocaine which the driver and passenger were planning to sell later.

Which one of the following statements describes the position in relation to the defence of illegality?

The defence of illegality is available because the driver and the passenger were planning to sell cocaine. The passenger will therefore get no damages.

The defence of illegality is not available because the passenger has not committed an illegal act.

The defence of illegality is not available because there was not a direct link between the passenger’s injury and the illegal act of planning to sell cocaine.

The defence of illegality is available because the driver and the passenger were planning to sell cocaine. The passenger’s damages would be reduced.

A

The defence of illegality is not available because there was not a direct link between the passenger’s injury and the illegal act of planning to sell cocaine.

Well done! Illegality would not apply here as there is no direct link between the passenger’s injury and the illegal act (Delaney v Pickett [2011] EWCA Civ 1532).

How well did you know this?
1
Not at all
2
3
4
5
Perfectly
85
Q

Lagden v O’Connor [2004] 1 AC 1067

A

This case held that the ‘thin skull’ rule applies even if the damage has been aggravated by the claimant’s own lack of monetary funds.

How well did you know this?
1
Not at all
2
3
4
5
Perfectly
86
Q

Which of the following cases held that once it has been established that the type of damage is reasonably foreseeable, there is no need to foresee the exact way in which the damage occurred?

Lagden v O’Connor [2004] 1 AC 1067

Wagon Mound (No.1) [1961] AC 388

Hughes v Lord Advocate [1963] AC 837

Vacwell Engineering v BDH Chemicals [1971] 1 QB 88

A

Hughes v Lord Advocate [1963] AC 837

Well done! Once it has been established that the type or kind of damage is reasonably foreseeable, then there is no need for it also to be reasonable for the defendant to foresee the exact circumstances leading up to the damage. Overwhelmingly, subsequent cases have followed this approach.

How well did you know this?
1
Not at all
2
3
4
5
Perfectly
87
Q

Which case established the ‘multiple factors’ or ‘economic reality’ test for determining whether an employee/employer relationship exists?

Viasystems Ltd v Thermal Transfer Ltd and Others [2005] EWCA Civ 1151

Mersey Docks and Harbour Board v Coggins and Griffiths [1947] AC 1

Ready Mixed Concrete v Minister of Pensions [1968] 2 QB 497

Barclays Bank v Various Claimants [2020] UKSC 13

A

Ready Mixed Concrete v Minister of Pensions [1968] 2 QB 497

Well done! In this case, the court indicated that there is a three-part approach to take when considering whether there is an employment relationship: (i) remuneration in exchange for personal services and mutuality of obligations; (ii) control; and (iii) all other contractual factors consistent with an employment relationship.

How well did you know this?
1
Not at all
2
3
4
5
Perfectly
88
Q

Barclays Bank v Various Claimants [2020] UKSC 13

A

This case establishes that a party who is clearly carrying on their own independent business is not in a relationship akin to employment, and so an employer cannot be vicariously liable for any torts that they commit.

How well did you know this?
1
Not at all
2
3
4
5
Perfectly
89
Q

A primary victim is….

…someone who is first to suffer an injury as a result of the tortious act concerned.

…someone who suffers psychiatric harm as a result of reasonable fear for their own physical safety.

…someone ‘outside’ the danger zone but foreseeably at risk of injury.

…someone who suffers a psychiatric injury which is foreseeable to the tortfeasor.

A

…someone who suffers psychiatric harm as a result of reasonable fear for their own physical safety.

Well done! The other answers are wrong because: ‘primary’ is not a reference to being first in time; whether someone is a primary victim is not really a question of foreseeability of psychiatric harm; and a primary victim is inside the danger zone, not outside it.

How well did you know this?
1
Not at all
2
3
4
5
Perfectly
90
Q

Hinz v Berry is authority for the proposition that…

…secondary victims can only claim in more limited circumstances than primary victims.

…depression is not a medically recognised psychiatric injury.

…depression is not a foreseeable consequence of a traumatic event.

…sorrow is not a medically recognised psychiatric injury.

A

…sorrow is not a medically recognised psychiatric injury.

Correct. In contrast, depression is a recognised psychiatric injury. The proposition that “secondary victims can only claim in more limited circumstances than primary victims” is true, but Hinz v Berry is not a good authority for this proposition. The proposition that “depression is not a foreseeable consequence of a traumatic event” is simply incorrect. A party can only recover in relation to medically recognised psychiatric injury, not for ‘sorrow’.

…secondary victims can only claim in more limited circumstances than primary victims.
Incorrect. This proposition is true, but Hinz v Berry is not a good authority for this proposition. Revisit this element.

How well did you know this?
1
Not at all
2
3
4
5
Perfectly
91
Q

Pure economic loss is best defined as….

Loss directly flowing from the breach of duty concerned.

Economic loss that arises where the duty was specifically to protect from financial (or other non-physical) harm.

Economic loss that arises where there has been no damage to the claimant’s property or injury to their person.

Financial harm suffered as a result of the destruction of physical goods or of personal injury.

A

Economic loss that arises where there has been no damage to the claimant’s property or injury to their person.

Well done!

How well did you know this?
1
Not at all
2
3
4
5
Perfectly
92
Q

Which case is the best authority for the proposition that the general rule is that no duty of care is owed in respect of pure economic loss?

Page v Smith

Hedley Byrne v Heller

Caparo v Dickman

Spartan Steel

A

Spartan Steel

Correct
Well done. Although Hedley Byrne v Heller and Caparo Dickman could arguably be authorities for this proposition, Spartan Steel is most directly relevant, and therefore it is the best choice.

How well did you know this?
1
Not at all
2
3
4
5
Perfectly
93
Q

Which one of the following correctly describes the differences between private and public nuisance?

Private nuisance is actionable per se, public nuisance is not.

Private nuisance requires a continuous state of affairs causing interference with the Claimant’s use or enjoyment of land, whereas a public nuisance can be a one-off isolated event.

Claims in private nuisance can be brought by an individual, whereas public nuisance claims can only be brought by an attorney general or local authority.

Private nuisance is a crime as well as a tort. Public nuisance is a tort only.

A

Private nuisance requires a continuous state of affairs causing interference with the Claimant’s use or enjoyment of land, whereas a public nuisance can be a one-off isolated event.

How well did you know this?
1
Not at all
2
3
4
5
Perfectly
94
Q

Where the defendant’s activity is being carried out on the basis of statutory authority, then….

The defendant will escape liability, provided they comply with the precise wording of the statute concerned.

this may be relevant to other causes of action, but not to nuisance.

the defendant will escape liability for private nuisance if they have exercised all due care and the nuisance is an inevitable consequence of the activity.

The defendant may be still be liable, but any damages will be assessed at nil.

A

the defendant will escape liability for private nuisance if they have exercised all due care and the nuisance is an inevitable consequence of the activity.

Correct. Well done!

How well did you know this?
1
Not at all
2
3
4
5
Perfectly
95
Q

The duty under the Occupiers’ Liability Act 1957 is a duty to…

…take reasonable care to ensure that the premises are free from defects or dangers not obvious on reasonable inspection.

…take reasonable care to ensure the visitor is reasonably safe in using the premises for the purposes for which they are permitted to be there.

…take reasonable steps to ensure that visitors, whilst on the premises with permission, do not suffer foreseeable harm.

…ensure the premises are safe for reasonable use.

A

…take reasonable care to ensure the visitor is reasonably safe in using the premises for the purposes for which they are permitted to be there.

Correct. See s 2(2) of the Act.

How well did you know this?
1
Not at all
2
3
4
5
Perfectly
96
Q

Which of the following best summarises how warning notices might help avoid liability in relation to trespassers?

The duty to take reasonable care might be satisfied by a warning notice which warns of the particular risk, or which discourages people from trespassing.

The duty to take reasonable care might be satisfied by a warning notice which discourages people from trespassing.

A warning notice has no impact on duty / breach as far as trespassers are concerned.

The duty to take reasonable care might be satisfied by a warning notice which warns of the particular risk.

A

The duty to take reasonable care might be satisfied by a warning notice which warns of the particular risk, or which discourages people from trespassing.

Correct. See s 1(5) of the OLA 1984.

How well did you know this?
1
Not at all
2
3
4
5
Perfectly
97
Q

The foreseeability of harm is very often part of the test for a duty of care. It is sometimes said that the test of foreseeability is ‘objective’. This means we ask:

What the claimant foresaw

What a reasonable person would consider appropriate circumstances to impose a duty.

What a reasonable person could have been expected to foresee.

What the defendant foresaw

A

This is what we mean when we say that the test of foreseeability is objective.

To consider what the defendant (or indeed the claimant) foresaw would be to adopt a subjective test – to look at what the subjects themselves (the defendant or claimant) foresaw. This is not correct.

To consider what a reasonable person would consider appropriate circumstances to impose a duty would be an ‘objective’ test, but it throws out the idea of foreseeability all together. This is not how the court approaches the question of whether to impose a duty.

How well did you know this?
1
Not at all
2
3
4
5
Perfectly
98
Q

In Caparo Industries v Dickman Lord Bridge indicated that the necessary ingredients in a situation giving rise to a duty of care are:

Foreseeability of damage, a relationship of proximity between the parties, and that it be fair, just and reasonable to impose a duty.

Foreseeability of damage, a relationship of proximity between the parties, and that it be economically productive to impose a duty.

Knowledge of the likelihood of harm, a relationship of proximity between the parties, and that it be fair, just and reasonable to impose a duty.

Foreseeability of damage, an assumption of duty by one party to the other, and that it be fair, just and reasonable to impose a duty.

A

Foreseeability of damage, a relationship of proximity between the parties, and that it be fair, just and reasonable to impose a duty.

How well did you know this?
1
Not at all
2
3
4
5
Perfectly
99
Q

Which seminal case established the general principle that that you must take reasonable care to avoid acts or omissions which you can reasonably foresee would be likely to injure someone who is so closely and directly affected by your act that you ought reasonably to have them in mind when considering how to act?

Caparo Industries v Dickman

Cassidy v Ministry of Health

Robinson v Chief Constable of West Yorkshire Police

Donoghue v Stevenson

Watson v British Boxing Board of Control

A

Donoghue v Stevenson

This case established an important principle, but later case law now provides a better indication as to how you should determine whether a duty of care is owed. The other cases listed are important cases in relation to duty of care, but are not cases that established the principle set out in the question.

How well did you know this?
1
Not at all
2
3
4
5
Perfectly
100
Q

The general rule is that…

…no liability is imposed on a mere failure to act

…liability is imposed on a failure to act on the same principles as it is imposed in relation to acts.

A

…no liability is imposed on a mere failure to act

Correct. This is the general rule (Smith v Littlewoods Organisation Limited [[1987] AC 241), but of course there are many exceptions.

How well did you know this?
1
Not at all
2
3
4
5
Perfectly
101
Q

Reeves v Commissioner of Police for the Metropolis [1999] 3 All ER 897 held that the police had a duty to take action (ie to avoid omissions) to protect a prisoner’s health, including protecting him from self-harm, because…

Statute imposed such a duty.

The police had a high degree of control over the defendant.

The defendant was a vulnerable person at significant risk of harm.

The police were in a relevant contractual relationship with the defendant.

A

The police had a high degree of control over the defendant.

This is the correct answer. The other answers are incorrect. Whilst statutes can require action and this can give rise to a duty in tort, this is not at the heart of this particular case. Similarly, a tortious duty to act can arise from a contractual relationship, but this was not relevant in this case – there was no contract between police and prisoner. The defendant may indeed have been vulnerable, but this is not really the important point in the case. Had the defendant been vulnerable but not in the police’s control, it is unlikely that they would have had the same duty to act.

How well did you know this?
1
Not at all
2
3
4
5
Perfectly
102
Q

Which of the following has a positive duty to respond to emergency calls?

The ambulance service

The police

The fire service

A

The ambulance service

Correct. The ambulance service has a positive duty to respond to emergency calls. The answering by the service of a call creates sufficient proximity between the caller and the service. Note, however, that even though the service has a duty to respond, it will not necessarily breach that duty if it responds late due to the unavailability of ambulances. The other answers are incorrect. Neither the police nor the fire service owe a duty to respond to emergency calls: but if they do respond, they may well be under a duty not to make the situation worse.

How well did you know this?
1
Not at all
2
3
4
5
Perfectly
103
Q

The police are transporting two dangerous criminals in a police van to be interviewed about a recent murder. The criminals escape from the van and seriously injure a passer-by in the process.

Which of the following statements is most accurate in relation to whether a duty of care is owed by the police to the passer-by?

It is unlikely a duty of care would be owed. The criminals were not under the care and control of the police at the time the passer-by was injured, and the police had not assumed responsibility for the welfare of the passer-by. The passer-by was an unidentifiable member of a massive group.

It is unlikely a duty of care would be owed. Whilst the criminals were under the care and control of the police at the time the passer-by was injured, the police had not assumed responsibility for the welfare of the passer-by. The passer-by was an unidentifiable member of a massive group.

It is likely a duty of care would be owed. The criminals were under the care and control of the police at the time the passer-by was injured, and the passer-by was an identifiable member of a small group at risk over and above the public at large.

It is likely a duty of care would be owed. The criminals were under the care and control of the police at the time the passer-by was injured, and the police assume responsibility for the safety of the public.

A

It is unlikely a duty of care would be owed. Whilst the criminals were under the care and control of the police at the time the passer-by was injured, the police had not assumed responsibility for the welfare of the passer-by. The passer-by was an unidentifiable member of a massive group.

How well did you know this?
1
Not at all
2
3
4
5
Perfectly
104
Q

In the case of Stansbie v Troman the defendant decorator owed the claimant a duty of care for failing to secure the claimant’s property, which allowed for a third party to break in and commit a burglary. On what basis did the defendant (Troman) owe the claimant (Stansbie) a duty of care?

There was sufficient proximity because the claimant was an identifiable victim at risk of damage over and above the public at large.

The defendant had assumed responsibility for the claimant’s welfare.

There was sufficient proximity between the claimant and defendant because the defendant knew the claimant – he was working in the claimant’s property.

There was sufficient proximity between the claimant and defendant because of the contractual relationship between the parties. In addition, the defendant created the danger by failing to secure the property.

A

There was sufficient proximity between the claimant and defendant because of the contractual relationship between the parties. In addition, the defendant created the danger by failing to secure the property.

Correct. Two of the exceptions to the general rule in relation to acts of third parties were satisfied and so the courts held that a duty of care was owed (sufficient proximity between the parties and creation of the danger).

How well did you know this?
1
Not at all
2
3
4
5
Perfectly
105
Q

The general rule is that there is no duty owed for a failure to prevent a third-party causing harm. However, there are a number of exceptions to this rule which are:

There is sufficient proximity between the defendant and claimant; and/or
There is sufficient proximity between the defendant and third party; and/or
The defendant created the danger; and/or
The risk was on the defendant’s premises.

Where the defendant assumes responsibility for the claimant.
Where the defendant has sufficient control over the claimant.
Where the defendant creates the risk.

There is sufficient proximity between the defendant and claimant; and
There is sufficient proximity between the defendant and third party; and
The defendant created the danger; and
The risk was on the defendant’s premises.

Where there is statutory duty.
Where there is contractual duty.
Where the defendant has sufficient control over the claimant.
Where the defendant creates the risk.

A

There is sufficient proximity between the defendant and claimant; and/or
There is sufficient proximity between the defendant and third party; and/or
The defendant created the danger; and/or
The risk was on the defendant’s premises.

Correct. These are the four exceptions to the general rule.

How well did you know this?
1
Not at all
2
3
4
5
Perfectly
106
Q

When considering whether public bodies owe a duty of care, the starting point should be…

…that public bodies do not owe a duty of care, other than in exceptional circumstances.

…to find precedents that concern public bodies, and to apply those.

…to apply principles of public law.

…to apply the same principles that are applicable to private individuals.

A

…to apply the same principles that are applicable to private individuals.

Correct. This should be your starting point.

For this reason, it would be wrong to start with precedents that concern public bodies, as this would be to take too restrictive an approach. It is wrong to say that public bodies owe a duty of care only in exceptional circumstances, because the starting point is that they owe a duty when a private individual would. Public law and the law of tort are two separate things, and are best considered separately.

How well did you know this?
1
Not at all
2
3
4
5
Perfectly
107
Q

A public body has a power to repair a highway. It fails to do so, and a cyclist suffers an injury as a result. Which of the following is the best way to analyse whether the public body owes a duty of care?

The public body is likely to owe a duty to repair the highway because choosing not to do so was an act which would foreseeably cause injury.

The public body will owe a duty of care to repair the highway because it had a power to repair the highway, and the failure to do so would foreseeably cause injury.

The public body is unlikely to owe a duty to repair the highway unless the normal principles applying to individuals would suggest liability for omitting to do so.

A

The public body is unlikely to owe a duty to repair the highway unless the normal principles applying to individuals would suggest liability for omitting to do so.

Correct
This is the correct answer.

The other answers are incorrect.

The case of CN and GN v Poole Borough Council [2019] UKSC 25 makes clear that when looking at omissions, the usual law applying to individuals would again apply, and it would be very hard to argue that because a public body has a duty or power to act in a particular area, that a failure to act would give rise to a duty in negligence.

For this reason, it would be wrong to say that the public body will owe a duty of care to repair the highway because it had a power to do so.

It would be wrong to say that not repairing the highway is an act. It is an omission.

How well did you know this?
1
Not at all
2
3
4
5
Perfectly
108
Q

We often talk about ‘policy considerations’ in various contexts, including when considering whether a public body should owe a duty of care. When we talk about the ‘floodgates’ policy consideration, we mean concern that…

…imposing a duty of care now would help to avoid undesirable practices by the public body in the future

… imposing a duty of care would lead to so many claims that the public body might be financially or practically overwhelmed.

… imposing a duty of care would lead to a significant increase in the number of claims against the public body.

… imposing a duty of care would lead to a release of money to successful claimants in a way that would make it difficult for the claimants to manage that money.

A

… imposing a duty of care would lead to so many claims that the public body might be financially or practically overwhelmed.

How well did you know this?
1
Not at all
2
3
4
5
Perfectly
109
Q

Blyth v Birmingham Waterworks tells us that a defendant must….

… act as a prudent and reasonable person would.

…behave to the best of their abilities.

…avoid causing harm to those to whom they owe a duty.

A

… act as a prudent and reasonable person would.

This is the general rule that a defendant must behave as a reasonable person would in all the circumstances.

How well did you know this?
1
Not at all
2
3
4
5
Perfectly
110
Q

Which seminal case established that the standard of care expected of someone working in a professional capacity is that “of the ordinary reasonable man exercising and professing to have that special skill”.

Mullin v Richards

Caparo Industries v Dickman

Bolam v Friern Hospital Management Committee

Roberts v Ramsbottom

A

Bolam v Friern Hospital Management Committee

How well did you know this?
1
Not at all
2
3
4
5
Perfectly
111
Q

The most important thing to learn from Nettleship v Weston is that:

A person will be required to meet the standard appropriate for the act they are carrying out.

A person will be required to act with all reasonable care and skill.

No allowance is made for the fact that someone is learning to do something when determining the standard of care they are required to meet.

A

A person will be required to meet the standard appropriate for the act they are carrying out.

In Nettleship v Weston the court held that a learner driver was required to meet the standard of the ordinarily competent driver. No allowance was made for her lack of driving experience.

How well did you know this?
1
Not at all
2
3
4
5
Perfectly
112
Q

When determining if there has been a breach of duty, you must first ascertain the standard of care, and secondly ascertain if the defendant has fallen below that standard. Which of the following statements is correct in relation to those issues?

Ascertaining the standard of care and ascertaining whether the defendant has fallen below it are both questions of fact.

Ascertaining the standard of care and ascertaining whether the defendant has fallen below it are both questions of law.

Ascertaining the standard of care is a question of fact, ascertaining whether the defendant has fallen below it is a question of law.

Ascertaining the standard of care is a question of law, ascertaining whether the defendant has fallen below it is a question of fact.

A

Ascertaining the standard of care is a question of law, ascertaining whether the defendant has fallen below it is a question of fact.

Well done. This means that when considering whether the defendant has fallen below a given standard of care, precedents can give you useful principles, but each case will turn on its facts.

How well did you know this?
1
Not at all
2
3
4
5
Perfectly
113
Q

If a defendant was to be able to show that he or she acted in accordance with usual or common practice, this would…

…not be relevant when determining breach.

…make it less likely that the defendant breached his or her duty.

…show that the defendant did not breach its duty.

A

…make it less likely that the defendant breached his or her duty.

Well done. This would help the defendant, but it is still possible that a defendant has been negligent even if acting in accordance with usual or common practice, for example if the common practice was illogical: see Re Herald of Free Enterprise.

How well did you know this?
1
Not at all
2
3
4
5
Perfectly
114
Q

In which one of the following cases was the defendant held to have fallen below the required standard of care?

Latimer v AEC Ltd

Bolton v Stone

Watson v BBBC

Mullin v Richards

A

Watson v BBBC

Correct. The Court of Appeal concluded that the defendant had fallen below the standard of care expected in this case.

How well did you know this?
1
Not at all
2
3
4
5
Perfectly
115
Q

Which of the following best expresses the meaning of the ‘state of the art’ defence?

The defendant will have a defence to a claim if there were others in the field / profession acting as it did.

The courts must assess the defendant’s actions against the best practice (‘state of the art’ practice) in the particular field / profession concerned.

The courts must assess the defendant’s actions against the knowledge in the profession and/or accepted practice at the time of the alleged breach.

A

The courts must assess the defendant’s actions against the knowledge in the profession and/or accepted practice at the time of the alleged breach.

Correct
Well done. It is wrong to say that the courts must assess the defendant’s actions against the best practice, because a defendant is not expected to be a ‘paragon of circumspection’ ie as cautious possible. It is wrong to say that the defendant will have a defence to a claim if there were others in the field / profession acting as it did, because compliance with usual / common practice alone will not prevent a defendant from being in breach, although it is a relevant consideration.

How well did you know this?
1
Not at all
2
3
4
5
Perfectly
116
Q

According to the case of Bolam v Friern HA, which of the following best describes the test for determining whether the defendant has fallen below the professional standard of care?

A professional is not guilty of negligence if they followed common practice in their particular field.

A professional is not guilty of negligence if they have acted in accordance with a practice accepted as proper by a responsible body of professionals skilled in that particular art.

A professional is not guilty of negligence if another professional in their field supports the practice they adopted.

A

A professional is not guilty of negligence if they have acted in accordance with a practice accepted as proper by a responsible body of professionals skilled in that particular art.

How well did you know this?
1
Not at all
2
3
4
5
Perfectly
117
Q

An approach will be in accordance with a responsible body of opinion if:

At least 50 per cent of professionals in the field concerned have that opinion.

At least 1 per cent of professionals in the field concerned have that opinion.

At least 10 per cent of professionals in the field concerned have that opinion.

It is not possible to say a specific percentage would constitute a responsible body of opinion.

A

It is not possible to say a specific percentage would constitute a responsible body of opinion.

Well done. The answers which give a specific percentage are wrong for two reasons. Firstly, the courts have not fixed a percentage on ‘responsible body’. Secondly, whatever percentage of people hold an opinion, the court must separately consider whether that opinion is responsible. Even in a professional context, the court can find a common professional practice to be negligent.

How well did you know this?
1
Not at all
2
3
4
5
Perfectly
118
Q

Which of the following best summarises the key learning point from Montgomery v Lanarkshire Health Board?

Medical professionals are generally obliged to tell patients about material risks involved in any recommended treatment and of any reasonable alternative treatments in any areas which the patient has expressed an interest in.

Medical professionals are generally obliged to advise patients on all risks and alternative treatments.

Medical professionals are generally obliged to advise patients on those material risks and alternative treatments in a manner consistent with a responsible body of medical opinion.

Medical professionals are generally obliged to tell patients about material risks involved in any recommended treatment and of any reasonable alternative treatments.

A

Medical professionals are generally obliged to tell patients about material risks involved in any recommended treatment and of any reasonable alternative treatments.

Correct
Well done. This is exactly what the court said. When it comes to advising in relation to risks, the court has determined what approach a medical professional should take, rather than leaving this to the medical professionals to determine: it is not enough to act in a manner consistent with a responsible body of medical opinion. It is probably true that a medical professional must advise in relation to areas the patient has expressed an interest in, but this alone would not be sufficient. A medical professional is not obliged to advise on all risks and alternative treatments – this would impose too great a burden on medical professionals and would be unhelpful for patients.

How well did you know this?
1
Not at all
2
3
4
5
Perfectly
119
Q

Which of the following sentences correctly sets out that the ‘but for’ test is satisfied?

But for the defendant’s breach, the claimant would not have suffered their loss at that time and in that way.

But for the defendant’s duty, the claimant would have suffered their loss at that time and in that way.

But for the defendant’s duty, the claimant would not have suffered their loss at that time and in that way.

But for the defendant’s breach, the claimant would have suffered their loss at that time and in that way.

But for the claimant’s loss, the claimant would not succeed in the tort of general negligence.

A

But for the defendant’s breach, the claimant would not have suffered their loss at that time and in that way.

This is the correct answer. In order to establish factual causation, the claimant must prove on the balance of probabilities, that if it were not for the defendant’s breach, they would not have suffered their loss at that time and in that way.

How well did you know this?
1
Not at all
2
3
4
5
Perfectly
120
Q

A patient attends their doctor with chest pain. The doctor briefly examines the patient and tells them to stay in the waiting room, where the patient has a heart attack and dies. Medical evidence suggests that if the doctor had treated the patient properly, there was a 45% chance of living.

Which of the following statements is correct in relation to the ‘but for’ test?

The ‘but for’ test is not satisfied as there was a 55 per cent chance the patient would have died without the breach.

The ‘but for’ test is satisfied as there was only a 55 per cent chance the patient would have lived without the breach.

The ‘but for’ test is not satisfied as there was a 45 per cent chance the patient would have died without the breach.

The ‘but for’ test is not satisfied as there was not a 100 per cent chance that the patient would have lived without the breach.

The ‘but for’ test is satisfied as there was a 45 per cent chance the patient would have lived were it not for the breach.

A

The ‘but for’ test is not satisfied as there was a 55 per cent chance the patient would have died without the breach.

This is the correct answer. In order to establish factual causation, the claimant must prove on the balance of probabilities, that if it were not for the defendant’s breach, they would not have suffered their loss at that time and in that way. The patient in this scenario cannot do this. On the balance of probabilities, if it were not for the doctor’s breach, they would have died anyway.

How well did you know this?
1
Not at all
2
3
4
5
Perfectly
121
Q

A patient was in hospital being treated for heart disease. A nurse misread the consultant’s notes and negligently prescribed the patient incorrect medication (the breach). The patient suffered from a cardiac arrest and subsequent brain damage. Medical evidence suggests that the cardiac arrest was caused by either the heart disease or the breach. Which of the following is most accurate in relation to the ‘but for’ test?

The ‘but for’ test cannot be satisfied as the patient would have suffered the loss irrespective of the breach.

The ‘but for’ test is satisfied as the breach was one of the causes of the patient’s loss.

The ‘but for’ test is satisfied as there is a 50 per cent chance that the breach caused the patient’s loss.

The ‘but for’ test cannot be satisfied as there are two equally probable causes of the patient’s loss.

The ‘but for’ test cannot be satisfied as there is more than one potential cause of the patient’s loss.

A

The ‘but for’ test cannot be satisfied as there are two equally probable causes of the patient’s loss.

This is the correct answer. In order to satisfy the ‘but for’ test, the claimant must prove that there is a greater than 50 cent chance that the breach caused their loss. The patient cannot only prove that there was a 50 per cent chance. Factual causation fails. See Wilsher.

How well did you know this?
1
Not at all
2
3
4
5
Perfectly
122
Q

A patient undergoes an operation on their face that carries a 5% risk of causing permanent scarring. The risk materialises. The surgeon did not inform the patient of the risk prior to the operation.

Which of the following statements is most accurate in relation to factual causation?

Factual causation is not satisfied as there was only a 5% risk of the scarring.

Factual causation is satisfied if the patient can show that they would not have had the operation had they been told of the risk.

Factual causation is satisfied if the patient can show that they would have deferred the operation had they been told of the risk.

Factual causation is not satisfied as the patient consented to the operation.

Factual causation is satisfied if the patient can show that they would not have had the operation or would have deferred the operation to a later date had they been told of the risk.

A

Factual causation is satisfied if the patient can show that they would not have had the operation or would have deferred the operation to a later date had they been told of the risk.

This is the correct answer. In clinical negligence cases where the breach is a failure to advise on risk, the claimant must prove on the balance of probabilities that they would not have gone ahead with the treatment had they been informed of the risks. If they cannot do this, they can still satisfy factual causation if they can prove on the balance of probabilities that they would have deferred the treatment had they been informed of the risks. The rationale behind this being that ‘but for’ the doctor’s negligent failure to warn the claimant of the risk, the actual injury would not have occurred when it did and the chance of it occurring on a subsequent occasion (if the claimant had chosen to have the operation at a later date) was small.​

How well did you know this?
1
Not at all
2
3
4
5
Perfectly
123
Q

Which of the following most accurately explains when the courts might apply the material contribution test and what the claimant must prove in order to satisfy the test?

The court might apply the material contribution test where there is more than one potential cause of the claimant’s loss, and the causes have acted together to cause the loss. The claimant must prove that the breach made a significant contribution to their loss.

The court might apply the material contribution test where there is more than one potential cause of the claimant’s loss, and the causes have acted together to cause the loss. The claimant must prove that the breach made a greater than 50 per cent contribution to their loss.

The court might apply the material contribution test where there is more than one potential cause of the claimant’s loss, and the breach increased the risk of the loss. The claimant must prove that the breach made a more than negligible contribution to their loss.

The court might apply the material contribution test where there is more than one potential cause of the claimant’s loss, and the causes have acted together to cause the loss. The claimant must prove that the breach made a more than negligible contribution to their loss.

The court might apply the material contribution test where there is more than one potential cause of the claimant’s loss, and the causes have acted independently to cause the loss. The claimant must prove that the breach made a more than negligible contribution to their loss.

A

The court might apply the material contribution test where there is more than one potential cause of the claimant’s loss, and the causes have acted together to cause the loss. The claimant must prove that the breach made a more than negligible contribution to their loss.

This is the correct answer. See Bonnington for an illustration of the material contribution test being applied in practice.

How well did you know this?
1
Not at all
2
3
4
5
Perfectly
124
Q

A patient visits A&E with severe stomach pain. After a five hour delay, they are seen by a doctor and diagnosed with appendicitis. The patient requires immediate surgery, but just before the surgery begins, their appendix ruptures and causes an infection. Medical evidence suggests that the appendicitis and the delay contributed to their rupture and infection.

Which of the following statements is most accurate in relation to factual causation?

Factual causation fails as it could have been the appendicitis alone, not the delay, that caused the rupture and infection.

Factual causation cannot be established as it cannot be shown that ‘but for’ the breach (the delay) the patient would not have suffered a rupture and infection.

The delay materially increased the risk of rupture and infection meaning factual causation is satisfied.

Given the appendicitis and delay contributed to the rupture and infection, the material contribution test is satisfied.

There is a greater chance that the delay caused the rupture and infection than the appendicitis alone so factual causation is satisfied.

A

Given the appendicitis and delay contributed to the rupture and infection, the material contribution test is satisfied.

This is the correct answer. This is a case where medical science cannot establish the probability that ‘but for’ an act of negligence (the delay) the rupture and infection would not have happened. However, it can establish that the contribution of the delay was more than negligible (therefore the material contribution test is satisfied) (Bailey v Ministry of Defence [2008] EWCA Civ 883). Medical evidence established that the delay and appendicitis together caused the rupture and infection.

How well did you know this?
1
Not at all
2
3
4
5
Perfectly
125
Q

Which of the following most accurately explains when the courts might apply the material increase in risk test and what the claimant must prove in order to satisfy the test?

The court might apply the material increase in risk test to industrial disease cases where there is more than one potential cause of the claimant’s loss. The claimant must prove that the breach made a significant contribution to the risk.

The court might apply the material increase in risk test to single agent industrial disease cases where there is more than one potential cause of the claimant’s loss. The claimant must prove that the breach made a greater than de minimus contribution to the risk.

The court might apply the material increase in risk test to single agent industrial disease cases where there is more than one potential cause of the claimant’s loss. The claimant must prove that the breach made a more than negligible contribution to their loss.

The court might apply the material increase in risk test to single agent industrial disease cases where there is more than one potential cause of the claimant’s loss. The claimant must prove that the breach made a greater than 50% contribution to the risk.

The court might apply the material increase in risk test to industrial disease cases where there is more than one potential cause of the claimant’s loss. The claimant must prove that the breach made a greater than de minimus contribution to the risk.

A

The court might apply the material increase in risk test to single agent industrial disease cases where there is more than one potential cause of the claimant’s loss. The claimant must prove that the breach made a greater than de minimus contribution to the risk.

This is the correct answer. See McGhee v National Coal Board [1973] 1 WLR 1 (HL) which established and applied this test.

How well did you know this?
1
Not at all
2
3
4
5
Perfectly
126
Q

Which of the following best describes apportionment?

Where there are multiple tortious factors which have caused the claimant’s loss, the courts will divide liability between the various defendants reflecting the respective fault of each defendant.

Where there are multiple tortious factors at play, the courts will rely on apportionment to establish factual causation and divide liability between the various defendants and claimant.

Where there are multiple tortious factors which have caused the claimant’s loss, the courts will divide liability between the various defendants and claimant reflecting the respective fault of each claimant.

Where there are multiple tortious factors which have caused the claimant’s loss, the courts will divide liability equally amongst the various defendants.

Where there are multiple tortious factors at play, the courts will rely on apportionment to establish factual causation and divide liability between the various defendants.

A

Where there are multiple tortious factors which have caused the claimant’s loss, the courts will divide liability between the various defendants reflecting the respective fault of each defendant.

This is the correct answer. Apportionment is a calculation applied once factual causation has been established.

How well did you know this?
1
Not at all
2
3
4
5
Perfectly
127
Q

What was a key difference between Baker and Jobling that might help to explain the contrasting decisions?

In Baker there were two tortious events and in Jobling there was a tort followed by a natural event.

In Baker the second defendant could not be found whereas in Jobling the second defendant could be found.

In Jobling there were two tortious events and in Baker there was a tort followed by a natural event.

In Baker the second defendant could be found whereas in Jobling the second defendant could not be found.

A

In Baker there were two tortious events and in Jobling there was a tort followed by a natural event.

This is the correct answer. In Jobling, the further back injury would have happened anyway, whether the claimant was injured or not. This was not the case in Baker.

How well did you know this?
1
Not at all
2
3
4
5
Perfectly
128
Q

A claimant suffered a serious arm injury at work due to their employer’s negligence and was signed off work for four months. Two months after the accident at work, the claimant was diagnosed with arthritis in the same arm and had to stop working completely. The arthritis was entirely unconnected with the accident at work and/or the injury that resulted.

Which of the following best describes the employer’s liability for the claimant’s losses?

The employer will be liable for all the claimant’s losses because a defendant is liable for any losses that occur after their breach.

The employer will be liable for all of the claimant’s losses even past the point of the arthritis.

The employer will be liable for all the claimant’s losses if the second defendant cannot be found.

The employer will not be liable for any of the claimant’s losses as the claimant would have suffered arthritis in any event.

The employer will be liable for the claimant’s losses up until the point that the arthritis developed.

A

The employer will be liable for the claimant’s losses up until the point that the arthritis developed.

How well did you know this?
1
Not at all
2
3
4
5
Perfectly
129
Q

Which of the following lists most accurately the correct types of novus actus interveniens?

Acts of claimant; acts of God; and acts of third parties.

Acts of claimant; acts of God; acts of third parties; and acts of defendant.

Acts of claimant; acts of God; and acts of defendant.

Acts of claimant; acts of God; and acts of medical negligence.

Acts of claimant; acts of third parties; acts of medical negligence; and acts of defendant.

A

Acts of claimant; acts of God; and acts of third parties.

This is the correct answer. These are the three types of novus actus interveniens.

130
Q

Which of the following statements is most accurate in relation to the legal test for an act of claimant?

The act of claimant must be unreasonable and unforeseeable in order to break the chain of causation.

The act of claimant must be highly unreasonable in order to break the chain of causation.

The act of claimant must be unforeseeable in order to break the chain of causation.

The act of claimant must be grossly negligent in order to break the chain of causation.

The act of claimant must be unreasonable in order to break the chain of causation.

A

The act of claimant must be highly unreasonable in order to break the chain of causation.

This is the correct answer. See case law eg McKew v Holland.

131
Q

A cyclist negligently collides with a pedestrian who was crossing the road. The pedestrian falls and injures their leg. A passer-by attempts to help the pedestrian and when moving the pedestrian from the road to the pavement, drops the pedestrian. The pedestrian hits their head and suffers a head injury. Which of the following is most accurate in relation to the passer-by’s actions?

The act of the passer-by was foreseeable and does not therefore break the chain of causation between the cyclist’s breach and the pedestrian’s leg injury. The cyclist will only be liable for the pedestrian’s leg injury.

The act of the passer-by was unreasonable and therefore breaks the chain of causation between the cyclist’s breach and pedestrian’s head injury. The cyclist will only be liable for the pedestrian’s head injury.

The act of the passer-by was foreseeable and does not therefore break the chain of causation between the cyclist’s breach and the pedestrian’s head injury. The cyclist will be liable for the pedestrian’s leg and head injury.

The act of the passer-by was unforeseeable and therefore breaks the chain of causation between the cyclist’s breach and pedestrian’s head injury. The cyclist will only be liable for the pedestrian’s leg injury.

The act of the passer-by was unforeseeable and therefore breaks the chain of causation between the cyclists’ s breach and pedestrian’s leg injury. The cyclist will not be liable for any of the pedestrian’s injuries.

A

The act of the passer-by was foreseeable and does not therefore break the chain of causation between the cyclist’s breach and the pedestrian’s head injury. The cyclist will be liable for the pedestrian’s leg and head injury.

This is the correct answer. An act of a third party (here, the passer-by) will only break the chain of causation between the first defendant’s breach and the claimant’s loss if its is highly unforeseeable (something very unlikely to happen as a result of the defendant’s breach). Here, it is foreseeable that a third party would try and help the injured pedestrian and as a result could make the pedestrian’s injuries worse.

132
Q

The current test for remoteness in general negligence claims comes from which one of the following cases?

Caparo v Dickman

The Wagon Mound (No1)

Tremain v Pike

A

The Wagon Mound (No1)

Well done. The Wagon Mound (No1) is the authority for the current test.

133
Q

Which one of the following is a correct summary of the thin skull rule from Smith v Leech Brain?

The defendant must take their victim as they find them, regardless of whether the injury suffered during the act is more than expected.

The defendant will pay less damages if the victim has a thin skull.

The defendant must take their victim as they find them, regardless of any pre-existing illness.

The defendant must pay additional damages where the victim has a skull injury.

The defendant need only pay damages up to the amount a reasonable person would have suffered but not any additional damage which arises from the victim’s thin skull.

A

The defendant must take their victim as they find them, regardless of any pre-existing illness.

134
Q

Larry, a model, suffers facial injuries as a result of falling from a theme park ride. The theme park had negligently failed to install appropriate safety measures. Larry is taken to hospital where a surgeon operates on his face. Larry has an allergic reaction to the anaesthetic and suffers additional nerve damage. Larry is unable to accept any modelling work for six months. Which of the following is most accurate in relation to remoteness?

Neither Larry’s personal injury nor the consequential economic loss were reasonably foreseeable consequences of the breach.

Larry’s personal injury was a reasonably foreseeable consequence of the breach but the consequential economic loss was not.

Larry’s consequential economic loss was a reasonably foreseeable consequence of the breach but the personal injury was not.

Both Larry’s personal injury and the consequential economic loss were reasonably foreseeable consequences of the breach.

Larry’s personal injury and consequential economic loss were reasonably foreseeable consequences of the breach but he has acted unreasonably so the losses will be too remote.

A

Both Larry’s personal injury and the consequential economic loss were reasonably foreseeable consequences of the breach.

Well done. The facial injuries and nerve damage are a reasonably foreseeable consequence of the breach (personal injury is a single indivisible type of harm – Page v Smith) and it is reasonably foreseeable that, as a result of personal injury, a victim may not be able to work for a period of time.

135
Q

Which of the following is not one of the four elements which the defendant must show to establish the defence of consent?

The claimant had full knowledge of the nature and extent of the risks.

The claimant agreed to the risk of injury.

The claimant’s agreement to the risk was voluntary.

The claimant had capacity to give valid consent to the risks.

The claimant could have benefitted from the risks, or avoided a loss.

A

The claimant could have benefitted from the risks, or avoided a loss.

Well done. This is not part of the defence. The other things listed are part of the defence.

136
Q

Which of the following is authority for the proposition that whether the claimant had full knowledge of the nature and extent of the risks is a subjective test, not a question of what the reasonable claimant would have known?

Nettleship v Weston

Smith v Charles Baker & Sons

Morris v Murray

Reeves v Commissioner of Police for the Metropolis

A

Morris v Murray

137
Q

To establish the defence of consent, a defendant must show that the claimant consented to the risk concerned ‘voluntarily’. When considering whether an employee’s consent is voluntary…

…an employee will have consented to risks that it was fully informed of when choosing to start / continue working.

…it is not possible to establish that an employee consented to risks when it comes to a claim against the employer.

…it will be hard but not impossible for an employer to rely on the defence of consent when faced with an employee’s claim.

A

…it will be hard but not impossible for an employer to rely on the defence of consent when faced with an employee’s claim.

It would be wrong to say that ‘…an employee will have consented to risks that it was fully informed of when choosing to start / continue working’ because Smith v Charles Baker & Sons makes clear that the requirement of voluntary consent is in addition to knowledge of the risk. Employees who know of the risks of their jobs are not necessarily voluntarily running those risks, since they may have little real option if they wish to keep their job.

138
Q

To establish the defence of contributory negligence, the defendant must show:

(a) The claimant failed to take reasonable steps for their own safety; and (b) this failure contributed to the accident happening.
(a) The claimant breached a duty owed to the defendant; and (b) this failure contributed to the claimant’s damage.
(a) The claimant breached a duty owed to the defendant; and (b) this failure contributed to the accident happening.
(a) The claimant failed to take reasonable steps for their own safety; and (b) this failure contributed to the claimant’s damage.

A

(a) The claimant failed to take reasonable steps for their own safety; and (b) this failure contributed to the claimant’s damage.

Correct. See Jones v Livox.

139
Q

Pete, a passenger on Saraf’s motorbike, refuses to wear a helmet and is seriously injured when Saraf’s negligent driving leads to a crash. Which one of the following is correct?

Pete has been contributorily negligent and s1(1) Law Reform (Contributory Negligence) Act 1946 will apply

Pete has been contributorily negligent and s1(1) Law Repeal (Contributory Negligence) Act 1946 will apply

Pete has been contributorily negligent and s1(1) Law Reform (Contributory Negligence) Act 1945 will apply

Pete has been contributorily negligent and s1(1) Law Repeal (Contributory Negligence) Act 1945 will apply

A

Pete has been contributorily negligent and s1(1) Law Reform (Contributory Negligence) Act 1945 will apply

Section 1(1) Law Reform (Contributory Negligence) Act 1945 will apply.

140
Q

Where the defence of contributory negligence has been established…

…damages are reduced by a percentage which is just and equitable considering the number of parties who contributed to the injury.

…damages are reduced by a percentage which is just and equitable considering both the claimant’s and defendant’s culpability.

…the claim fails.

…damages are reduced by 25%.

…damages are reduced by 50%.

A

…damages are reduced by a percentage which is just and equitable considering both the claimant’s and defendant’s culpability.

Well done. Whilst percentages of 50% / 25% will be appropriate in certain cases, there is no set rule. The number of parties who contributed to the injury is not the best way to approach the relevant question.

141
Q

The defence of illegality is also known as:

Ex turpi causa non oritur actio

Volenti non fit injuria

A

Ex turpi causa non oritur actio

142
Q

Which of the following best expresses the first part of the two-part approach that we recommend in considering the defence of illegality?

Has the defendant committed a tort?

Has the claimant committed an illegal act?

Has the defendant committed an illegal (or grossly immoral) act?

Has the claimant committed an illegal (or grossly immoral) act?

Has the claimant committed a tort?

A

Has the claimant committed an illegal (or grossly immoral) act?

Correct
Well done. Whilst in almost all cases the defence involves illegality, it appears that gross immorality may also allow the defence to be raised.

It is neither necessary, nor sufficient, that the claimant has committed a tort.

It is true that unless the defendant had committed a tort, you would not be looking at defences at all: but the defendant committing a tort is not part of the defence itself.

The illegality must be on the part of the claimant.

143
Q

If the claimant has committed an illegal (or grossly immoral) act, the next part of the test is to consider the three considerations identified in Patel v Mirza in order to determine whether allowing the claim would harm the integrity of the legal system. Which of the following is not one of the three considerations as set out in that case?

Whether denial of the claim would be a proportionate response to the illegality.

Any other relevant public policy on which the denial of the claim may have an impact.

The underlying purpose of the prohibition which has been transgressed and whether that purpose will be enhanced by denial of the claim.

Whether the claimant’s claim is founded upon their own criminal or immoral act.

A

Whether the claimant’s claim is founded upon their own criminal or immoral act.

Correct
Well done. Prior to Patel v Mirza it was generally thought that the defence only arises if the facts which give rise to the claim are inextricably linked with the criminal activity. The court made clear that that was no longer the test, but this is still likely to be a relevant consideration when considering proportionality. The other three answers are the three considerations identified in Patel v Mirza.

144
Q

Which one of the below correctly describes the position in relation to damages following death?

Only spouses of the deceased may bring a claim (under the Fatal Accidents Act 1976).

The estate of the deceased may bring a claim for losses suffered by the deceased under the Law Reform (Miscellaneous Provisions) Act 1934.

The estate of the deceased may bring a claim for losses suffered by the deceased under the Fatal Accidents Act 1976

It is not possible to claim for damages following death because the deceased cannot bring the claim, and no-one can bring it on their behalf.

Only parents of the deceased may bring a claim (under the Fatal Accidents Act 1976).

A

The estate of the deceased may bring a claim for losses suffered by the deceased under the Law Reform (Miscellaneous Provisions) Act 1934.

Well done. The damages in relation to the losses suffered by the deceased will be calculated on the same basis as a normal personal injury award. Remember that certain dependants of the deceased may also bring a claim under the Fatal Accidents Act 1976 – this includes both parents and spouses (and potentially some other categories of dependants).

145
Q

Which of the following is not an example of special damages?

The cost of medical care already incurred before trial.

The cost of repairing a car before trial.

The value of a mobile phone damaged in an accident.

Loss of future earnings.

A

Loss of future earnings.

Correct. This would be general damages, because it cannot be specifically proven. The others are all good examples of special damages.

146
Q

The aim of damages in the law of tort is to…

… put the claimant in the position they would have been in but for the defendant’s tortious act.

…. restore to the claimant any benefit obtained from the defendant.

… award fair compensation for the harm caused to the claimant.

… put the claimant in the position they were in prior to their engagement / interaction with the defendant.

A

… put the claimant in the position they would have been in but for the defendant’s tortious act.

Well done. Whilst it is fair to say that damages aim to award fair compensation, this is too vague to be a good summary of the aim of damages in the law of tort. The aim of damages may often mean that the damages put the claimant in the position they were in prior to their engagement / interaction with the defendant, but this is not always the case, and this is not the best way of explaining the aim of the damages in the law of tort. It would be wrong to say that the aim is to restore to the claimant any benefit obtained from the defendant. In many cases that would lead to no damages at all, as often the defendant does not benefit at all from (for example) its negligence.

147
Q

This element (Employers’ primary liability) was about…

…the duties which employees owe to others.

…how the law of negligence applies to the relationship between employer and employee.

…the duties which an employer owes to an employee.

…how the law of negligence applies to determine when an employer is responsible for the negligence of an employee.

A

…how the law of negligence applies to the relationship between employer and employee.

Well done. This element was not about when an employer is responsible for the negligence of an employee – that is the topic of vicarious liability, which is something different. It would be too broad to say this element was about the duties which an employer owes to an employee – this element focused only on the law of negligence, not (for example) duties imposed by statute. This element was not concerned with duties which employees owe to others.

148
Q

The duty imposed on employers is ‘non-delegable’, meaning…

The employer can delegate performance of the duty but will still be liable if it is breached.

The employer cannot argue that an employee has consented to the risk of negligence.

The employer cannot delegate performance of the duty.

The employer cannot delegate liability for breach nor performance of the duty.

A

The employer can delegate performance of the duty but will still be liable if it is breached.

149
Q

Which of the following is not part of the employer’s duty owed to its employees according to the traditional formulation in Wilsons and Clyde Coal Ltd v English?

Safe systems of work, with adequate supervision and instruction.

Safe and proper plant and equipment.

Safe and competent employees.

Safe place of work/premises, including safe access and way out.

Safe transport to and from a place of work.

A

Safe transport to and from a place of work.

Well done. An employer does not generally have a duty to provide safe transport to and from work (certainly in relation to travel to the usual place of work). An employer does have a duty to provide the other things listed.

150
Q

Which of the following is the best authority for the proposition that an employer owes a duty to provide a safe place of work even in relation to premises which are not the employer’s own premises?

Latimer v AEC

Bux v Slough Metals

Wilson v Tyneside Cleaning Co

Wilsons and Clyde Coal Ltd v English

A

Wilson v Tyneside Cleaning Co

151
Q

The defendant who is vicariously liable incurs what is called ‘strict liability’, meaning…

…they are liable even though they are not at fault.

…they have fallen below the required standard but not caused any damage.

…they have been held to (and fallen below) a higher than usual standard of care.

…they are presumed liable unless they can show they have met the required standard.

A

…they are liable even though they are not at fault.

Well done. This is the right answer. Vicarious liability has nothing to do with standard of care, presumptions in relation to liability nor whether any damage has been caused.

152
Q

Which one of the following is not one of the three elements that must be shown to establish that Party B is vicariously liable for a tort committed by Party A?

Party A’s actions were authorised by Party B, or were an unauthorised way of doing something authorised by Party B.

A tort has been committed by Party A.

Party A is an employee of Party B, or failing that, Party A is in a relationship akin to employment with Party B.

The tort was committed in the course of Party A’s employment / quasi-employment.

A

Party A’s actions were authorised by Party B, or were an unauthorised way of doing something authorised by Party B.

Well done. This IS NOT one of the three elements that must be shown to establish that Party B is vicariously liable for a tort committed by Party A. This idea used to be an important part of showing that a tort was committed in the course of Party A’s employment, but a different approach is now taken, following Lister v Hesley Hall.

153
Q

Lister v Hesley Hall established that a tort will be ‘in the course of employment’…

…if the tort was committed during the employee’s working hours and/or on the employer’s premises or other agreed workplace.

…if the employee’s employment gave the employee the opportunity to commit the tort.

…if the tort was an ‘accidental’ tort (such as negligence) rather than an ‘intentional’ tort (such as assault or battery).

…if there is a sufficiently close connection between the employee’s tort and the role he / she is employed to do.

A

…if there is a sufficiently close connection between the employee’s tort and the role he / she is employed to do.

Well done. This is the ratio of Lister v Hesley Hall. Whilst the tort taking place in working hours or on the employer’s premises is highly likely to be relevant in showing a close connection between tort and employment, neither is necessary. Similarly, the employee’s employment giving the opportunity to commit the tort might suggest the tort was committed in the course of employment, but it is neither conclusive, nor necessary.

154
Q

Tom works as a gym instructor at a gym called ‘Fighting Fit’. His contract states that he is self-employed. He wears a Fighting Fit uniform and is paid a monthly salary for working set hours each week. If Tom is unable to work due to illness, he arranges a substitute gym instructor, selected from a bank of agency workers who are pre-approved by Fighting Fit. Which one of the following correctly describes whether Tom is an employee of Fighting Fit or not?

Tom is likely to be an employee of Fighting Fit because the label given to the working arrangement in the contract is irrelevant and would be ignored by a court.

Tom will not be an employee of Fighting Fit because he is able to arrange for a substitute to work in his place.

Tom is likely to be an employee because he is paid a monthly salary and pay is the most important indicative factor to determining employment status.

Tom is likely to be an employee because the arrangement satisfies the Ready Mixed Concrete v Minister of Pensions test.

Tom will not be an employee of Fighting Fit because his contract clearly states that he is self-employed.

A

Tom is likely to be an employee because the arrangement satisfies the Ready Mixed Concrete v Minister of Pensions test.

Well done. Tom is paid a salary and there is mutuality of obligations as he is required to work set hours. Fighting Fit have a significant degree of control over Tom and provide him with a uniform. The fact that the contract states he is self-employed (the ‘label’ given to the relationship) does not necessarily mean that he is, but it is a relevant consideration. Being able to send a substitute does suggest Tom is self-employed, but much less so where Tom can only select a substitute from a pre-approved bank of workers. Pay alone is relevant but not decisive.

155
Q

Barclays Bank Plc v Various Claimants considered which type of relationships would or would not be ‘akin to employment’ in the context of vicarious liability. How did that case describe the type of party that would definitely not be in a relationship akin to employment?

Someone who is paying their own taxes and National Insurance.

Someone who has some control over their own work independent of the alleged employer.

Someone who is clearly carrying on their own independent business.

A

Someone who is clearly carrying on their own independent business.

Well done. The other two answers are wrong. A party might have some control over their own work and/or pay their own income tax / National Insurance, but still be in a relationship akin to employment.

156
Q

The general rule when one employer ‘lends’ an employee (X) to another employer is that…

…both employers are jointly and severally liable for any torts committed by X in the course of employment.

…the second employer is vicariously liable for any torts committed by X in the course of employment.

…the first employer remains vicariously liable for any torts committed by X in the course of employment.

A

…the first employer remains vicariously liable for any torts committed by X in the course of employment.

Well done - see Mersey Docks and Harbour Board v Coggins and Griffiths. Dual liability can arise, but it is the exception rather than the rule – see Viasystems Ltd v Thermal Transfer Ltd.

157
Q

A witness sees the immediate aftermath of a car accident and sees someone screaming in pain and covered in blood. As a result, the witness develops post-traumatic stress disorder. Which of the following is correct?

The witness is a secondary victim.

The witness is a bystander and is therefore a primary victim.

The witness is not a secondary victim as they did not witness the accident.

The witness is an actual victim.

The witness is a primary victim.

A

The witness is a secondary victim.

This is the correct answer. The witness has suffered psychiatric harm due to fear for someone else’s safety. They witnessed the immediate aftermath of the traumatic event but were not in the foreseeable range of physical injury.

158
Q

A neighbour notices their elderly neighbour’s house is on fire. Despite all the smoke, the neighbour can just about see that their elderly neighbour is fast asleep downstairs. The neighbour breaks into the house to rescue the elderly neighbour. Following the rescue, the neighbour suffers respiratory failure due to inhalation of smoke and also suffers clinical depression. Which of the following is correct?

The neighbour is a rescuer and is therefore a primary victim.

The neighbour is an actual victim.

The neighbour is a rescuer and is therefore a secondary victim.

The neighbour is a secondary victim.

The neighbour is a primary victim.

A

The neighbour is an actual victim.

This is the correct answer. The neighbour is an actual victim as they have suffered physical injury (respiratory failure) as well as psychiatric harm.

159
Q

The organisers of a rugby match negligently let too many people into the stadium and people start getting crushed against the barriers. The rugby match is being broadcast live on television and although individuals cannot be identified, those watching on television can see that some people are being crushed against the barrier. A mother is watching the game on television and knows her son is at the game. Following the event, the mother suffers from post-traumatic stress disorder.

Which of the following is correct?

The mother is a primary victim because her son was at the event. It will be hard for her claim to succeed as the courts will be concerned about opening the floodgates and crushing liability.

The mother is a secondary victim because she saw the event on television. It will be hard for her claim to succeed as the courts will be concerned about opening the floodgates and crushing liability.

The mother is an actual victim. It will be hard for the mother’s claim to succeed as the courts will be concerned about opening the floodgates and crushing liability.

The mother is a primary victim. It will be hard for her claim to succeed as the courts will be concerned about opening the floodgates and crushing liability.

The mother is a secondary victim. It will be hard for the mother’s claim to succeed as the courts will be concerned about opening the floodgates and crushing liability.

A

The mother is a secondary victim. It will be hard for the mother’s claim to succeed as the courts will be concerned about opening the floodgates and crushing liability.

Correct
This is the correct answer. The mother is a secondary victim as she has suffered psychiatric harm as a result of being in fear for someone else’s safety.

It is hard for secondary victim claims to succeed. Some of the policy reasons behind this include fear of the floodgates opening and crushing liability. For example, if the mother were able to claim against the organisers, this would lead to a significant increase in the class of claimants who could recover for psychiatric harm (i.e. anyone who witnesses a traumatic event on television and suffers psychiatric harm). This in turn would lead to crushing liability on behalf of the defendant who could be faced with thousands of claims.

160
Q

An elderly woman got trapped in an overcrowded lift for three hours. In the lift she became upset and tearful as a result of being trapped. Following the incident, the elderly woman had difficulty sleeping and suffered nightmares for several days. Which of the following is correct?

As a primary victim, the woman is likely to succeed in claiming damages for upset and nightmares.

The woman can claim for her tearfulness and upset but not the nightmares.

As an actual victim, the woman is likely to succeed in claiming for upset and nightmares.

It is very unlikely the woman can claim damages for upset and nightmares.

The woman can claim for her nightmares but not her tearfulness and upset.

A

It is very unlikely the woman can claim damages for upset and nightmares.

This is the correct answer. Claimants can only recover damages for medically recognised psychiatric harm. It is highly unlikely that upset and nightmares will be recognised as psychiatric harm. This reaction would be a normal emotion in the face of an unpleasant experience.

161
Q

A man notices his elderly neighbour’s house is on fire. Despite all the smoke, he can just about see that his neighbour is fast asleep downstairs. The man breaks into the house to rescue his neighbour. Following the rescue, he suffers clinical depression. It transpires that the neighbour’s electrician negligently started the fire. Which of the following is correct?

A duty of care would not be owed as psychiatric injury was not reasonably foreseeable as a result of the defendant’s negligence.

The man is a rescuer and could therefore be owed a duty of care.

A duty of care could be owed as physical injury was reasonably foreseeable as a result of the defendant’s negligence.

A duty of care could be owed as psychiatric injury was reasonably foreseeable as a result of the defendant’s negligence.

A duty of care would not be owed as physical injury was not reasonably foreseeable as a result of the defendant’s negligence.

A

A duty of care could be owed as physical injury was reasonably foreseeable as a result of the defendant’s negligence.

correct
This is the correct answer. It is reasonably foreseeable to an electrician that if they negligently start a fire in someone’s house, somebody could suffer physical injury. As the ‘rescuer’ is a primary victim, only physical injury needs to be foreseeable not psychiatric injury in order to establish a duty of care.

incorrect
A duty of care could be owed as psychiatric injury was reasonably foreseeable as a result of the defendant’s negligence.

This is incorrect. The ‘rescuer’ is a primary victim, so it is irrelevant whether psychiatric harm was reasonably foreseeable when determining duty of care. Please review this element to ensure you understand the duty of care test for a primary victim.

162
Q

A man was working at an oil storage facility when there was a huge explosion resulting in a fire. The fire was confined to a building one kilometre from where the man was working, but he still feared for his safety. Following the event, he suffered from post-traumatic stress disorder.

Which of the following is correct?

The man will not be classified as a primary victim because he did not fear for anyone else’s safety.

The man will be classified as a primary victim as he was in the danger zone.

The man will not be classified as a primary victim because he did not reasonably fear for his own safety.

The man will not be classified as a primary victim because he was not physically injured.

The man will be classified as a primary victim as he feared for his own safety.

A

The man will not be classified as a primary victim because he did not reasonably fear for his own safety.

This is the correct answer. A primary victim is a person who suffers psychiatric harm as a result of reasonably fearing for their own safety. The test is objective not subjective. A reasonable person would not fear for their own safety if the fire was one kilometre away.

163
Q

Bart goes to the local go-kart track to take part in a race. One of the other competitors loses concentration and swerves into Bart’s go-kart at high speed. This causes Bart’s go-kart to flip upside down. Bart’s stepmother, Fatima, who has lived with and cared for Bart since he was five, is also at the track. At the time of the accident, she was in the nearby car park. On her way back to the track, Fatima hears a message over the loudspeakers indicating that there has been a serious accident. Worried, Fatima runs over to the track. By the time she arrives, Bart has just been taken to hospital. Fatima drives over to the hospital immediately and sees Bart screaming in pain, covered in dirt and blood. As a result of fearing for Bart, Fatima suffers from clinical depression.

Which one of the following statements best explainsthe legal position in the tort of negligence?

As a secondary victim, Fatima must have been within the foreseeable range of physical injury.

As a primary victim, Fatima must establish that personal injury to her was reasonably foreseeable.

As a secondary victim, Fatima should be able to prove that she has close ties of love and affection with Bart, but it will be harder for her to prove that she satisfies the requirement of proximity in terms of time and space.

As a primary victim, Fatima must establish that psychiatric harm was reasonably foreseeable.

Fatima is a secondary victim and her claim is unlikely to succeed because she will be unable to prove close ties of love and affection with Bart, and unable to prove that she had proximity in terms of time and space.

A

As a secondary victim, Fatima should be able to prove that she has close ties of love and affection with Bart, but it will be harder for her to prove that she satisfies the requirement of proximity in terms of time and space.

This is the correct answer.

Fatima is a secondary victim as she has suffered psychiatric harm as a result of fearing for someone else’s safety. She will need to satisfy the Alcock criteria in order to establish a duty of care. Two out of the four criteria are mentioned in this answer. Fatima would have to prove that she has close ties of love and affection with Bart. Although Fatima does not fall within one of the relationships where there is a rebuttable presumption of close ties of love and affection, she is Bart’s stepmother. The courts are likely to extend the relationship of parent to include stepmothers if she has a close relationship with Bart.

It will be harder for Fatima to satisfy the requirement of proximity in terms of time and space, but she might be able to. She would have to prove that she came across the immediate aftermath and the courts will therefore be interested in how long after the event she arrived at the hospital and exactly what she saw when she did arrive.

164
Q

A company is testing a racing car. The driver’s wife watches the test drive from the stands. Due to the company’s negligence, the car crashes and the driver suffers extensive burns. His wife watches the whole incident. She subsequently suffers from Post-Traumatic Stress Disorder.

Which of the following best describes whether the company owes the wife a duty of care?

The wife is a secondary victim. The company owes her a duty of care because the Alcock criteria are satisfied, i.e. psychiatric harm in a person of ordinary fortitude was reasonably foreseeable, there was proximity of relationship between the woman and the victim, there was proximity in time and space, the woman’s injury is the result of a sudden shock and it is fair, just and reasonable to impose a duty.

The wife is a secondary victim. The company owes her a duty of care because the Caparo criteria are satisfied, i.e. physical injury was reasonably foreseeable, there was proximity between the parties and it is fair, just and reasonable to impose a duty.

The wife is a primary victim. The company owes her a duty of care because the Caparo criteria are satisfied, i.e. physical injury was reasonably foreseeable, there was proximity between the parties and it is fair, just and reasonable to impose a duty.

The wife is a primary victim. The company owes her a duty of care because the Alcock criteria are satisfied, i.e. psychiatric harm in a person of ordinary fortitude was reasonably foreseeable, there was proximity of relationship between the woman and the victim, there was proximity in time and space, the woman’s injury is the result of a sudden shock and it is fair, just and reasonable to impose a duty.

The wife is an actual victim. The company owes her a duty of care because they employed her husband to drive the racing car.

A

The wife is a secondary victim. The company owes her a duty of care because the Alcock criteria are satisfied, i.e. psychiatric harm in a person of ordinary fortitude was reasonably foreseeable, there was proximity of relationship between the woman and the victim, there was proximity in time and space, the woman’s injury is the result of a sudden shock and it is fair, just and reasonable to impose a duty.

Correct
This is the correct answer.

This answer correctly identifies the wife as a secondary victim and sets out the Alcock criteria for establishing if a duty of care is owed. The other answers might seem plausible but they are each incorrect. The wife is not an actual victim as she has not suffered any physical injury nor would she be owed a duty just because her husband was employed by the company. The wife is not a primary victim as she was never in reasonable fear of her own safety. Finally, where the claimant is a secondary victim, it is the Alcock criteria (not Caparo) that is used to establish duty of care.

165
Q

Which of the following is NOT part of the Alcock criteria?

The psychiatric harm must be shock induced.

Physical injury must be reasonably foreseeable in a person of ordinary fortitude.

There must be proximity in time and space to the accident or its immediate aftermath.

There must be proximity of relationship between the claimant and the victim.

Psychiatric harm must be reasonably foreseeable in a person of ordinary fortitude.

A

Physical injury must be reasonably foreseeable in a person of ordinary fortitude.

Correct
This is the correct answer.

A secondary victim must prove that psychiatric harm was reasonably foreseeable, not just physical injury. This acts as a control device to limit the number of potential claimants who can claim for psychiatric harm.

166
Q

Sandra and her aunt Patricia go for a haircut and colour. The hairdresser uses the wrong hair dye on Sandra which causes a severe burning rash and Sandra’s skin starts to peel away. Patricia is so upset when she sees Sandra’s skin peeling away that she cannot stop thinking about it several weeks later. Patricia is later diagnosed with post-traumatic stress disorder.

Which one of the following is correct?

Patricia is a secondary victim. Her claim will likely succeed as she witnessed the traumatic event with her own unaided senses.

Patricia is a primary victim. Her claim will fail due to insufficient ties of love and affection with Sandra.

Patricia is a secondary victim. Her claim will likely succeed as psychiatric harm was reasonably foreseeable as a result of the defendant’s negligence.

Patricia is an actual victim. Her claim will fail due to insufficient ties of love and affection with Sandra.

Patricia is a secondary victim. Her claim will likely fail due to insufficient ties of love and affection with Sandra.

A

Patricia is a secondary victim. Her claim will likely fail due to insufficient ties of love and affection with Sandra.

correct
Correct
This is the correct answer.

Patricia is a secondary victim as she has suffered psychiatric harm as a result of fearing for someone else’s safety. It is likely that her claim would fail as she may not have close ties of love and affection with Sandra (second stage of the Alcock criteria when establishing duty of care). However, of course, we would need to know more about their relationship to be sure, but the aunt/niece relationship does not fall within one of the relationships where there is a rebuttable presumption of close ties of love and affection.

incorrect

Patricia is a secondary victim. Her claim will likely succeed as psychiatric harm was reasonably foreseeable as a result of the defendant’s negligence.

Incorrect
This is incorrect.

Patricia is a secondary victim as she has suffered psychiatric harm as a result of fearing for someone else’s safety. However, there would be additional criteria that Patricia would have to satisfy in order to prove she was owed a duty of care (not just that psychiatric harm was reasonably foreseeable). Please review this element to ensure you understand the duty of care test for secondary victims.

167
Q

Noah worked as a teacher at a school. He worked around 70 hours per week. Six months ago, he spoke to the headteacher about his workload and explained that he was not coping well. The following day he was signed off work by his GP for three weeks due to stress and depression. On return to work, Noah explained to the headteacher that he was not coping with his workload and it was becoming detrimental to his health. Two months later he lost control in the classroom and shouted at the students. He left the school and has not returned to work. He has been signed off work with depression by his GP.

Which of the following statements is most accurate?

The school owed Noah a duty of care in respect of psychiatric harm caused by stress at work. It is unlikely that the school is in breach of this duty as psychiatric harm was not reasonably foreseeable; Noah never said he was going to suffer a nervous breakdown/depression.

Noah is not an actual, primary or secondary victim and therefore has no claim.

The school owed Noah a duty of care in respect of psychiatric harm caused by stress at work. It is likely that the school is in breach of this duty as psychiatric harm was reasonably foreseeable; he had three weeks off work for depression and two meetings with the headteacher.

Noah will be unable to establish a duty of care as psychiatric harm was not reasonably foreseeable.

The school owed Noah a duty of care in respect of psychiatric harm caused by stress at work. It is unlikely that the school is in breach of this duty as although psychiatric harm was reasonably foreseeable, other teachers at the school probably work 70 hours per week and do not suffer from stress.

A

The school owed Noah a duty of care in respect of psychiatric harm caused by stress at work. It is likely that the school is in breach of this duty as psychiatric harm was reasonably foreseeable; he had three weeks off work for depression and two meetings with the headteacher.

This is the correct answer.

There is a precedent establishing that employers owe their employees a duty of care in respect of psychiatric harm caused by stress at work. For Noah to show breach of duty, he must prove the following:

Psychiatric harm to the claimant was (or ought to have been) reasonably foreseeable to the employer. Psychiatric harm was foreseeable given the reasons below;
Foreseeability depends upon the relationship between the characteristics of the claimant and the requirements made of them by the employer, including:
(a) The nature and extent of the work being undertaken. Seventy hours per week suggests Noah was working longer hours than the ‘normal’ teacher given the average working week is 40 hours;

(b) Signs of stress shown by the claimant. Noah had shown signs of impending harm to his health which would have been plain enough for any reasonable employer to realise they should do something: he had been signed off work for three weeks with stress and depression and had had two conversations with the headteacher about his workload; and
(c) The size and scope of the business and availability of resources. This includes the interests of other employees and the need to treat them fairly. Always consider the steps which the employer could and should have taken. The school most likely could have employed someone else to help reduce Noah’s workload.

168
Q

To establish breach in occupational stress claims, psychiatric harm to the claimant must be reasonably foreseeable to the employer. What factors do the court take into account to assess whether psychiatric harm was reasonably foreseeable?

The nature and extent of the work being undertaken; signs of stress from the claimant; and the size and scope of the business and availability of resources.

The nature and extent of the work being undertaken; whether the claimant informed the employer that they were suffering from stress; and the size and scope of the business and availability of resources.

Signs of stress from the claimant; and the size and scope of the business and availability of resources.

The nature and extent of the work being undertaken; and signs of stress from the claimant.

The nature and extent of the work being undertaken; whether the claimant had previously suffered a nervous breakdown; and the size and scope of the business and availability of resources.

A

The nature and extent of the work being undertaken; signs of stress from the claimant; and the size and scope of the business and availability of resources.

169
Q

Which case authority sets out the guidelines for breach in occupational stress claims?

Barber v Somerset County Council

Walker v Northumberland County Council

AB v Leeds Teaching Hospital NHS Trust

Waters v Commissioner of Police for the Metropolis

Swinney v Chief Constable of Northumbria Police

A

Barber v Somerset County Council

170
Q

Which of the following is correct in relation to losses recoverable in the tort of negligence?

Pure economic loss is generally recoverable, whilst physical damage and consequential economic loss are not.

Physical damage is generally recoverable, whilst pure and consequential economic loss are not.

Physical damage, pure and consequential economic loss are generally recoverable.

Physical damage and consequential economic loss are generally recoverable, whilst pure economic loss is not.

A

Physical damage and consequential economic loss are generally recoverable, whilst pure economic loss is not.

This is the correct answer.

171
Q

Lisa uses her Dad’s vacuum cleaner for her cleaning business. It explodes, injuring Lisa and breaking her client’s vase. Lisa sues the manufacturing company of the vacuum cleaner for her losses (i.e. for the cost of replacing the vacuum cleaner and vase, and for her injuries). Which of the following is correct?

Lisa may be able to recover damages for her personal injuries only.

Lisa may be able to recover damages for her personal injuries and for property damage to the vase.

Lisa cannot sue the manufacturing company for any losses as it was not her vacuum cleaner.

Lisa may be able to recover damages for her personal injuries and for the cost of replacing the vacuum cleaner.

Lisa may be able to recover damages for the cost of replacing the vacuum cleaner only.

A

Lisa may be able to recover damages for her personal injuries only.

Correct
This is the correct answer. Lisa would not be able to recover damages for the property damage to the vase as it was not her property. Her client would have to sue the manufacturing company instead. Lisa would not be able to recover damages for the cost of replacing the defective vacuum cleaner. The cost of repairing/replacing a defective item is classified as pure economic loss and is not recoverable. Lisa’s dad would have to sue the manufacturing company in the law of contract. Lisa can try and recover damages for her personal injuries. The manufacturing company could owe Lisa a duty of care not to cause her personal injury when using their product.

172
Q

Sebastian runs a restaurant. He engages Tank Specialities Ltd to supply and install a fish tank to store his lobsters in. The tank needs an electric pump to circulate the seawater for the purpose of oxygenation. Tank Specialities Ltd subcontracts the manufacture of the pumps to Motor Ltd. The pumps fail on a Monday morning when the restaurant is closed and 20 lobsters die from lack of oxygen. The lobsters cannot be used on Tuesday. Tank Specialities Ltd is insolvent. Sebastian sues Motor Ltd for the cost of replacing the pumps, for the value of the lost lobsters and for the loss of profit on the intended sale of the lost lobsters.

Which of the following is correct?

Sebastian cannot recover damages from Motor Ltd for any of the heads of loss claimed i.e. the value of the lobsters, the loss of profit on the lost lobsters and for the cost of replacing the pumps.

Sebastian can recover damages from Motor Ltd for all three heads of loss i.e. the cost of replacing the pumps, the value of the lost lobsters and the loss of profit on the lost lobsters.

Sebastian can recover damages from Motor Ltd for the value of the lost lobsters but cannot recover damages for the loss of profit on the lost lobsters or for the cost of replacing the pumps.

Sebastian can recover damages from Motor Ltd for the value of the lost lobsters and for the loss of profits on the intended sale of those lobsters. He cannot recover the cost of replacing the pumps.

Sebastian can only recover damages for the cost of replacing the pumps from Motor Ltd.

A

Sebastian can recover damages from Motor Ltd for the value of the lost lobsters and for the loss of profits on the intended sale of those lobsters. He cannot recover the cost of replacing the pumps.

Correct
This is the correct answer. The cost of replacing the pumps is pure economic loss (the cost of replacing a defective product). The general rule is that there is no duty of care owed for pure economic loss and none of the exceptions to this general rule apply. The damaged lobsters will be classified as property damage and a duty of care would therefore be owed in respect of this loss. The loss of profit on the intended sale of the damaged lobsters would be consequential economic loss and a duty of care would therefore be owed in respect of this loss. Sebastian can only recover for the physical damage to the lobsters and for financial loss suffered as a consequence of that physical damage.

incorrect
Sebastian can recover damages from Motor Ltd for the value of the lost lobsters but cannot recover damages for the loss of profit on the lost lobsters or for the cost of replacing the pumps.

Incorrect
This is incorrect. Sebastian can recover damages for the value of the lost lobsters. However, he can also recover damages for the loss of profit on the lost lobsters. Please review this element to ensure you understand the definitions of property damage, consequential economic loss and pure economic loss (and when a duty of care is owed in relation to each type of loss).

173
Q

Which of the following statements is correct?

A duty of care will be owed for pure economic loss caused by a negligent statement and by a negligent act.

A duty of care will not be owed for pure economic loss caused by a negligent statement or by a negligent act.

A duty of care will not be owed for pure economic loss caused by a negligent act but might be owed if caused by a negligent statement.

A duty of care will not be owed for pure economic loss caused by a negligent statement but might be owed if caused by a negligent act.

A duty of care will be owed for pure economic loss caused by a negligent statement and might be owed for pure economic loss caused by a negligent act.

A

A duty of care will not be owed for pure economic loss caused by a negligent act but might be owed if caused by a negligent statement.

174
Q

An employer provides a disparaging, yet accurate reference for an ex-employee. The ex-employee cannot find a job as a result.

Which of the following is correct in relation to whether the employer owed the employee a duty of care?

The employer owed the person requesting the reference a duty of care to provide an accurate reference, but not the ex-employee.

The employer owed the ex-employee a duty of care to provide an accurate reference.

The employer does not owe the ex-employee a duty of care because they have only suffered pure economic loss.

The employer did not owe the ex-employee a duty of care to provide an accurate reference.

The employer owed the ex-employee a duty of care to provide a reference.

A

The employer owed the ex-employee a duty of care to provide an accurate reference.

Correct
This is the correct answer.

This precedent for duty of care was established in Spring v Guardian Assurance Plc & Others [1995] 2 AC 296. A referee owes a duty of care to the subject of the reference to provide an accurate reference. It appears the employer has done this.

175
Q

Whist alive, Naghina’s deceased grandmother had told her that she would inherit £50,000. However, Naghina is told by the solicitors dealing with her grandmother’s will, that the will is invalid and that her grandmother’s previous will (which left everything to Naghina’s sister) would have to be followed instead. This is because Naghina’s grandmother failed to sign the most recent will. The solicitor’s copy was filed without checking the signature was present.

Which of the following is correct?

Naghina will not be owed a duty of care by the solicitors for the consequential economic loss she has suffered because she was not their client.

Naghina will not be owed a duty of care by the solicitors as she has suffered pure economic loss which is not recoverable.

Naghina will be owed a duty of care by the solicitors for the pure economic loss she has suffered as a result of the negligently filed will.

Naghina will be owed a duty of care by the solicitors for the consequential economic loss she has suffered as a direct result of the negligently filed will.

Naghina will not be owed a duty of care by the solicitors for the pure economic loss she has suffered because she was not their client.

A

Naghina will be owed a duty of care by the solicitors for the pure economic loss she has suffered as a result of the negligently filed will.

Correct
This is the correct answer.

Although Naghina has suffered pure economic loss (economic loss not flowing from damage to person or property) and the general rule is that this is not recoverable, there are exceptions to this general rule. Naghina’s loss falls into one of these exceptions.

There is well established precedent that solicitors drawing up a will owe a duty of care to the testator’s beneficiaries who should be allowed to sue the solicitor for the loss of an intended legacy. Naghina will therefore be owed a duty of care by her deceased grandmother’s solicitors for the loss of £50,000.

176
Q

A friend of Trevor’s provides him with a survey report for a house Trevor wants to buy. Trevor reads the report and buys the house. His friend failed to spot structural movement in the floorboards. The value of the house decreases by £100,000.

Which one of the following is correct?

Trevor has suffered pure economic loss caused by a negligent misstatement.

Trevor has suffered property damage and pure economic loss, both of which were caused by a negligent misstatement.

Trevor has suffered property damage caused by a negligent misstatement.

Trevor has suffered pure economic loss but has no claim in negligence as the survey was provided by his friend.

Trevor has suffered consequential economic loss caused by a negligent misstatement.

A

Trevor has suffered pure economic loss caused by a negligent misstatement.

This is the correct answer. Trevor has suffered financial loss as a result of his friend’s negligent survey. This will be categorised as pure economic loss because the house will be considered a defective product (when Trevor bought the house it was inherently defective). The decrease in value of the home does not flow from property damage or personal injury and is also therefore pure economic loss.

177
Q

Which case authority gave guidelines for the criteria to apply when determining whether the defendant assumed responsibility for the correctness of their statement to the claimant? What were these guidelines?

Hedley Byrne v Heller.

Guidelines: the defendant communicated advice to the claimant or knew it would be; the defendant knew the purpose for which the claimant would use the advice; the defendant knew or reasonably believed the claimant would rely on the advice; and the claimant acted on the advice to their detriment.

Caparo Industries plc v Dickman and others.

Guidelines: the defendant communicated advice to the claimant or knew it would be; the defendant knew the purpose for which the claimant would use the advice; the defendant knew or reasonably believed the claimant would rely on the advice; and the claimant acted on the advice to their detriment.

Hedley Byrne v Heller.

Guidelines: reasonable reliance by the claimant on the defendant’s advice; voluntary assumption of responsibility by the defendant for the advice; and a special relationship of trust and confidence between the defendant and the claimant.

Caparo Industries plc v Dickman and others.

Guidelines: reasonable reliance by the claimant on the defendant’s advice; voluntary assumption of responsibility by the defendant for the advice; and a special relationship of trust and confidence between the defendant and the claimant.

Banca Nazionale del Lavoro v Playboy Club & Others.

Guidelines: the defendant communicated advice to the claimant or knew it would be; the defendant knew the purpose for which the claimant would use the advice; the defendant knew or reasonably believed the claimant would rely on the advice; and the claimant acted on the advice to their detriment.

A

Caparo Industries plc v Dickman and others.

Guidelines: the defendant communicated advice to the claimant or knew it would be; the defendant knew the purpose for which the claimant would use the advice; the defendant knew or reasonably believed the claimant would rely on the advice; and the claimant acted on the advice to their detriment.

Correct
This is the correct answer. In Caparo Industries plc v Dickman and others, the claimant investor relied on an audit report prepared by the defendant for the shareholders. The defendant did not owe the claimant a duty of care as the report was not prepared for the claimant or for purpose of investing. The House of Lords set out the above guidelines which should be applied when determining whether the defendant assumed responsibility for the correctness of their statement to the claimant.

178
Q

If a claimant suffers pure economic loss as a result of relying on a negligent statement, and the statement contained a disclaimer: ‘this advice is provided without responsibility’, which statute or statutes might the disclaimer be subject to?

The Unfair Contract Terms Act 1977.

The Consumer Rights Act 2016 and the Unfair Contracts Terms Act 1975.

The Customer Rights Act 2015 and the Unfair Contracts Terms and Disclaimers Act 1977.

The Consumer Rights Act 2015.

The Consumer Rights Act 2015 or the Unfair Contract Terms Act 1977.

A

The Consumer Rights Act 2015 or the Unfair Contract Terms Act 1977.

Correct
This is the correct answer. The Consumer Rights Act 2015 (CRA 2015) regulates disclaimers between traders and consumers so will be relevant where the claimant is a consumer and the defendant a trader. The Unfair Contract Terms Act 1977 (UCTA 1977) regulates disclaimers in contracts between businesses, so will be relevant where both parties are acting in the course of business.

179
Q

The defendant obtained planning permission to extend their steel manufacturing factory. The factory is now just 50 metres from the claimant’s house. The claimant can no longer enjoy their property given the constant noise created by the lorries continuously arriving and leaving from the defendant’s factory. Which of the following is correct in relation to the tort of private nuisance?

The courts will consider whether the planning permission justifies the nuisance when assessing whether the defendant’s use of their land is reasonable.

The planning permission is completely immaterial to whether there has been private nuisance.

The planning permission will have changed the locality of the area from residential to commercial and there will therefore be no actionable private nuisance.

The planning permission will not authorise the nuisance. However, the terms of the planning permission may have a bearing on whether or not a nuisance exists.

The claimant cannot sue in private nuisance as the defendant had planning permission to extend their factory. The noise created is therefore reasonable.

A

The planning permission will not authorise the nuisance. However, the terms of the planning permission may have a bearing on whether or not a nuisance exists.

This is the correct answer. The planning permission does not make the defendant’s use of their land reasonable. The courts might consider the terms of the planning permission in order to help them decided whether the use of the land is reasonable. For example, if the planning permission allows the factory to operate between 9am and 3pm, and the lorries are arriving and leaving outside of these hours, this could indicate that the use of land is unreasonable.

180
Q

What losses are recoverable in private nuisance?

Physical damage to property, SPD, and consequential economic loss.

Physical damage to property, SPD, and pure economic loss.

Physical damage to person or property, SPD, and consequential economic loss.

Physical damage to person or property, SPD, pure economic loss, and consequential economic loss.

Physical damage to person, SPD, and pure economic loss.

A

Physical damage to property, SPD, and consequential economic loss.

This is the correct answer. These three types of losses are recoverable in private nuisance.

181
Q

What factors might the court consider when deciding whether the defendant’s use of land was unreasonable where the claimant’s loss is physical damage to property?

Time and duration, locality, lack of care, excessive behaviour, and public benefit.

Time and duration, abnormal sensitivity, lack of care, excessive behaviour, and public benefit.

Time and duration, lack of care, and excessive behaviour.

Time and duration, locality, abnormal sensitivity, malice, lack of care, excessive behaviour, and public benefit.

Time and duration, abnormal sensitivity, malice, lack of care, excessive behaviour, and public benefit.

A

Time and duration, abnormal sensitivity, malice, lack of care, excessive behaviour, and public benefit.

Correct
This is the correct answer. The courts can take into account any of these factors if relevant. Locality is omitted from the list as this is not relevant where the loss is physical damage to property.

182
Q

Rosie has just opened a yoga business next to a hairdresser. The hairdresser has operated at that site for 21 years. Rosie is finding it impossible to run her yoga business given the high level of noise made by the hairdresser. Prior to Rosie, her yoga premises were occupied by a small chocolate making company for 30 years. Which of the following is correct in the law of private nuisance?

The hairdresser can rely on the defence of prescription and contributory negligence.

The hairdresser can rely on the defence of prescription and consent.

The hairdresser will be unable to rely on the defence of prescription and will be unable to argue that Rosie moved to the nuisance.

The hairdresser will be able to rely on the defence of prescription and will be able to argue that Rosie moved to the nuisance.

The hairdresser will be able to rely on the defence of prescription but unable to argue that Rosie moved to the nuisance.

A

The hairdresser will be unable to rely on the defence of prescription and will be unable to argue that Rosie moved to the nuisance.

This is the correct answer. The hairdresser cannot rely on the defence of prescription as their activity only became a nuisance when Rosie moved in next door. We are told that she has only just done this. The 20 years starts running from when the activity becomes a nuisance. Moving to the nuisance is not currently a defence to a private nuisance claim.

incorrect

The hairdresser will be able to rely on the defence of prescription but unable to argue that Rosie moved to the nuisance.

This is incorrect. The hairdresser cannot rely on the defence of prescription as Rosie has only just moved in next door. Currently a defendant cannot argue that the claimant moved to the nuisance, so this part of the statement is correct. Please review this element to ensure you understand the different defences available to the defendant.

183
Q

Following on from question one, if the hairdresser was found to have committed a private nuisance, what remedy is Rosie most likely to be awarded?

Damages in lieu of an injunction.

Partial injunction.

Abatement.

Full injunction.

Damages.

A

Partial injunction.

This is the correct answer. Prima facie, Rosie would be awarded an injunction. This is likely to be a partial injunction, not a full injunction, given that the hairdresser has operated from the site for 21 years and it is likely that the business is of some public benefit (used by the public and as an employer). The courts could, for example, stipulate what type of machinery the hairdresser can use.

incorrect

Full injunction.

This is incorrect. Prima facie, Rosie would be awarded an injunction. However, it is unlikely to be a full injunction as this would require the hairdresser to stop operating all together. Please review this element to ensure you understand what remedies are available to a claimant in private nuisance and when.

184
Q

Simon buys a house with a huge garden. Fifteen years before Simon purchased the house, the local authority installed a water pipe at the very back of his garden. It was not installed properly and frequently leaks causing flooding on the next-door neighbour’s property. Simon is unaware of the water pipe or that it is leaking as his neighbour has not mentioned the flooding to him.

Which defence/s might Simon be able to rely upon if his neighbour sues him in the law of private nuisance?

Act of a third party.

Prescription.

Act of God.

Statutory authority.

Consent.

A

Act of a third party.

This is the correct answer. The nuisance has been created by a third party (the local authority) for whom Simon bears no responsibility. He has not carried on the nuisance nor is it reasonable for him to put a stop to it as he is unaware of the pipe and that it is leaking.

185
Q

Mishal has a party at her house. Her friends park their cars across many of Mishal’s neighbours’ driveways so the neighbours cannot get out. Gita is a beautician who visits her clients at their houses. She is parked in one of her client’s driveways when it is blocked by Mishal’s friend’s car. As a result, she cannot leave for the rest of the afternoon and so she misses her remaining appointments. She suffers a loss of income.

Which of the following is correct in the tort of public nuisance?

A class of people have been affected by Mishal’s actions. Gita cannot bring a claim as an individual.

A class of people have been affected by Mishal’s actions. Gita cannot sue Mishal in public nuisance as it was not Mishal’s car blocking the client’s driveway.

A class of people have been affected by Mishal’s actions. Gita cannot bring a claim as she does not own the land affected.

A class of people have been affected by Mishal’s actions. Gita can bring a claim for her economic loss as an individual.

A class of people have been affected by Mishal’s actions. Gita cannot bring a claim for economic loss in public nuisance.

A

A class of people have been affected by Mishal’s actions. Gita can bring a claim for her economic loss as an individual.

This is the correct answer. It is likely a class of people have been affected as we are told that many neighbours have been blocked in. Gita can bring a claim as an individual as she has suffered special damage (i.e. over and above the rest of the class). The class have suffered inconvenience. Gita has suffered inconvenience and economic loss. Economic loss is recoverable in the tort of public nuisance.

incorrrect
A class of people have been affected by Mishal’s actions. Gita cannot bring a claim as an individual.

This is incorrect. It is likely a class of people have been affected as we are told that many neighbours have been blocked in. However, Gita should be able to sue as an individual as she has suffered special damage. Please review this element to ensure you understand who can sue in public nuisance and the types of losses that are recoverable in public nuisance.

186
Q

As a joke, Troy sends 50 of his friends envelopes full of talcum powder with a note saying ‘anthrax’. Which of the following is correct in the law of public nuisance?

Troy’s friends cannot bring a claim in public nuisance as they have suffered personal injury.

Troy’s friends cannot bring a claim in public nuisance as Troy’s acts amount to a crime.

Troy’s friends cannot bring a claim in public nuisance as they have not suffered a common injury.

Troy’s friends cannot bring a claim in public nuisance as they consented to Troy’s actions by being his friends.

Troy’s friends cannot bring a claim in public nuisance as 50 people will not amount to a ‘class’.

A

Troy’s friends cannot bring a claim in public nuisance as they have not suffered a common injury.

This is the correct answer. In order to be defined as a ‘class’, the friends must have suffered a ‘common injury’, i.e. be affected at more or less the same time and in the same location. This has not happened here. For example, in R v Rimmington the defendant sent around 500 racially offensive items to people across the country. The people who received the items were not a ‘class’ in the way required by public nuisance.

incorrect

Troy’s friends cannot bring a claim in public nuisance as 50 people will not amount to a ‘class’.

This is incorrect. There is no exact number of people who have to be affected for there to be a public nuisance. It will depend upon the facts. Please review this element to ensure you understand how to determine whether a class has been affected.

Troy’s friends cannot bring a claim in public nuisance as Troy’s acts amount to a crime.

This is incorrect. Even though Troy’s actions might amount to a crime, the claimants could also bring a claim in public nuisance if they wanted common law damages as a remedy. Please review this element to ensure you understand what can amount to a public nuisance.

187
Q

What losses are recoverable in public nuisance?

Property damage, consequential economic loss, personal injury, pure economic loss, and inconvenience.

Property damage, consequential economic loss, personal injury, pure economic loss, and SPD.

Property damage, consequential economic loss, personal injury, and inconvenience.

Property damage, consequential economic loss, personal injury, and pure economic loss.

Property damage, consequential economic loss, pure economic loss, and inconvenience.

A

Property damage, consequential economic loss, personal injury, pure economic loss, and inconvenience.

This is the correct answer. A wide range of losses are recoverable in public nuisance. Personal injury and pure economic loss are recoverable unlike in private nuisance and Rylands v Fletcher.

incorrect

Property damage, consequential economic loss, personal injury, pure economic loss, and SPD.

This is incorrect. SPD is relevant to private not public nuisance. Please review this element to ensure you understand the types of loss recoverable in public nuisance.

188
Q

What losses are recoverable in the rule in Rylands v Fletcher?

Property damage, consequential economic loss, and SPD.

Property damage, consequential economic loss, and personal injury.

Property damage and pure economic loss.

Property damage and consequential economic loss.

Property damage.

A

Property damage and consequential economic loss.

This is the correct answer. These are the only two types of loss that are recoverable in the rule in Rylands v Fletcher.

189
Q

Max operates a mechanics business from his home. He stores chemicals used to “cure” car body filler. The chemicals are stored in barrels. However, there is a crack in one of the barrels, and the chemicals leak onto Max’s neighbour’s property killing all his neighbour’s plants. The presence of the crack was not known about and the leak was not apparent.

Which of the following is most accurate in the tort of Rylands v Fletcher?

The neighbour will be able to claim for damage to his plants as any property damage caused by an escape of a substance from the defendant’s land is recoverable.

The neighbour will be unable to claim for damage to his plants as Max did not know about the leak.

The neighbour will be able to claim for damage to his plants as the harm suffered was foreseeable if the chemicals escaped.

The neighbour will be unable to claim for damage to his plants as the escape of the chemicals was not foreseeable.

The neighbour will be able to claim for damage to his plants as Max is responsible for anything he brings onto his land.

A

The neighbour will be able to claim for damage to his plants as the harm suffered was foreseeable if the chemicals escaped.

This is the correct answer. It was reasonably foreseeable that if the chemicals escaped, they could kill plants. The escape does not need to be foreseeable.

190
Q

Which defences are applicable to the rule in Rylands v Fletcher?

Common benefit, act or default of the claimant, statutory authority, act of third party, act of God, and prescription.

Common benefit, statutory authority, act of third party, act of God, contributory negligence and consent.

Common benefit, act or default of the claimant, statutory authority, act of third party, and act of God.

Common benefit, act or default of the claimant, statutory authority, act of third party, act of God, and contributory negligence.

Common benefit, act or default of the claimant, statutory authority, act of third party, act of God, contributory negligence and consent.

A

Common benefit, act or default of the claimant, statutory authority, act of third party, act of God, contributory negligence and consent.

This is the correct answer. These are all the relevant defences.

191
Q

During working hours, a building contractor is injured on the occupier’s premises when he falls down a steep staircase with no handrail. Which of the following is correct in explaining why the contractor is a visitor?

He has implied permission to use the stairs.

He has contractual permission to be on the premises.

Under s.2(6) OLA 1957 he has lawful authority to be on the premises.

He has express and contractual permission to be on the premises.

He has express permission to be on the premises.

A

He has express and contractual permission to be on the premises.

This is the correct answer. The contractor has been invited onto the occupier’s premises and will have entered into a contract with the occupier to carry out the building work.

incorrect

He has contractual permission to be on the premises.

This is incorrect. The contractor would have entered into a contract with the occupier to carry out building work but would be a visitor for an additional reason too. Please review this element to ensure you understand how a claimant might be considered a visitor.

He has express permission to be on the premises.

This is incorrect. The contractor has been invited onto the occupier’s premises but would be a visitor for an additional reason too. Please review this element to ensure you understand how a claimant might be considered a visitor.

192
Q

A university student, Jessica, visits a castle that is open to the public. There is a small sign at the entrance of the castle which, amongst other conditions, informs visitors that they must not enter the outhouse at the very back of the castle grounds. Jessica does not see the sign. She enters the outhouse and twists her ankle on some very uneven paving slabs. Which of the following statements is correct?

Jessica will be classed as a visitor as she had express permission to be on the premises.

Jessica will be classed as a visitor as she had implied permission to enter the outhouse.

Jessica will be classed as a visitor as she had not seen the sign denying access to the premises.

Jessica will not be classed as a visitor as she did not have permission to enter the outhouse.

Jessica will be classed as a visitor as she had lawful authority to enter the outhouse.

A

Jessica will be classed as a visitor as she had express permission to be on the premises.

This is the correct answer. Jessica has express permission to visit the castle which is open to the public. The castle has not gone far enough to restrict her permission by area and turn her status into that of a trespasser when she entered the outhouse. The sign was very small containing lots of other information and does not appear to be located near the outhouse itself.

incorrect
Jessica will be classed as a visitor as she had implied permission to enter the outhouse.

This is incorrect. Jessica need not rely on implied permission. She has express permission to be on the castle’s grounds. Please review this element to ensure you understand the categories of visitor and when a visitor’s permission can be restricted.

193
Q

A nine year old boy is at a playground with his mother. The playground is owned by the local authority. The boy falls over and cuts his knee badly on glass from a broken bottle. His mother was not watching him at the time. Which of the following is correct under the OLA 1957?

The local authority owes the boy a higher standard of care than adult visitors and it is likely to be in breach of duty given the playground is an area commonly used by children.

The local authority owes the boy the same standard of care as adult visitors. They are not in breach as an adult would have seen the broken bottle.

The local authority owes the boy a higher standard of care than adult visitors, but it is unlikely to have breached its duty as the boy’s mother should have been supervising him.

The local authority owes the boy a higher standard of care than adult visitors but it is unlikely to be in breach of duty as it is not their fault someone left a broken bottle at the playground.

The local authority owes the boy a higher standard of care than adult visitors and it is likely to be in breach as the boy has been injured.

A

The local authority owes the boy a higher standard of care than adult visitors and it is likely to be in breach of duty given the playground is an area commonly used by children.

This is the correct answer. Under s.2(3)(a) OLA 1957 occupiers owe children a higher standard of care. There is likely to be a breach here as the local authority should have got rid of the broken bottle. Children will regularly be in the area, so it is important to take extra precautions in order to keep the premises reasonably safe.

194
Q

Dushal is a professional electrician. He is working at a wealthy client’s house when the wooden stairs give way. He falls and suffers serious injuries. The client thought the stairs required some maintenance work.

Which of the following is correct under the OLA 1957?

Dushal is owed a lower standard of care than the ordinary visitor as he is a person entering in the exercise of a calling. It is therefore unlikely the client is in breach of duty as Dushal should have protected against all risks.

Dushal is owed a lower standard of care than the ordinary visitor as he is a person entering in the exercise of a calling. However, the client is likely to be in breach of duty as the risk of the stairs collapsing was not a risk incidental to being an electrician.

Dushal is owed a higher standard of care than the ordinary visitor as he is a professional visitor. It is likely that the client is in breach of duty.

Dushal is owed the same standard of care as any ordinary visitor. It is likely that the client is in breach of duty.

Dushal is owed the same standard of care as an ordinary visitor. It is unlikely that the client is in breach of duty as it is known that stairs can be dangerous.

A

Dushal is owed the same standard of care as any ordinary visitor. It is likely that the client is in breach of duty.

This is the correct answer. Dushal will be owed the standard of care under s.2(2) OLA 1957. Although he is a person entering in exercise of a calling, a lower standard is only owed in relation to risks incidental to the person’s trade/job. The risk of stairs collapsing is not a risk incidental to being an electrician. The occupier is likely to be in breach if the stairs were unsafe: he knew the stairs required maintenance and had the money to repair them.

195
Q

An occupier of premises puts up a large sign on their electrical fence saying “Danger. Electrical fence. Do not touch.” A visitor touches the fence and is electrocuted. Which of the following is correct under the OLA 1957?

The occupier has discharged their duty of care to the visitor as they have warned of the danger concerned (s.2(4)(a) OLA 1957).

The occupier has discharged their duty of care to the visitor as the warning was enough to ensure the premises were reasonably safe (s.2(4)(a) OLA 1957).

The occupier has discharged their duty of care to the visitor as the warning is in writing (s.2(4)(a) OLA 1957).

The occupier has discharged their duty of care to the visitor as the warning was enough to enable the visitor to be reasonably safe (s.2(4)(a) OLA 1957).

The occupier has discharged their duty of care to the visitor as the danger was an obvious danger (s.2(4)(a) OLA 1957).

A

The occupier has discharged their duty of care to the visitor as the warning was enough to enable the visitor to be reasonably safe (s.2(4)(a) OLA 1957).

This is the correct answer. Under s.2(4)(a) OLA 1957 the occupier discharges the duty of care owed to a visitor if they warn the visitor of the danger concerned, and the warning is enough to enable the visitor to be reasonably safe. Case law suggests that this means informing the visitor of what the danger is, where it is and how to avoid this. The occupier’s warning (“Danger. Electrical fence. Do not touch.”) does this.

196
Q

A university student is temporarily banned from the university premises. He enters the premises to meet some of his friends at the student bar. When he is leaving, he falls down a steep staircase which has no handrail or lighting. He breaks his neck and is left paralysed.

Which of the following is correct under the OLA 1984?

The student will be owed a duty of care by the university if they were aware of the danger, knew that someone would be in the vicinity of the danger, and the risk was one against which the university ought to offer protection.

The student will not be owed a duty of care by the university as he was banned from the university premises.

The student will not be owed a duty of care by the university as he has suffered personal injury.

The student will be owed a duty of care by the university if they were aware of the danger and knew the claimant would be in the vicinity of the danger.

The student will not be owed a duty of care by the university as his injuries were caused by his activity on the premises, rather than the state of the premises themselves.

A

The student will be owed a duty of care by the university if they were aware of the danger, knew that someone would be in the vicinity of the danger, and the risk was one against which the university ought to offer protection.

This is the correct answer. These are the three criteria which a claimant must satisfy in order to prove that they are owed a duty of care under the OLA 1984 (ss.1(3)(a)-(c)).

197
Q

Following on from question one, the university had erected a temporary barrier to try and prevent students using the staircase. The student had difficulty climbing over the barrier to get to the staircase. Which of the following is correct under the OLA 1984?

A physical barrier is likely to be a sufficient warning under the OLA 1984.

A physical barrier is not a sufficient warning under the OLA 1984 especially as the student was able to climb over it.

A physical barrier is not sufficient for the purpose of warning of a danger under the OLA 1984. A sufficient warning must make it clear what the danger is, where it is and how to avoid it.

The university do not need to warn trespassers of any dangers.

A physical barrier is not sufficient for the purpose of warning of a danger under the OLA 1984. There should have been a written sign accompanying the barrier denying access.

A

A physical barrier is likely to be a sufficient warning under the OLA 1984.

This is the correct answer. It is easier for the occupier to discharge their duty under the OLA 1984 than the OLA 1957. Under s.1(5) OLA 1984 the occupier will discharge their duty of care if they have warned the trespasser of the danger or discouraged persons from incurring the risk. The physical barrier is likely to be enough to discourage persons from incurring the risk, as it was in the case of Titchener v British Railway Board.

198
Q

A defendant can argue the following defences in response to a claim under the OLA 1984:

Contributory negligence and illegality.

Consent and contributory negligence.

Consent, contributory negligence and illegality.

Consent and illegality.

Necessity and consent.

A

Consent and contributory negligence.

This is the correct answer.

199
Q

A client would like advice on whether they can exclude their liability under the OLA 1957. The client owns a small sweet shop and wants to protect themselves against claims from customers who injure themselves on their premises.

Which of the following is correct?

The exclusion notice will be subject to UCTA 1977. The client can exclude liability for any type of loss provided the consumer notice is considered reasonable.

The exclusion notice will be subject to the CRA 2015. The client cannot exclude liability for any personal injury caused by their negligence but can for other types of loss if the notice is considered fair.

The exclusion notice will be subject to the CRA 2015. The client can exclude liability for any type of loss provided the consumer notice is considered fair.

The exclusion notice will be subject to UCTA 1977. The client cannot exclude liability for any personal injury caused by their negligence but can for other types of loss if the notice is reasonable.

The exclusion notice will be subject to common law restrictions. The client can exclude liability for any type of loss if it is considered reasonable to do so.

A

The exclusion notice will be subject to the CRA 2015. The client cannot exclude liability for any personal injury caused by their negligence but can for other types of loss if the notice is considered fair.

This is the correct answer.

The customers would be consumers and the client would be a trader. The CRA 2015 would therefore apply to any consumer notice. Section 65(1) CRA 2015 says that a trader cannot by a term of a consumer contract or by a consumer notice exclude or restrict liability for death or personal injury resulting from negligence (this includes a breach under the OLA 1957).In respect of damage other than death or personal injury, an unfair notice is not binding on the consumer (s.62(2) CRA 2015).

200
Q

Which of the following statements is correct?

Under the OLA 1957, exclusion/limitation clauses can be subject to section 3 OLA 1957; UCTA 1977; CRA 2015; or common law restrictions. Under the OLA 1984, exclusion/limitation clauses can be subject to UCTA 1977; CRA 2015; or common law restrictions.

Under the OLA 1957 and the OLA 1984, exclusion/limitation clauses can be subject to section 3 OLA 1957; UCTA 1977; CRA 2015; or common law restrictions.

Under the OLA 1957 and OLA 1984, exclusion/limitation clauses can be subject to UCTA 1977; CRA 2015; or common law restrictions.

Under the OLA 1984, exclusion/limitation clauses can be subject to section 3 OLA 1957; UCTA 1977; CRA 2015; or common law restrictions.

Under the OLA 1957, exclusion/limitation clauses can be subject to section 3 OLA 1957; UCTA 1977; CRA 2015; or common law restrictions.

A

Under the OLA 1957, exclusion/limitation clauses can be subject to section 3 OLA 1957; UCTA 1977; CRA 2015; or common law restrictions.

201
Q

A householder invites friends over for lunch. They are having a tennis court built on their property and have erected a sign at the entrance to their property reading “Tennis court under construction on this property and the construction site is dangerous. Under no circumstances will the occupier be liable for any injury caused to those on/near the construction site.” One of the householder’s friends decides to look at the tennis court. The friend’s heel gets stuck in wet clay, causing them to fall. They break their ankle and wrist. The friend sues the householder for breach of duty under the Occupiers’ Liability Act 1957.

Which of the following statements is correct?

The sign is an exclusion notice but will be subject to section 3 OLA 1957, so the householder is unable to exclude or restrict the duty of care that they owe their friend.

The sign is an exclusion notice and may operate as a potential defence to the claim.

The sign is an exclusion notice but will be subject to the CRA 2015, so the householder is unable to exclude liability for personal injury.

The sign is an exclusion notice but will be subject to the UCTA 1977 and the CRA 2015, so the householder is unable to exclude liability for personal injury.

The sign is an exclusion notice but will be subject to the UCTA 1977, so the householder is unable to exclude liability for personal injury.

A

The sign is an exclusion notice and may operate as a potential defence to the claim.

This is the correct answer.

The exclusion notice will not be subject to UCTA 1977 or the CRA 2015. The householder is a private occupier, so not occupying the premises for business purposes or acting as a trader. The exclusion notice may be subject to the common law restrictions but seems unlikely to succeed here, although further information needed.

202
Q

Can a party bring a claim on the basis of both negligence and the Consumer Protection Act 1987?

Yes

No

A

Yes

for product liability

203
Q

The Consumer Protection Act 1987 aimed to introduce a strict liability regime, meaning:

A regime where parties could be found liable without it being necessary to show fault on their part

A regime which holds a manufacturer strictly liable for all losses which it causes, regardless of the normal rules on remoteness.

A regime which increased the protections available to consumers, beyond those under the general rules of negligence.

A regime which applies to all manufacturers / producers.

A

A regime where parties could be found liable without it being necessary to show fault on their part

Correct. Well done! None of the other explanations reflect the meaning of ‘strict liability’, although it is true that the Act aimed to increase the protections available to consumers, beyond those under the general rules of negligence.

204
Q

Under the Act, liability potentially arises in relation to damage caused by a ‘defect’ in a product. A product is defective if it…

is not as safe as people are generally entitled to expect

is not fit for the purpose for which it was designed.

is not fit for the purposes for which it is being used.

causes loss other than by the negligence / carelessness of the user.

A

is not as safe as people are generally entitled to expect

Correct. The other answers state matters which might be relevant but which are not strictly what is meant by ‘defect’.

205
Q

John buys a drying machine (for drying clothes). The drying machine catches fire during normal use, damaging the house owned by him and his wife (the damage exceeds £2,000). The attendance of the fire service stops customers attending a sandwich shop next door, and so the sandwich shop suffers a substantial loss of profits. Who can potentially bring a claim under the Act?

No party can claim.

John and his wife

Just John.

John, his wife and the sandwich shop.

A

John and his wife

Correct
Correct. John can claim because he has suffered property damage in excess of £275 and the property is intended for private use. His wife can claim because she has suffered property damage in excess of £275 and the property is intended for private use – it doesn’t matter that she didn’t purchase the product. The sandwich shop cannot claim – it has not suffered death, personal injury or damage to any property (even if it had suffered damage to any property it is unlikely that property would have been intended and ordinarily intended for private use).

206
Q

Can exemption clauses exclude or limit liability for claims under the CPA 1987?

Yes. It is possible to exclude and limit liability under the Act.

No. It is not possible to exclude or limit liability under the Act.

Exemption clauses can only exclude or limit liability under the Act to the extent that it is reasonable to do so.

Exemption clauses can only exclude or limit liability for property damage under the Act

A

No. It is not possible to exclude or limit liability under the Act.

Correct. Section 7 CPA 1987 prohibits any exclusion or limitation of liability.

207
Q

A pharmaceutical company produces medication for a rare blood disorder. The medication is subsequently found to be defective causing serious side effects to its consumers. At the time the product was produced the state of scientific and technical knowledge was not such that would have enabled the medication to be produced without the side effects. Can the company rely on this as a defence under the Consumer Protection Act 1987?

The defence under s4(1)(e) CPA 1987 could only be relied upon if the company could show that the side effects caused were not reasonably foreseeable.

No, the defence under s4(1)(e) CPA 1987 only applies where the state of scientific and technical knowledge was not such that the defect could have been discovered.

Yes, the defence under s4(1)(e) CPA 1987 could be relied upon.

No, there are no defences to a claim under the CPA 1987.

A

No, the defence under s4(1)(e) CPA 1987 only applies where the state of scientific and technical knowledge was not such that the defect could have been discovered.

Correct. Section 4(1)(e) refers to an inability to discover, not an inability to fix. If a manufacturer is aware of a defect but the state of scientific / technical knowledge is such that the defect cannot be fixed, this will not be a defence.

208
Q

Donoghue v Stevenson [1932] AC 562 is authority for the proposition that a manufacturer owes a duty of care to those people who use its products. Does a manufacturer owe a duty to a party that neither bought nor used the product, but who comes into contact with it?

No

Yes

A

Yes

Correct - Stennett v Hancock [1939] 2 All ER 578

209
Q

Applying the rules of negligence, can a party recover in negligence in relation to the loss of the product itself?

No – this sort of loss is rarely foreseeable.

Yes – this loss is highly foreseeable.

Yes – the loss of the product is property damage.

No – this would ordinarily be considered pure economic loss, and not recoverable.

A

No – this would ordinarily be considered pure economic loss, and not recoverable.

210
Q

When assessing whether a manufacturer has breached any duty owed, the key question is…

Whether the manufacturer fell below the standard of a reasonably competent manufacturer.

Whether the products reached a reasonable standard.

Whether the manufacturer could foresee harm being caused by the product.

Whether the manufacturer failed to produce a product which meets the expectations of a reasonable man.

A

Whether the manufacturer fell below the standard of a reasonably competent manufacturer.

Correct. The focus has to be on the manufacturer, rather than on the product – this is perhaps a difference between liability in tort and liability under the Consumer Protection Act 1987. Foreseeability of harm is not enough to show breach.

211
Q

Due to a driver’s negligent driving, his passenger was seriously injured in a road traffic accident. When emergency services attended the accident scene, the car was found to contain a large packet of cocaine which the driver and passenger were planning to sell later.

Which one of the following statements best describes the position in relation to defences in the tort of negligence?

Select one alternative:

It is likely the driver can rely on the defence of illegality because both he and his passenger were planning to sell cocaine. The passenger’s claim would be negated.

It is unlikely the driver can rely on the defence of illegality because there was not a direct link between the passenger’s injury and the illegal act.

It is likely the driver can rely on the defence of illegality because both he and his passenger were planning to sell cocaine. The passenger’s damages would be reduced.

It is unlikely the driver can rely on the defence of illegality because the crime committed by the passenger was not sufficiently serious.

It is likely the driver can rely on the defence of illegality because the cocaine was in the car. The passenger’s claim would be negated.

A

It is unlikely the driver can rely on the defence of illegality because there was not a direct link between the passenger’s injury and the illegal act.

212
Q

A driver drives through a red light, hitting a pedestrian. The pedestrian breaks her arm and hits her head. A witness to the accident notices that the pedestrian is struggling to breathe. The witness uses her penknife to make an incision into the pedestrian’s windpipe, helping the pedestrian to breathe more easily. However, the pedestrian later suffers from an infection and dies. The infection was caused by the penknife together with the pedestrian’s weak condition which affected her ability to fight infection.

Which of the following statements is most accurate in relation to factual causation for the pedestrian’s negligence claims?

(2 marks)

Select one alternative:

The driver is the factual cause of the pedestrian’s broken arm, head injury and death. The witness is the factual cause of the pedestrian’s death.

The driver is the factual cause of the pedestrian’s broken arm and head injury. The witness is the factual cause of the pedestrian’s death.

The driver and witness are the factual cause of the pedestrian’s death.

The driver is the factual cause of the pedestrian’s broken arm, head injury and death. The witness is not the factual cause of any of the pedestrian’s injuries.

The driver is not the factual cause of any of the pedestrian’s injuries. The witness is the factual cause of the pedestrian’s death.

A

The driver is the factual cause of the pedestrian’s broken arm, head injury and death. The witness is the factual cause of the pedestrian’s death.

213
Q

A man has been drinking in the pub and is very drunk. One of his friends, who has not been drinking alcohol, offers to give the man a lift home on her motorbike. She explains to the man that since the motorbike’s main headlight is broken they will have to proceed slowly, using the bike’s sidelights. The man accepts the lift but fails to fasten his helmet properly and he smokes an illegal marijuana cigarette on the way home. The rider negligently misjudges a bend and crashes. The man’s left leg is broken in the crash.

Which of the following statements best describes the effectiveness of any defence/s the rider might seek to rely upon?

The defence of consent would succeed as the man had full knowledge of the danger of riding without lights.

The defence of illegality would succeed as the man was committing a criminal act at the time of the accident.

The defence of consent would not succeed as the man is too drunk to be capable of giving valid consent.

The defence of contributory negligence would not succeed as the man’s failures did not contribute to the accident.

The defence of contributory negligence would not succeed as the man’s failures did not contribute to his losses.

A

The defence of contributory negligence would not succeed as the man’s failures did not contribute to his losses.

214
Q

A local council have put a substantial amount of yellow and black tape across the doorway of a wooden castle
in a children’s playground, as it has been damaged by fire. Teenagers are known to visit the playground in the
evenings, although the large sign on the gate says ‘No entry after 5pm’. The gate is locked at 5pm. Late one
evening (about 10pm), a teenager jumps over the gate and removes the tape across the doorway of the
wooden castle. They climb up the staircase of the castle. The staircase collapses and they fall, breaking their
leg.
Which of the following is most accurate in relation to duty of care and breach of duty in a claim by the teenager
against the local council in occupiers’ liability?

The local council does not owe a duty of care to the teenager under the Occupiers’ Liability Act 1984 as
the risk is not one which the local council would reasonably be expected to protect against, so there is no
breach.

The local council does owe a duty of care to the teenager under the Occupiers’ Liability Act 1984, but
they are not in breach having taken reasonable steps to warn of the danger.

The local council does owe a duty of care to the teenager under the Occupiers’ Liability Act 1984, and
they are in breach as they have failed to place a warning sign to warn of the specific danger.

The local council does not owe a duty of care to the teenager under the Occupiers’ Liability Act 1984 as
the teenager is a trespasser, so there is no breach.

The local council does not owe a duty of care to the teenager under the Occupiers’ Liability Act 1984 as
the teenager chose to climb up the staircase, so there is no breach.

A

The local council does owe a duty of care to the teenager under the Occupiers’ Liability Act 1984, but
they are not in breach having taken reasonable steps to warn of the danger.

215
Q

A university is building a new library on campus on land owned by the university. It has contracted with a
building firm to build it. At the end of a working day, the builders leave the building site on which there is an
uncovered hole. There is a large sign at the entrance to the building site which states ‘Be careful - building
work in progress’. Late that night a student, who has had several alcoholic drinks, does not notice the sign,
and falls into the hole, breaking their arm.
Which is the most accurate statement as to who would be an occupier in the student’s claim under the
Occupiers’ Liability Act 1957?
(2 marks)
Select one alternative:
The university and the building firm are both occupiers because they both have a sufficient degree of
control over the building site.
The building firm is the occupier because it has the most control over the building site. The university is
not an occupier.
The building firm is the occupier because it has a sufficient degree of control over the building site. The
university is not an occupier.
The university is the occupier because it owns the building site. The building firm is not an occupier.
The university is the occupier because it has a sufficient degree of control over the building site. The
building firm is not an occupier.

A

The university and the building firm are both occupiers because they both have a sufficient degree of
control over the building site.

216
Q

A man buys a bicycle costing £500 new from a retailer of sports equipment and cycles home. The front wheel
falls off and he is thrown over the handlebars. He suffers a broken arm. His personal injury is valued at £150.
His watch is broken - it has been valued at £300.
Assuming liability can be established under the Consumer Protection Act 1987 (CPA), which of these losses
can he recover under that Act?

The man can recover damages for his personal injury and damaged watch.

The man can recover damages for his damaged watch only.

The man can recover damages for his personal injury, damaged watch and damaged bicycle.

The man can recover damages for the damaged bicycle only.

The man can recover damages for his personal injury only.

A

The man can recover damages for his personal injury and damaged watch.

217
Q

A seven day food festival is held opposite a car repair garage run by a sole trader. During the seven days the
garage can do no business at all; no customers are able to access the garage to drop off their vehicles,
because the traffic around the area is so heavy, and the access points are blocked by parked coaches. There
are no other businesses on the affected road, but several residents do live on the road, and they were
inconvenienced by the heavy traffic.
Which of the following represents the most accurate advice in relation to a public nuisance claim against the
organiser of the festival?
(2 marks)
Select one alternative:
The owner of the garage cannot sue as an individual as their losses were not reasonably foreseeable.
The owner of the garage can sue as an individual as they have suffered special damage.
The owner of the garage cannot sue as an individual as they have not suffered special damage. They
will have to ask the attorney general to sue on their behalf and their neighbours who were affected by the
traffic.
The owner of the garage cannot sue as an individual as their losses are not direct and substantial.
The owner of the garage cannot sue as an individual as they have not suffered special damage. They
will have to ask the local authority to sue on their behalf and their neighbours who were affected by the
traffic

A

The owner of the garage can sue as an individual as they have suffered special damage.

218
Q

A roofing company is repairing a roof over pools at a fish farm that farm salmon and cod. The roofing
company’s contract is with the owner of the building, not with the fish farm which is a tenant of the building. The
scaffolding has been poorly constructed by the roofing company and falls into some of the salmon pools.
These pools are filled with debris which kills the salmon. The fish farm has to close for a week while these
pools are cleared, re-filled and re-stocked with new salmon. The fish farm wishes to claim for the dead salmon,
the profit that would have been made on those salmon when sold in their shop and the loss of profits on the
cod that were not killed but would have been sold during the week that the pools were closed.
Which of the following reflects the best advice for the fish farm if they succeed in an action in negligence
against the roofing company?
(2 marks)
Select one alternative:
The fish farm will not be able to recover any of their losses.
The fish farm will be able to recover all their losses including the value of the dead salmon, the lost profits
from not being able to sell the dead salmon, and the lost profits on the cod that could not be sold during
the week of closure.
The fish farm will be able to recover for the value of the dead salmon and the lost profits on the cod that
could not be sold during the week of closure, but will not be able to recover the lost profits from not being
able to sell the dead salmon.
The fish farm will be able to recover the value of the dead salmon, plus the lost profits from not being able
to sell those dead salmon, but will not be able to recover lost profits on the cod that could not be sold
during the week of closure.
The fish farm will be able to recover the value of the dead salmon, but will not be able to recover the lost
profits from not being able to sell those dead salmon or the cod that could not be sold during the week of
closure.

A

The fish farm will be able to recover the value of the dead salmon, plus the lost profits from not being able
to sell those dead salmon, but will not be able to recover lost profits on the cod that could not be sold
during the week of closure.

219
Q

A company is demonstrating a new type of helicopter. The helicopter is manned by one pilot and the pilot’s
husband attends the demonstration. Due to the company’s negligence, the helicopter explodes during the
demonstration and the pilot dies. The pilot’s husband watches the incident from three quarters of a mile away.
He subsequently suffers from clinical depression.
Which of the following statements best describes the husband and whether the company owed him a duty of
care for his psychiatric harm?
(2 marks)
Select one alternative:
The husband is a primary victim because he would have reasonably feared for his own safety at the
demonstration. He is unlikely to establish that the company owed him a duty of care as his injury was not
reasonably foreseeable.
The husband is a secondary victim because he was fearful for the safety of the pilot. He is unlikely to
establish that the company owed him a duty of care as he will be unable to show close ties of love and
affection with the victim.
The husband is neither a primary nor secondary victim. He is unlikely to establish that the company owed
him a duty of care.
The husband is a secondary victim because he was fearful for the safety of the pilot. He is likely to
establish that the company owed him a duty of care as he has close ties of love and affection with the
victim, and he witnessed the incident.
The husband is a primary victim because he would have reasonably feared for his own safety at the
demonstration. He is likely to establish that the company owed him a duty of care as there is sufficient
geographical proximity and it would be fair, just and reasonable.

A

The husband is a secondary victim because he was fearful for the safety of the pilot. He is likely to
establish that the company owed him a duty of care as he has close ties of love and affection with the
victim, and he witnessed the incident.

220
Q

A hotel instructs a gas engineer to check their boiler. The boiler has been leaking carbon monoxide but the
hotel was not aware of this. The gas engineer has not brought their carbon monoxide detector. They also
forget to switch off the boiler and open the window. The gas engineer ends up with carbon monoxide
poisoning and consequently loses sight in one eye.
Which statement best describes whether the hotel breached its duty of care to the gas engineer under the
Occupiers’ Liability Act 1957?
(2 marks)
Select one alternative:
The hotel did breach its duty to the gas engineer as it should have known about and warned the gas
engineer about the carbon monoxide leak.
The hotel did not breach its duty to the gas engineer as the gas engineer failed to take care of their own
safety.
The hotel did not breach its duty to the gas engineer as it may reasonably have expected that the gas
engineer would have appreciated and guarded against the risk of a carbon monoxide leak.
The hotel did breach its duty to the gas engineer as it should have provided the gas engineer with a
carbon monoxide detector.
The hotel did not breach its duty to the gas engineer as it was not aware of the carbon monoxide leak.

A

The hotel did not breach its duty to the gas engineer as it may reasonably have expected that the gas
engineer would have appreciated and guarded against the risk of a carbon monoxide leak.

221
Q

A woman lives next door to a man with two children. The man is painting the outdoor of his house. He places a
large barrel of paint against the fence that divides his garden from the garden of the woman next door. Whilst
playing in the garden, his two children push the barrel of paint over and the paint starts leaking into the
woman’s garden. The paint kills her award winning sunflowers which she sells for a profit.
Which of the following statements is most accurate in relation to a claim by the woman under the tort of
Rylands v Fletcher?
(2 marks)
Select one alternative:
The woman might succeed in Rylands v Fletcher because the escape of the paint was reasonably
foreseeable.
The woman might succeed in Rylands v Fletcher as the children reasonably foresaw the risk of damage
as a result of pushing over the paint.
The woman might succeed in Rylands v Fletcher because the type of damage suffered was reasonably
foreseeable in the event of the paint escaping from the man’s land.
The woman might succeed in Rylands v Fletcher because the escape of the paint was reasonably
foreseeable and, as a result, so was the damage suffered.
The woman might succeed in Rylands v Fletcher but will not be able to recover her consequential
economic loss, as this is not recoverable in Rylands v Fletcher.

A

The woman might succeed in Rylands v Fletcher because the type of damage suffered was reasonably
foreseeable in the event of the paint escaping from the man’s land.

222
Q

A passenger is on a passenger ship which receives a call from an oil rig which has caught fire. The fire was
started negligently. The ship goes to provide assistance, and the passenger, who remains on the deck
watching the events unfold, is terrified by the scenes they witness, and later suffers post traumatic stress
disorder. They are not physically hurt, and although the passenger felt that they were in danger, the ship was
never in danger, keeping a safe distance from the fire.
Which of the following statements is correct in relation to a claim for psychiatric harm in the tort of general
negligence?
(2 marks)
Select one alternative:
The passenger cannot claim for psychiatric harm. They are neither a primary nor secondary victim
because physical injury was not reasonably foreseeable.
The passenger can claim for psychiatric harm. They are a primary victim because they felt as though they
were in danger and the ship was near the fire.
The passenger can claim for psychiatric harm. They are a primary victim because they witnessed a
horrific event.
The passenger cannot claim for psychiatric harm. They are not a primary victim because they were never
objectively in physical danger, and they cannot claim as a secondary victim as they have no close ties of
love and affection with the workers from the oil rig.
The passenger can claim for psychiatric harm. They are a secondary victim because they witnessed a
horrific event.

A

The passenger cannot claim for psychiatric harm. They are not a primary victim because they were never
objectively in physical danger, and they cannot claim as a secondary victim as they have no close ties of
love and affection with the workers from the oil rig.

223
Q

A couple, living in a quiet, rural village, have been subjected to a considerable amount of noise, from the
neighbouring property, due to the neighbour testing the flying speed of his drones. One of the drones crash
lands on their outdoor furniture causing the glass table to smash. As a result, shards of glass scatter and cause
the couple to suffer cuts to their legs. The couple want to know whether they have a claim in private nuisance
for their losses. They own the house they live in.
Which statement constitutes the best advice in a claim for private nuisance?
(2 marks)
Select one alternative:
The couple should be advised to bring a claim for the noise interference. They will not be able to bring a
claim for the personal injury and property damage they have suffered.
The couple should be advised to bring a claim for their property damage. They will not be able to bring a
claim for the noise interference and personal injury.
The couple should be advised to bring a claim for the noise interference, the property damage and the
personal injury they have suffered.
The couple should be advised to bring a claim for the noise interference and the property damage they
have suffered. They will not be able to claim for their personal injury.
The couple should be advised to bring a claim for the noise interference and the personal injury they
have suffered. They will not be able to bring a claim for their property damage.

A

The couple should be advised to bring a claim for the noise interference and the property damage they
have suffered. They will not be able to claim for their personal injury.

224
Q

A trainee bus driver is having a driving lesson in her bus. She has a headache and feels slightly dizzy, but she
does not want to miss her lesson. One dizzy spell is so bad that the trainee driver temporarily blacks out and
swerves, causing the bus to mount the pavement and hit a blind pedestrian. The pedestrian suffers multiple
injuries as a result.
Which of the following statements best describes how the court would approach the question of standard of
care in relation to the trainee driver?
(2 marks)
Select one alternative:
The court would make no allowance for the fact that the trainee driver was not qualified nor for the fact
that she was unwell.
The court would make no allowance for the fact that the trainee driver was not qualified nor for the fact
that she was unwell but would take into account the fact the pedestrian was blind.
The court would make allowance for the fact that the trainee driver was not qualified, but would make no
allowance for the fact that she was unwell.
The court would make no allowance for the fact that the trainee driver was not qualified, but would make
allowance for the fact that she was unwell.
The court would make allowance for both the fact that the trainee driver was not qualified and for the fact
that she was unwell.

A

The court would make no allowance for the fact that the trainee driver was not qualified nor for the fact
that she was unwell.

225
Q

Due to a driver’s negligent driving, his passenger was seriously injured in a road traffic accident. When
emergency services attended the accident scene, the car was found to contain a large packet of cocaine which
the driver and passenger were planning to sell later.
Which one of the following statements best describes the position in relation to defences in the tort of
negligence?
(2 marks)
Select one alternative:
It is unlikely the driver can rely on the defence of illegality because there was not a direct link between
the passenger’s injury and the illegal act.
It is likely the driver can rely on the defence of illegality because both he and his passenger were
planning to sell cocaine. The passenger’s claim would be negated.
It is likely the driver can rely on the defence of illegality because both he and his passenger were
planning to sell cocaine. The passenger’s damages would be reduced.
It is unlikely the driver can rely on the defence of illegality because the crime committed by the passenger
was not sufficiently serious.
It is likely the driver can rely on the defence of illegality because the cocaine was in the car. The
passenger’s claim would be negated.

A

It is unlikely the driver can rely on the defence of illegality because there was not a direct link between
the passenger’s injury and the illegal act.

226
Q

A chain of restaurants has numerous branches. Each branch has an appointed manager who is responsible
for its day to day operations. A new employee begins working in one of the kitchens on a particularly busy day,
so the manager does not train him first. The manager has failed to train new employees on several occassions
previously. The new employee suffers burns when he lifts a boiling pan of soup from the hob.
Which of the following most accurately describes all of the available claims in tort?
(2 marks)
Select one alternative:
The new employee can bring a claim in employers’ primary liability against the manager and vicariously
against the owner of the restaurants.
The new employee can bring a claim against the manager in general negligence and against the owner
of the restaurants vicariously for the manager’s tort. The new employee can also bring a claim in
employers’ primary liability against the owner of the restaurants.
The new employee can bring a claim in employers’ primary liability against his employer, the owner of
the restaurants.
The new employee can bring a claim against the manager in general negligence and vicariously against
the owner of the restaurants. The new employee and manager can also bring a claim in employers’
primary liability against the owner of the restaurants.
The new employee can bring a claim against the manager in general negligence and vicariously against
the owner of the restaurants.

A

The new employee can bring a claim against the manager in general negligence and against the owner
of the restaurants vicariously for the manager’s tort. The new employee can also bring a claim in
employers’ primary liability against the owner of the restaurants.

227
Q

A delivery company is recruiting self-employed drivers to protect itself from vicarious liability should any
accidents occur. The contracts state that the drivers are (1) self-employed; (2) required to work a minimum of
four shifts per week; (3) able to work for other companies; and (4) able to send substitute drivers to carry out
their shifts. The drivers will be paid according to the hours they drive and will not be entitled to holiday or sick
pay.
Which of the following best describes the potential liability the delivery company may face if one of their
driver’s commits a tort whilst driving?
(2 marks)
Select one alternative:
The drivers will be independent contractors but the company will still be vicariously liable for the drivers’
torts.
The drivers will be employees as they seem fully integrated into the business of the company; therefore
the company could be held vicariously liable for the drivers’ torts.
The drivers will be employees as the company has the requisite control to be the drivers’ employer;
therefore the company could be held vicariously liable for the drivers’ torts.
The drivers will be employees as there is mutual obligation between the drivers and the company;
therefore the company could be vicariously liable for the drivers’ torts.
The drivers will be independent contractors and therefore the company will not be vicariously liable for
the drivers’ torts.

A

The drivers will be independent contractors and therefore the company will not be vicariously liable for
the drivers’ torts.

228
Q

A man leaves his car unlocked with the keys in the ignition whilst he runs into his local shop to buy some milk.
A teenager drives off in the car, failing to stop at a red light and hitting an eight-year-old girl as she crosses the
road. The girl is seriously injured. The man is not insured in relation to the vehicle.
Which of the following statements is most accurate regarding whether a duty of care was owed by the man to
the eight-year-old girl?
(2 marks)
Select one alternative:
The man might owe a duty of care to the eight-year-old girl as he created the dangerous situation.
The man might not owe a duty of care to the eight-year old girl as he did not know her.
The man might owe a duty of care to the eight-year-old girl if a reasonable driver would not have left their
keys in the ignition.
The man might not owe a duty of care to the eight-year-old girl as he was not insured.
It is very unlikely that the man owed the eight-year-old girl a duty of care; it was a third party that caused
the harm.

A

The man might owe a duty of care to the eight-year-old girl as he created the dangerous situation.

229
Q

A man has been drinking in the pub and is very drunk. One of his friends, who has not been drinking alcohol,
offers to give the man a lift home on her motorbike. She explains to the man that since the motorbike’s main
headlight is broken they will have to proceed slowly, using the bike’s sidelights. The man accepts the lift but
fails to fasten his helmet properly and he smokes an illegal marijuana cigarette on the way home. The rider
negligently misjudges a bend and crashes. The man’s left leg is broken in the crash.
Which of the following statements best describes the effectiveness of any defence/s the rider might seek to rely
upon?
(2 marks)
Select one alternative:
The defence of contributory negligence would not succeed as the man’s failures did not contribute to his
losses.
The defence of consent would succeed as the man had full knowledge of the danger of riding without
lights.
The defence of contributory negligence would not succeed as the man’s failures did not contribute to the
accident.
The defence of consent would not succeed as the man is too drunk to be capable of giving valid consent.
The defence of illegality would succeed as the man was committing a criminal act at the time of the
accident.

A

The defence of contributory negligence would not succeed as the man’s failures did not contribute to his
losses.

230
Q

A hot air balloon company takes a woman on a hot air balloon flight. Due to negligent flying by the company,
the balloon crash lands. The woman, who has osteoporosis (a condition that weakens the bones and makes
them more liable to break) suffers a broken pelvis, broken arms and broken legs. A passenger without
osteoporosis would have only suffered from broken legs in the crash.
Which of the following statements best describes the losses the woman can claim damages for against the hot
air balloon company in the tort of negligence?
(2 marks)
Select one alternative:
The woman cannot claim for the broken pelvis or broken arms because the hot air balloon company did
not know about the osteoporosis at the time of the accident.
The woman can only claim for her broken legs as a passenger without osteoporosis would have suffered
from this loss.
The woman can claim for all of her injuries, i.e. the broken pelvis, broken arms and broken legs, because
the hot air balloon company must take the woman as they find her.
The woman can claim for all her injuries because in negligence the defendant is liable for any personal
injury.
The woman cannot claim for the broken pelvis or broken arms because the osteoporosis was a preexisting condition.

A

The woman can claim for all of her injuries, i.e. the broken pelvis, broken arms and broken legs, because
the hot air balloon company must take the woman as they find her.

231
Q

A charity employs a member of staff to disinfect donated clothes using spray chemicals. The disinfecting takes
place in the basement where there are no windows; it is therefore poorly ventilated. The employee is provided
with a face mask, but not given any instructions on when and how to use it. The employee finds wearing the
face mask makes it difficult to breathe and decides not to use it. Shortly afterwards the employee develops a
lung disease as a consequence of the exposure to the disinfectant.
Which of the following best describes the charity’s liability for the employee’s disease?
(2 marks)
Select one alternative:
The charity will not be in breach of the employers’ primary duty of care because it provided the employee
with proper safety equipment. It was the employee who chose not to wear it.
The charity will not be in breach of the employers’ primary duty of care for failing to provide a safe place
of work due to the poor ventilation because it provided face masks.
The charity will be in breach of the employers’ primary duty of care for failing to provide a safe place of
work due to the poor ventilation and also for failing to provide a safe system of work by not encouraging
the use of the face mask.
The charity will be in breach of the employers’ primary duty of care as the employers’ duty of care is nondelegable and strict.
The charity will be in breach of the employers’ primary duty of care by not providing adequate ventilation
in the basement.

A

The charity will be in breach of the employers’ primary duty of care for failing to provide a safe place of
work due to the poor ventilation and also for failing to provide a safe system of work by not encouraging
the use of the face mask.

232
Q

A woman attends hospital with severe gastric pains and a fever. The Doctor suspects a ruptured appendix and
decides to do an immediate appendectomy (removal of the appendix). Medical journals over the past 6 months
have routinely emphasised the importance of monitoring patients before removing the appendix. Some doctors
state they would have carried out an immediate appendectomy although the majority would have monitored
the patient for 12 hours first. Following surgery, it becomes clear that the woman did not have a ruptured
appendix and she now has considerable post -operative pain.
Which of the following best explains the legal position in relation to the potential breach of duty by the doctor?
(2 marks)
Select one alternative:
The doctor has not breached his duty of care. The courts accept that it is perfectly valid to have conflicting
bodies of medical opinion and do not prefer one body of opinion over another.
The doctor has breached his duty of care. Even if the doctor has a body of doctors supporting his actions
and saying they would have acted in the same way, the courts can challenge a body of medical opinion if
it is illogical and indefensible.
The doctor has breached his duty of care because he misdiagnosed a patient’s condition.
The doctor has not breached his duty of care. A doctor is not expected to keep up to date with every
article written in every medical journal.
The doctor has not breached his duty of care. As long as another body of doctors would have carried out
an immediate appendectomy in this situation, there is no breach.

A

The doctor has breached his duty of care. Even if the doctor has a body of doctors supporting his actions
and saying they would have acted in the same way, the courts can challenge a body of medical opinion if
it is illogical and indefensible.

233
Q

A farmer keeps a large silo of grain on his farm, but the grain is leaking from a crack at the bottom of the silo. This has attracted large numbers of rats to the farm, which are then also going into the garden of a neighbouring householder and some have made a nest under the householder’s shed. While putting down rat poison, the householder is bitten by a rat and also develops Weil’s disease from the rats’ urine.

Which of the following provides the best advice as to any action the householder may bring in nuisance?

The householder cannot bring an action in any of the following: private nuisance, public nuisance or under the rule in Rylands v Fletcher

The householder can bring an action in private nuisance, public nuisance and under the rule in Rylands v Fletcher

The householder can bring an action in private nuisance and public nuisance, but not in Rylands v Fletcher

The householder can bring a claim in public nuisance, but not in private nuisance or under the rule in Rylands v Fletcher

The householder can bring an action in private nuisance and under the rule in Rylands v Fletcher, but not in public nuisance.

A

The householder cannot bring an action in any of the following: private nuisance, public nuisance or under the rule in Rylands v Fletcher

Correct: The damage caused is personal injury, which cannot be pursued in private nuisance or Rylands. Furthermore, the farmer has not brought the rats onto his land and accumulated them there. They have been attracted by the grain, which he has accumulated, but the grain has not escaped and caused damage. Although PI may be claimed in public nuisance, there is no action on this basis either as there is no evidence that a class of her majesty’s subjects have been materially affected, of which the householder is a member who has suffered special damage (PI).

234
Q

Two adjacent shops in a shopping centre are occupied by a butcher who smokes his own meat, and a high-end clothes designer. The clothes designer complains that smoke from the smoking process have led to complaints from customers and staff about the strong smell. The clothes designer wants to bring a claim for private nuisance.

In a claim for private nuisance what test will be applied to determine whether there has been an unreasonable interference?

Has the butcher acted maliciously?

Has the smoke caused physical injury?

Is the smoke unacceptable to the customers of the clothes designer?

Is the smoke unacceptable in the environment of a shopping centre?

Is the smoke more intense than it was a year ago?

A

Is the smoke unacceptable in the environment of a shopping centre?

This answer best reflects the law in relation to private nuisance. As per Sturges v Bridgman (1879) 11 Ch D 852, the court will consider the locality when looking at whether there was unreasonable interference. The other answers are other factors that might be taken into account, but the key test is whether the smoke is unacceptable in this environment.

235
Q

A farmer turns a field into a campsite for three months every summer, attracting hundreds of campers. The local villagers are upset by the increased traffic and rubbish caused by the campers and feel that their properties have lost value as a result.

The villagers have been advised that if they were to bring a claim as a class against the farmer in public nuisance their claim should be successful. The local authority or Attorney-General can being the claim on their behalf.

Which of the following best describes the remedies they could ask for?

The villagers could ask for both a full or partial injunction to stop or limit the camping and damages to compensate them for the value they have lost on their properties.

The villagers could only ask for a full or partial injunction to stop or limit the camping because those are the only remedies available.

The villagers could only ask for damages to compensate them for the value they have lost on their properties because camping is in the public interest.

The villagers could only ask for a full or partial injunction to stop or limit the camping because the loss of value on their properties was not reasonably foreseeable.

The villagers could ask for either a full or partial injunction to stop or limit the camping or damages to compensate them for the value they have lost on their properties.

A

The villagers could only ask for a full or partial injunction to stop or limit the camping because those are the only remedies available.

Correct. As the villagers would bring a claim in public nuisance as a class via their local authority or the Attorney General, a full or partial injunction would be the only remedy available.

236
Q

A young entrepreneur ran a business from the third floor of a building. An old woman owned the building and leased different parts to other business tenants. A third party person had blocked all the sinks in the lavatory on the fourth floor and turned on all the taps in order to cause a flood. This damaged the young entrepreneur’s stock.

Will the old woman be liable under Rylands v Fletcher?

The old woman will only be liable under Rylands v Fletcher if the young entrepreneur can establish that there was no natural use of the land.

The old woman will only be liable under Rylands v Fletcher if the young entrepreneur can establish that there was a natural use of the land.

The old woman will be liable under Rylands v Fletcher.

The old woman will not be liable under Rylands v Fletcher.

The old woman will only be liable under Rylands v Fletcher if the young entrepreneur can establish that there was a clear ‘escape’ of the flooding from the fourth floor property to his property.

A

The old woman will not be liable under Rylands v Fletcher.

Correct, the old woman will not be liable. The act which caused the damage was a wrongful act by a third party (Rickards v Lothian) and there was no non-natural use of land.

237
Q

A patient attends the Emergency Department of their local hospital with chest pain. The doctor briefly examines the patient and tells them to wait in the waiting room, where the patient suffers a heart attack and dies. Medical evidence proves that, if the doctor had treated the patient immediately, there was a 50% chance that the patient would have avoided the heart attack and survived.

Can the patient’s estate claim damages from the doctor for the patient’s death in the tort of negligence?

Select one alternative:

Yes, because the ‘material increase in risk’ test can be satisfied.

No, because a heart attack is a natural cause of death.

Yes, because the ‘material contribution’ test can be satisfied.

No, because the ‘but for’ test cannot be satisfied.

Yes, because the ‘but for’ test can be satisfied.

A

No, because the ‘but for’ test cannot be satisfied.

The correct answer is “No, because the ‘but for’ test cannot be satisfied”. But for the doctor’s breach (briefly examining the patient), the patient still would have died at that time and in that way on the balance of probabilities (Barnett v Chelsea and Kensington Hospital [1969] 1 QB 428). There was only a 50% chance that the patient would have lived if the doctor had treated the patient immediately i.e. there was a 50% chance that the patient would have died even if the doctor had not been negligent. In order to satisfy the ‘but for’ test, the patient’s estate would have to prove that there was a greater than 50% chance that the patient would not have died had it not been for the breach. The material contribution test does not apply here. This is not a case where medical science cannot establish the probability that ‘but for’ the breach the death would not have happened. Medical evidence confirms that there was a 50% chance of death even without the breach. Just because a heart attack can be a natural cause of death this does not mean that factual causation will fail. The question is whether the heart attack would have happened but for the breach on the balance of probabilities (Barnett). The material increase in risk test does not apply here as, currently, this is confined to industrial disease cases, namely mesothelioma and lung cancer caused by exposure to asbestos (Fairchild v Glenhaven Funeral Services Ltd & Others [2003] 1 AC 32).

238
Q

A company manufactures a state-of-the-art hairdryer which is distributed in England between January 2019 to January 2020, when concerns about the hairdryer overheating and causing fires brings production to a halt. In July 2019, a person buys the hairdryer from a shop (the ‘customer’). The hairdryer overheats and causes a small fire which results in £270 worth of damage to the customer’s carpet. The daughter of the customer was using the hairdryer at the time and suffers burns to her hands.

Which of the following statements best describes the customer’s and the daughter’s rights to redress under product liability law?

Select one alternative:

Only the customer has a viable claim for the damage caused by the faulty device as they purchased the product and were the legal owner at the time of the incident.

The customer has a claim under the Consumer Protection Act 1987. The customer’s daughter can claim under the Consumer Protection Act 1987 or common law.

Both the customer and their daughter have a claim under both the Consumer Protection Act 1987 and in common law.

The customer has a claim in common law only. The customer’s daughter can claim under the Consumer Protection Act 1987 or under common law.

Neither the customer nor their daughter has a claim in product liability as they have no contractual relationship with the company that produced the hairdryer.

A

The customer has a claim in common law only. The customer’s daughter can claim under the Consumer Protection Act 1987 or under common law.

The correct answer is “The customer has a claim in common law only. The customer’s daughter can claim under the Consumer Protection Act 1987 or under common law”. The customer has a claim in common law only as the only damage they have suffered is property damage to their carpet that is worth less than £275. Under s. 5(4) of the Consumer Protection Act 1987, no claim can be brought unless the property damage exceeds £275. The customer would therefore have to prove their claim in common law, that the company making the hairdryer fell below the standard of a reasonably competent manufacturer. The daughter can bring a claim under the Consumer Protection Act 1987 because anyone who suffers damage as a result of the defect can sue the manufacturer. They do not need to be the purchaser of the product to sue under the Act. The daughter has suffered personal injury caused by a defective product, so has the right to sue under the Act. She can also bring a claim in common law but would have to prove that the hairdryer manufacturer fell below the standard of care of a reasonable manufacturer. You do not have to have a contractual relationship with the manufacturer in order to sue them under the Consumer Protection Act 1987. Anyone who suffers damage as a result of the defect can sue. This is also why the option “Only the customer has a viable claim for the damage caused by the faulty device as they purchased the product and were the legal owner at the time of the incident” is incorrect.

239
Q

A homeowner lives in a residential area of London. Five hundred metres down the road is an oil factory and the homeowner is getting increasingly annoyed about being kept awake every night between around 1am to 3am when the oil trucks are moving in and out of the factory. In addition, the factory emissions produce a continuous offensive smell and an acidic residue that damages the paint on the homeowner’s car. The oil factory has planning permission to operate and has existed for 15 years. The homeowner has only lived at the property affected for 5 years.

Which of the following statements is the best advice to give the homeowner in the tort of private nuisance?

Select one alternative:

A claim in private nuisance is likely to succeed because the factory is a non-natural use of land in a residential location.

A claim in private nuisance is likely to succeed because of the oil company’s unreasonable use of their land. The noise from the trucks occurs very late at night and the emissions are continuous. The locality of the area is also important.

A claim in private nuisance is unlikely to succeed because the oil company has planning permission to operate in the area.

A claim in private nuisance is likely to succeed because of the oil company’s unreasonable use of their land. The factory is operating in a residential area and should move elsewhere.

A claim in private nuisance is unlikely to succeed because the homeowner moved to the nuisance. The factory had been operating for 10 years before the homeowner moved into their house.

A

A claim in private nuisance is likely to succeed because of the oil company’s unreasonable use of their land. The noise from the trucks occurs very late at night and the emissions are continuous. The locality of the area is also important.

The correct answer is “A claim in private nuisance is likely to succeed because of the oil company’s unreasonable use of their land. The noise from the trucks occurs very late at night and the emissions are continuous. The locality of the area is also important”. When assessing whether the factory’s use of land is unreasonable, the courts will consider time and duration; when the nuisance takes place, how long it continues and how frequently it is repeated (Kennaway v Thompson [1981] QB 88). The noise from the trucks occurs every night between 1am and 3am which is frequent and very late at night, and the offensive smell is continuous. In addition, the courts will consider the locality of the area. The factory is operating a commercial business in a residential area, so it is likely the court will consider this activity unreasonable despite the planning permission. ”A claim in private nuisance is likely to succeed because of the oil company’s unreasonable use of their land. The factory is operating in a residential area and should move elsewhere” is incorrect because it does not refer to time and duration. In addition, the court may not require the factory to move given it has planning permission but instead might issue partial injunctions restricting emissions and noise. Planning permission does not authorise a nuisance (Wheeler v JJ Saunders [1996] Ch 18). The defendant factory will not be able to argue that the homeowner moved to the nuisance. This is not a valid argument in private nuisance. Non-natural use of land is terminology relevant to a claim under the rule in Rylands v Fletcher, not private nuisance.

240
Q

A hotel guest explores their hotel. They want to look at the new swimming pool that is being built and enter at the main door of the swimming pool area despite reading a large sign on the door saying: “Do not enter. Staff only. This swimming pool is under construction and is very unsafe.” As the guest walks around the swimming pool, they slip on a large puddle of water, falling so hard that they break their leg and wrist. Their phone is smashed to pieces.

Which of the following statements is the most accurate advice for the guest in the tort of occupiers’ liability?

Select one alternative:

The guest is a visitor, and the hotel is unlikely to be in breach of any duty it owes the guest under the Occupiers’ Liability Act 1957. The warning is likely to be sufficient to discharge any duty owed by the hotel to the guest.

The guest is a trespasser, and the hotel is unlikely to be in breach of any duty it owes the guest under the Occupiers’ Liability Act 1984. The warning is likely to be sufficient to discharge any duty owed by the hotel to the guest.

The guest is a visitor, and the hotel is likely to be in breach of any duty it owes the guest under the Occupiers’ Liability Act 1957. The guest will be able to recover damages for their personal injuries and damaged phone.

The guest is a trespasser, and the hotel is likely to be in breach of any duty it owes the guest under the Occupiers’ Liability Act 1984. The guest will be able to recover damages for their personal injuries, but not for their damaged phone.

The guest is a trespasser, and the hotel is likely to be in breach of any duty it owes the guest under the Occupiers’ Liability Act 1984. The guest will be able to recover damages for their personal injuries and damaged phone.

A

The guest is a trespasser, and the hotel is unlikely to be in breach of any duty it owes the guest under the Occupiers’ Liability Act 1984. The warning is likely to be sufficient to discharge any duty owed by the hotel to the guest.

The correct answer is “The guest is a trespasser, and the hotel is unlikely to be in breach of any duty it owes the guest under the Occupiers’ Liability Act 1984. The warning is likely to be sufficient to discharge any duty owed by the hotel to the guest”. A trespasser is defined as a person who goes on to land without invitation, and whose presence is either unknown or practically objected to. Despite being a guest at the hotel, the sign on the door to the swimming pool makes it very clear to the guest that the swimming pool is not to be looked at or used. Under s.1(5) of the Occupiers Liability Act 1984 any duty owed by an occupier to a trespasser may be discharged if the occupier takes all reasonable steps “to give warning of the danger concerned or to discourage persons from incurring the risk.” The sign gives warning of the danger concerned – it is under construction and is very unsafe. Note that, if the trespasser’s claim were successful under the Occupiers’ Liability Act 1984, they would only be able to recover damages for their personal injuries, not their damaged phone.

241
Q

The director of a company, who was in charge of salaries, organised the company’s summer party on the company’s roof top terrace. The company paid for all food and drink at the party. Late at night at the party, some employees started questioning the director about the low salary increase they had received. The director got very angry and punched one of the employees. The employee fell and hit their head, causing traumatic brain damage. The director’s actions amount to the tort of battery.

Which of the following statements best explains whether the company might be vicariously liable for the brain damage caused to the injured employee?

Select one alternative:

The company could be vicariously liable. The director is an employee of the company who committed a tort (battery) during the course of their employment. There was a sufficient connection between their position as a director and their tort.

The company will not be vicariously liable. The director is an employee of the company, but they did not commit a tort (battery) during the course of employment. The tort occurred outside of working hours.

The company will not be vicariously liable. The director is an employee of the company, but they committed an intentional tort during the course of their employment, not negligence.

The company could be vicariously liable. The director is an employee of the company who committed a tort (battery) at the company’s summer party. The company is responsible for all the actions of its employees that occur on its premises.

The company could be vicariously liable. The director is an employee of the company who committed a tort (battery) at the company’s summer party. The company is responsible for all the actions of its employees at work events.

A

The company could be vicariously liable. The director is an employee of the company who committed a tort (battery) during the course of their employment. There was a sufficient connection between their position as a director and their tort.

The correct answer is “The company could be vicariously liable. The director is an employee of the company who committed a tort (battery) during the course of their employment. There was a sufficient connection between their position as a director and their tort”. This option covers the three-stage test for vicarious liability: (1) a tort (battery) has been committed (2) by an employee of the company and (3) during the course of employment. In relation to the third requirement, the close connection test is applied (Lister v Hesley all Ltd [2002] 1 AC 215). This is a two-stage test: (A) What functions or ‘fields of activities’ have been entrusted by the employer to the employee (what was the nature of their job)? (B) Was there sufficient connection between the position in which they were employed and their wrongful conduct to make it fair and just for the employer to be held liable? Here the director’s job was to take managerial decisions, particularly in relation to salaries and enforce their decisions. There was therefore a sufficient connection between the director’s position and their battery – they were exercising their authority over other employees who were challenging the director’s decisions regarding salary. The tort occurred at a work event on work premises. An employer is not responsible for all the actions of their employees at work events, only those that amount to a tort and that are deemed as being committed during the course of employment. Similarly an employer is not responsible for all the actions of their employees that occur on their premises, only those that amount to a tort and that are deemed as being committed during the course of employment. An employer can be vicariously liable for intentional torts committed by their employees, as well as negligence. The court might consider working hours when applying the close connection test, but it will not be determinative, especially where the event is a work event and held on the employer’s premises.

242
Q

A motorcyclist is involved in a road traffic accident caused by another driver’s negligent driving and they suffer a serious leg injury. Although the motorcyclist’s leg was fine immediately before the accident, medical evidence shows that the long-term effects of the leg injury were made substantially worse by a previous leg injury that the motorcyclist had suffered when rock-climbing a few years ago.

Can the negligent defendant driver argue that they only owe damages for the effects of the leg injury in a person who did not have a previous leg injury?

Select one alternative:

Yes. A leg injury is reasonably foreseeable as a result of driving negligently, but it is the motorcyclist’s fault for riding their motorbike when they had a pre-existing injury.

No. A leg injury is reasonably foreseeable as a result of driving negligently, and the defendant will be liable for the full extent of that harm, even if that harm has been aggravated by the motorcyclist’s own weakness.

Yes. A leg injury is reasonably foreseeable as a result of driving negligently, but it is not foreseeable that the claimant will have a pre-existing leg injury that could be made worse by an accident.

No. Personal injury is reasonably foreseeable as a result of driving negligently, and the defendant will be liable for the full extent of any personal injury caused.

No. A leg injury is reasonably foreseeable as a result of driving negligently, and the defendant need not foresee the exact way in which this harm occurs.

A

No. A leg injury is reasonably foreseeable as a result of driving negligently, and the defendant will be liable for the full extent of that harm, even if that harm has been aggravated by the motorcyclist’s own weakness.

The correct answer is “No. A leg injury is reasonably foreseeable as a result of driving negligently, and the defendant will be liable for the full extent of that harm, even if that harm has been aggravated by the motorcyclist’s own weakness”. The type of damage suffered here (leg injury) was reasonably foreseeable as a result of negligent driving. Once the type of damage suffered is reasonably foreseeable, the defendant is liable for the full extent of those damages (Vacwell Engineering v BDH Chemicals [1971] 1 QB 88), even if those damages have been aggravated by the claimant’s own weakness (Smith v Leech Brain [1962] 2 QB 405) – the ‘thin skull’ rule. So here the defendant would be liable for the full extent of the leg injury even though the extent of the injury has been aggravated by the motorcyclist’s pre-existing injury. Whilst it is true that the defendant need not foresee the exact way in which the harm occurs, this legal principle is not relevant to the facts of the question. The option referring to personal injury is true, but it is not as accurate as the option referring to the specific injury (the leg injury) and the ‘thin skull’ rule.

243
Q

Some nine-year-old children are playing a very common game played by children in a school playground. The playground supervisor tells the children to stop playing the game as they are worried one of the children will get hurt, even though the game is not actually banned at the school. The children do not listen, and a child accidently runs backwards into the supervisor, causing the supervisor to fall over and suffer a back injury. The supervisor sues the child in the tort of negligence for their back injury.

Which of the following statements best describes the standard of care expected of the child, and whether the child has fallen below this standard?

Select one alternative:

The child will be judged by the standard of care of a reasonably competent nine-year-old and is unlikely to have fallen below this standard. Children cannot be in breach of duty to adults.

The child will be judged by the standard of care of a reasonably competent nine-year-old and is likely to have fallen below this standard. A reasonable nine-year-old would have foreseen the risk of injury having been told to stop playing the game.

The child will be judged by the standard of care of a reasonably competent child and is unlikely to have fallen below this standard. It is normal for children to run backwards while not foreseeing injury resulting from their actions.

The child will be judged by the standard of care of a reasonably competent nine-year-old and is unlikely to have fallen below this standard. It is normal for nine-year-olds to run backwards while not foreseeing injury resulting from their actions.

The child will be judged by the standard of care of a reasonably competent person and is unlikely to have fallen below this standard. A reasonable person would not foresee injury from running backwards in a playground.

A

The child will be judged by the standard of care of a reasonably competent nine-year-old and is unlikely to have fallen below this standard. It is normal for nine-year-olds to run backwards while not foreseeing injury resulting from their actions.

The correct answer is “The child will be judged by the standard of care of a reasonably competent nine-year-old and is unlikely to have fallen below this standard. It is normal for nine-year-olds to run backwards while not foreseeing injury resulting from their actions”. Where the defendant is a child, the standard of care required will be that of the reasonable child of the defendant’s age (Mullin v Richards [1998] 1 All ER 920) so, in this case, that of a reasonable nine-year-old. It is highly unlikely the nine-year-old has fallen below this standard as a nine-year-old could not reasonably have foreseen any significant risk of the likelihood of injury (even though the children were told to stop playing, the game was not banned in the school). This explanation illustrates why the other options are incorrect. A child can be in breach of duty to adults if they have fallen below the standard of care of the reasonable child of the same age.

244
Q

A patient suffered from a painful benign tumour in their right foot. It was so painful, there were days they could not walk on their right foot at all. The patient’s doctor advised that they undergo a surgical procedure to remove the benign tumour. The patient agreed and had the surgery. Since the surgery, however, the patient has suffered increased pain, having developed chronic pain syndrome. This was a known 4% risk of the surgery performed by the doctor. The doctor did not warn the patient of this risk as it was minimal, even though other doctors would have done.

Which of the following statements best explains whether the doctor breached the duty of care owed to the patient?

Select one alternative:

The doctor is not in breach of duty because a 4% risk is not a material risk.

The doctor is in breach of duty because the doctor has failed to act in accordance with a practice accepted as proper by a responsible body of doctors.

The doctor is in breach of duty for failing to advise the patient of a material risk involved in recommended treatment.

The doctor is not in breach of duty because the patient would have had to have gone ahead with the surgery even if they had known about the 4% risk.
Answered and wrong

The doctor is in breach of duty for failing to advise the patient of all risks involved in recommended treatment.

A

The doctor is in breach of duty for failing to advise the patient of a material risk involved in recommended treatment.

The correct answer is “The doctor is in breach of duty for failing to advise the patient of a material risk involved in recommended treatment”. The case of Montgomery v Lanarkshire Health Board [2015] UKSC 11 confirmed that the correct legal test for determining whether a doctor is in breach of duty for failing to advise on risks, was the material risk test. Medical professionals are under a duty to take reasonable care to ensure that the patient is aware of any material risks involved in any recommended treatment, and of any reasonable alternative or variant treatments. A material risk is one which a reasonable person in the patient’s position would be likely to attach significance to, or the doctor is or should reasonably be aware that the particular patient would be likely to attach significance to. This is so the patient can give their informed consent. Here a 4% risk of increased chronic pain would be a risk that the reasonable person would attach significance to, when they are agreeing to surgery which is intended to alleviate pain. It is therefore a material risk which the doctor should have advised the patient about prior to the surgery. Note that the test is to advise on material risks, not all risks. The Bolam Test is not relevant to failure to advise on risk, so it is irrelevant that the doctor has not acted in accordance with a practice accepted as proper by a responsible body of doctors. There is no evidence that the patient would have had to go ahead with the surgery even if they had known about the risk.

245
Q

A homeowner asks their friend, who is a qualified architect, to draw up some plans for a loft conversion to their home. The homeowner is an accountant and has no skill or knowledge in relation to architectural plans, so makes it clear to the friend that they will be relying on their friend’s knowledge for the plans. The friend agrees to draw up the plans and insists that they will do it free of charge. The homeowner’s builders rely on the architectural plans drawn up by the friend and it soon becomes apparent that the plans relied upon were inaccurate and the loft is not suitable for use. This will cost the homeowner £50,000 to repair.

Which of the following statements best explains whether the homeowner is owed a duty of care by the architect for the £50,000 damage?

Select one alternative:

The homeowner is owed a duty of care because the homeowner reasonably relied on the architect’s plans.

The homeowner is owed a duty of care because the architect voluntarily assumed responsibility for the correctness of their plans. They did not include a disclaimer in their advice.

The homeowner is owed a duty of care because the homeowner reasonably relied on the architect’s plans and the architect voluntarily assumed responsibility for the correctness of their plans.

The homeowner is not owed a duty of care because they did not pay the architect for the plans.

The homeowner is not owed a duty of care because the architect was advising the homeowner on an informal basis.

A

The homeowner is owed a duty of care because the homeowner reasonably relied on the architect’s plans and the architect voluntarily assumed responsibility for the correctness of their plans.

The correct answer is “The homeowner is owed a duty of care because the homeowner reasonably relied on the architect’s plans and the architect voluntarily assumed responsibility for the correctness of their plans”. The homeowner has suffered pure economic loss (they need to repair a defective product – the loft). They might be owed a duty of care as this is pure economic loss caused by a negligent statement (the architect’s plans). Here it seems two of the tests from Hedley Byrne & Co Ltd v Heller & Partners Ltd [1964] AC 465 are satisfied, that of reasonable reliance and that of assumption of responsibility. Regarding reasonable reliance, the homeowner relied on the architect’s plans; it was reasonable for the homeowner to rely on the architect’s plans (the architect possessed special skill and knowledge which the homeowner did not have and, despite the advice being given for free, the architect held themselves out as giving considered advice that they knew was very important to the homeowner); and the architect knew that the homeowner was relying on the plans. In relation to assumption of responsibility, the facts are similar to Lejonvarn v Burgess [2017] EWCA 254 in which a professional landscape architect owed her friends a duty of care for advice she had given in relation to their garden, even though the advice was given free of charge. She possessed a special skill and had assumed responsibility for the work, on which the claimants had relied. In addition, looking at Caparo Industries plc v Dickman and others [1990] 2 AC 605 criteria, (1) the architect communicated the plans to the homeowner (2) the architect knew the purpose for which the homeowner would use the advice (3) the architect knew the claimant would rely on the plans without independent enquiry and (4) the homeowner acted upon the advice. This explanation illustrates why the other options are incorrect. The options that only refer to reasonable reliance or assumption of responsibility are incorrect, because are more accurate statement would refer to both tests.

246
Q

A cyclist is involved in a serious road traffic accident due to the negligence of a bus driver. The cyclist suffers brain damage as a result of the accident and can no longer work as an accountant.

Which of the following statements best describes what remedies the cyclist can claim from the bus driver in the tort of negligence?

Select one alternative:

The cyclist can claim special damages for lost earnings before trial. They can also claim general damages for the pain, suffering and loss of amenity for the brain damage.

The cyclist can claim special damages for lost earnings before trial. They can also claim general damages for (1) the pain, suffering and loss of amenity for the brain damage; and (2) lost earnings post-trial.

The cyclist can claim general damages for lost earnings before trial. They can also claim special damages for (1) the pain, suffering and loss of amenity for the brain damage; and (2) lost earnings post-trial.

The cyclist can claim special damages for any lost earnings pre- and post-trial. They can also claim general damages for the pain, suffering and loss of amenity for the brain damage.

The cyclist can claim special damages for any future loss of earnings. They can also claim general damages for the pain, suffering and loss of amenity for the brain damage.

A

The cyclist can claim special damages for lost earnings before trial. They can also claim general damages for (1) the pain, suffering and loss of amenity for the brain damage; and (2) lost earnings post-trial.

The correct answer is “The cyclist can claim special damages for lost earnings before trial. They can also claim general damages for (1) the pain, suffering and loss of amenity for the brain damage; and (2) lost earnings post-trial.” Special damages cover specifically provable and quantifiable financial losses at the time of trial. This would include the cyclist’s loss of earnings incurred up to the date of trial. General damages cover future financial losses, which cannot be specifically proven. This would include any future loss of earnings that the cyclist might suffer after the trial. General damages also includes a sum of money for the pain, suffering and loss of amenity caused by the defendant’s negligence.

247
Q

A company owns a warehouse where it stores several hundred mattresses, before shipping them to various shops around the country to be sold. A fire starts accidently at the warehouse and, given the flammability of the mattresses, the fire quickly spreads onto the neighbouring property causing substantial damage.

Which of the following statements is correct, in relation to a claim by the neighbour against the company under the rule in Rylands v Fletcher?

Select one alternative:

This claim will not succeed because the fire was an act of God.

This claim will succeed because the storage of several hundred mattresses is a non-natural use of land.

This claim will not succeed because there was no escape.

This claim will not succeed because the fire was not reasonably foreseeable.

This claim will succeed because the fire escaped from the company’s property to the neighbouring property, causing damage.

A

This claim will not succeed because there was no escape.

The correct answer is “This claim will not succeed because there was no escape”. To succeed in a claim under Rylands v Fletcher, the claimant must prove that the ‘thing’ (here, the mattresses) that the defendant brought onto their land escaped from the defendant’s land to the claimant’s land. This has not happened here – it is the fire that escapes, not the mattresses. The defendant did not bring the fire onto its property. The fact pattern in the question is very similar to the case of Stannard v Gore [2012] EWCA Civ 1248 where tyres spread the fire and the claim failed as it was not the tyres that escaped. It is not the fire that needed to be foreseeable for a claim to succeed, but the damage in the event of an escape of the mattresses. It is unlikely that storage of mattresses will be seen as a non-natural use of land – it is not a use which poses an increased risk over and above that of a normal use. There is no escape, so no liability under Rylands v Fletcher, therefore there is no need to consider defences (act of God).

248
Q

A local council owns a park with an artificial rock-climbing wall on site. A teenager visits the park, entering via a carpark and walking half a kilometre to the rock-climbing wall. There is a sign in the car park that reads ‘Take care when rock-climbing. The council will not be liable for any injuries or loss when rock-climbing howsoever caused.’ About 5 metres up the rock-climbing wall, one of the climbing holds (the ‘rocks’ that support a climber’s feet and hands) is loose. As the teenager grabs the loose climbing hold, it comes away from the wall and the teenager falls to the ground, breaking both legs.

Which of the following statements is most accurate in relation to a claim brought by the teenager against the council under the Occupiers’ Liability Act 1957?

Select one alternative:

It is highly unlikely the teenager’s claim will succeed against the council. The council has excluded liability for any injuries or loss caused by rock-climbing.

It is highly likely the teenager will succeed in a claim against the council. It was the council’s responsibility to guarantee the safety of the rock-climbing wall.

It is highly unlikely the teenager’s claim will succeed against the council. The teenager consented to the risk of injury by choosing to use the rock-climbing wall (an obvious danger).

It is highly unlikely the teenager’s claim will succeed against the council. The council has warned visitors to take care when rock climbing.

It is highly likely the teenager will succeed in a claim against the council. The warning was insufficient to discharge the duty owed by the council

A

It is highly likely the teenager will succeed in a claim against the council. The warning was insufficient to discharge the duty owed by the council

The correct answer is “It is highly likely the teenager will succeed in a claim against the council. The warning was insufficient to discharge the duty owed by the council”. Under s.2(1) of the Occupiers’ Liability Act 1957 (the ‘OLA 57’) the council (the occupier) owes the teenager (the visitor) a duty of care. Section 2(2) of the OLA 57 states that this is a duty to take reasonable care to see that the visitor is reasonably safe in using the premises (here, the rock-climbing wall which is part of the park). An occupier must therefore do all that they reasonably can to ensure their visitor is safe: it does not mean they have to guarantee the safety of the visitor. Where there is adequate warning of any danger, the occupier may have discharged this duty. To do this, the warning should tell the visitor what the danger is, where it is and how to avoid it (Roles v Nathan [1963] 1 WLR 1117). The council’s warning does not do this – it does not inform the teenager what the dangers of rock-climbing could be and how to avoid those dangers. The teenager has not consented to the risk of injury by using the rock-climbing wall – as a visitor they are entitled to expect that the rock-climbing wall will be reasonably safe. Under s.65(1) of the Consumer Rights Act 2015, the council cannot exclude or restrict their liability for personal injury resulting from a breach of the duty under the OLA 57.

249
Q

After consuming large quantities of alcohol, two friends drove home together from the pub (one was the driver, and one was the passenger). The passenger encouraged the driver to drive at 90mph in a 30mph zone, and to swerve across the road to scare other drivers driving in the opposite direction. The driver collided with another car and was killed. The passenger suffered from brain damage.

Which of the following defences (if any) are most likely to succeed if the passenger brings a negligence claim against the estate of the driver for their brain damage?

Select one alternative:

Illegality and Consent

Consent

Illegality and Contributory Negligence

Illegality

Contributory Negligence

A

Illegality

The correct answer is “Illegality”. The fact pattern in the question very much mirrors the facts from Pitts v Hunt [1991] 1 QB 24 in which illegality was successfully argued. Illegality is a complete defence. It is also likely that the test from Patel v Mirza [2016] UKSC 42 would be satisfied: the offence of dangerous driving aims to protect the public and denying the claim would enhance this purpose by deterring others from doing the same; and denying the claim would be a proportionate response (the offence is very serious and was intentional; the parties were equally culpable; and there was a causal link between the illegality and the negligent driving). The estate of the driver would be unable to rely on consent because s.149 of the Road Traffic Act 1988 prevents the use of consent by motorists facing claims from passengers. The estate of the driver could argue contributory negligence, but illegality is likely to succeed, and this is a complete defence.

250
Q

Paramedics were called to a football stadium which was on fire due to the negligence of the owners of the stadium. A vehicle parked at the stadium caught fire and exploded causing injury to some spectators of the football game and one of the paramedics. A programme seller at the match witnessed the explosion, but they were over 500 metres away from the vehicle that exploded (outside the blast radius) and so suffered no physical injury. Following the incident, however, the programme seller developed traumatic neurosis as a result of what they witnessed. The programme seller was in a new relationship with the injured paramedic.

Will the owners of the stadium owe the programme seller a duty of care for their traumatic neurosis?

Select one alternative:

It is highly unlikely a duty of care will be owed. The programme seller is a primary victim, but they do not have proximity with the explosion.

It is highly likely a duty of care will be owed. The programme seller is a secondary victim and in a relationship of close ties of love and affection with the actual victim.

It is highly likely a duty of care will be owed. The programme seller is a secondary victim and psychiatric harm was reasonably foreseeable.

It is highly unlikely a duty of care will be owed. The programme seller is a secondary victim and not in a relationship where there is a presumption of close ties of love and affection with the actual victim.

It is highly likely a duty of care will be owed. The programme seller is a primary victim and personal injury was reasonably foreseeable.

A

It is highly unlikely a duty of care will be owed. The programme seller is a secondary victim and not in a relationship where there is a presumption of close ties of love and affection with the actual victim.

The correct answer is “It is very unlikely a duty of care will be owed. The programme seller is a secondary victim and not in a relationship where there is a presumption of close ties of love and affection with the actual victim.” The programme seller is a secondary victim because they have suffered psychiatric harm due to fear for someone else’s safety. They were not in fear of their own safety, which is why they are not a primary victim. In order to prove that a duty of care is owed to a secondary victim, they must satisfy all of the Alcock criteria (proving psychiatric harm was foreseeable is not enough). These criteria include proximity of relationship between the claimant and actual victim (here, the programme seller suffering neurosis and the injured paramedic). In order for there to be the relevant proximity, there must be close ties of love and affection between the claimant and actual victim. These close ties must be proved by the claimant, although this can be rebuttably presumed in the case of parents, children, spouses and engaged couples. Every individual case turns on its facts but to date no claimant outside of the presumed relationships of close ties of love and affection has been successful.

251
Q

An orchestra company (the ‘Company’) employs a violinist. The violinist feels that the noise levels at rehearsals are much higher than any they have been used to, so brings a sound level meter to a rehearsal which shows a decibel reading well above the levels permitted by noise regulations. Following rehearsals, the violinist suffers from tinnitus (a ringing noise in their ears). The Company had told their employee conductor to ensure ear protection was worn by all musicians. However, whilst the conductor handed out ear protection to all musicians (including the violinist), the conductor also said repeatedly that the ear protection was a waste of time and a better performance could be achieved without the protection. None of the musicians wore the ear protection.

Which of the following statements is most accurate in relation to an employer’s primary liability claim brought by the violinist against the Company?

Select one alternative:

The Company has breached its duty of care owed to the violinist because it failed to provide competent fellow employees.

The Company has breached its duty of care owed to the violinist because it failed to provide the relevant safety equipment and failed to provide competent fellow employees.

The Company has not breached its duty of care owed to the violinist because the violinist chose not to wear the ear protection provided by the Company.

The Company has breached its duty of care owed to the violinist because it failed to provide a safe system of work and failed to provide competent fellow employees.

The Company has not breached its duty of care owed to the violinist because the fault lies with the conductor rather than the Company.

A

The Company has breached its duty of care owed to the violinist because it failed to provide a safe system of work and failed to provide competent fellow employees.

The correct answer is “The Company has breached its duty of care owed to the violinist because it failed to provide a safe system of work and failed to provide competent fellow employees”. It is not enough for an employer to simply provide the safety equipment (here, the ear protection), they are also under a duty to encourage or insist that it is worn (i.e. to implement a safe system of work). This duty is personal and non-delegable meaning the employer can delegate performance of the duty, but not liability for its breach, so it is irrelevant that it was the conductor who said the safety equipment was a waste of time. Employers are directly liable if those they have entrusted with responsibility fail to exercise reasonable care in respect of employee safety. The Company has also failed to provide competent fellow employees (the conductor) if they knew, or ought to know, that the conductor was telling musicians not to wear the ear protection. The Company is not in breach for failing to provide safety equipment because it did in fact provide the relevant safety equipment (it just failed to ensure that it was worn).

252
Q

An RAF pilot got so drunk one evening at the RAF base at which they were based that they fell asleep in a public area. A fellow RAF pilot saw the drunk pilot sleeping and took the drunk pilot to their bed. They did not stay with the drunk pilot, and during the night, the drunk pilot was sick and choked to death on their own vomit.

Did the fellow RAF pilot owe the drunk pilot a duty of care to keep them reasonably safe?

Select one alternative:

Yes. The fellow RAF pilot had assumed responsibility, through their conduct, for the welfare of the drunk pilot.

Yes. The fellow RAF pilot had a sufficient degree of control over the drunk pilot to justify the imposition of a duty of care.

Yes. The fellow RAF pilot had a contractual duty to look after the drunk pilot.

No. The fellow RAF pilot had not explicitly assumed responsibility for the welfare of the drunk pilot.

No. The drunk pilot was entirely responsible for their own actions and the fellow RAF pilot was not at fault.

A

Yes. The fellow RAF pilot had assumed responsibility, through their conduct, for the welfare of the drunk pilot.

The correct answer is “Yes. The fellow RAF pilot had assumed responsibility, through their conduct, for the welfare of the drunk pilot.” Although the general rule is that no duty of care is owed for a failure to act (so here, a failure to stay with the drunk pilot), there are limited exceptions to this rule, including where the defendant assumes responsibility for the claimant’s welfare. By taking the drunk pilot to their bed (taking action to care for the drunk pilot), the fellow pilot has arguably assumed responsibility for the drunk pilot’s welfare. Assumption of responsibility can be through conduct alone; it does not have to be explicit. The fellow RAF pilot did not owe the drunk pilot a contractual duty.

253
Q

Negligence sequence

A

C to prove:
Loss or damage
Duty
Breach
Causation
Remoteness

D to prove:
Defences

254
Q

Determining whether a duty of care is owed

A

is there a precedent making clear whether or not a duty is owed?

yes: apply precedent

no: analogy

255
Q

is there a precedent making clear whether or not a duty is owed? NO

A

consider whether duty should be imposed by analogy with existing cases

there will be no duty if harm was not reasonably foreseeable

consider proximity between C and D

consider what is fair, just and reasonable

duty to only develop the law incrementally

in practice, huge precedents!

256
Q

there is no duty owed in relation to omissions except

A

where there is a statutory duty

where there is a contractual duty

where D has sufficient control

where D assumes responsibility

where D creates the risk

where the omission is a failure to prevent a third party from causing harm and
- there is sufficient proximity between C and D
- there is sufficient proximity between D and third party
- D creates the danger
- the risk is on D’s premises

257
Q

standard of care x3

A

the lower standard
- children
- illness/disability

the ‘reasonable man’ test

the professional standard

258
Q

the reasonable man test

A

objective nature of the test
act, not the actor

lower and professional standard also

259
Q

factors relevant to breach

A

usual or common practice
likelihood of harm
magnitude of ahrm
practicality of precautions
benefit of D’s conduct
(sport
state of the art defence
breach / errors of judgement)

260
Q

breach equation

A

likelihood of harm x magnitude of harm

balanced against

cost of precaution + benefit of D’s conduct

261
Q

a doctor is not guilty of negligence is he

A

has acted in accordance with a practice accepted as proper by a responsible body of medical men skilled in that particular art… a doctor is not negligent, if he is acting in accordance with such a practice, merely because there is a body of medical opinion which takes the contrary view (Bolam)

two limitations

1 – court satisfied that they body of opinion relied upon can demonstrate that such opinion has a logical basis (Bolitho)

2 – when it comes to a doctor advising in relation to risks, a doctor is under obligation to take reasonable care to ‘ensure that the patient is aware of any material risk involved in any recommended treatment, and or any reasonable alternative or variant treatment’

262
Q

causation

A

factual causation AND legal causation

factual causation deals with establishing the link between the breach and the damage whereas legal causation involves considering whether there are any grounds upon which the link should be regarded as having been broken

263
Q

factual causation

A

BUT FOR the negligent act/omission, would C have suffered the loss?

No: factual causation is established

Uncertain: is the case one of CUMULATIVE CAUSES?

Yes: how likely is it that the injury would have occurred as a result of the non-tortious cause in any event?
- <50% = factual causation is established
- >50% = factual causation is not established
- no possible to say: was the breach a ‘more than negligible cause of the injury
– yes: factual causation established
–no: not established

No: did the tortious act/omission MATERIALLY INCREASE THE RISK of the injury
- yes: consider whether the fairchild/McGhee or loss of chance exceptions should apply
- no: factual causation is not established

264
Q

NAI

A

three types

acts of god: exceptional natural event

acts of third parties: high unforeseeable (but if medical: so gross and egregious test!)

acts of the claimant: high unreasonable and if occur AFTER the breach (If employer is negligent and employee breaks ankle but then employee does something so unreasonable then employee breaks leg then can can argue employer only liable for ankle not leg)

265
Q

But for test might not be satisfied where

A

there are multiple causes (eg 3 potential causes so cannot prove greater than 50% change that breach caused the loss)

Distinguish with causes that act independently and/or there is more than one cause!

Eg case law about premature baby that went blind
5 possible causes that were all equally probable
5 causes did not act together and only one cause was negligent

266
Q

material contribution test

A

multiple causes + causes act cumulatively

Esp in clinical negligence

Bonnignton: claimant exposed to negligent and non-negligent dust and both dust contributed to diseases so but for negligent dust would C suffered loss?

Experts could not say! Rather than let claim fail because no scientific answer, Court: negligent dust materially contributed and D liable for all the loss

Material contribution: more than negligible contribution

267
Q

material increase the risk test

A

Multiple causes but not cumulative (one causal agent!)

Single exposure to dust caused dermatitis but which dust caused it?

Liability if breach materially increased risk eg greater exposure = greater chance

Cases limited to industrial disease cases

268
Q

Not too remote even if

A

The precise way the damage occurred was not reasonably foreseeable

The full extent of the damage was not reasonably foreseeable

Thin skill rule! Not just physical eg also financial + beliefs

269
Q

remoteness x2

A

Was the damage reasonably foreseeable?
- yes: not too remote

If no, was the damage of the SAME TYPE reasonably foreseeable? Narrow or broad construction
- yes: not too remote
- no: too remote (C will not recover re that damage)

270
Q

defences

A

for D to prove on balance of probabilities + x3

illegality

consent

contributory negligence

271
Q

illegality

A

Two steps
1) Has claimant has committed illegal or grossly immoral act?

2) Apply Patel v Mizra test x3
- Underlying purpose of prohibition, is purpose enhanced by denying the claim
- Relevant public policy on which denial of the claim may have an impact
- Denial of the claim is proportionate response to illegality (punishment is for criminal courts!)
–Seriousness of conduct
–Centrality to the tort: ie is there a causal link between illegality and the tort
–Whether it was intentional
–Disparity in parties respective culpability

Look into precedents!!

272
Q

consent

A

Consent: complete defence (x4)
Capacity
Full knowledge of nature and extent of the risk; subjective
Agreed (express — signed or implicitly — engage in dangerous activity) to risk of injury
Agreed voluntarily

273
Q

Contributory negligence

A

Contributory negligence: partial defence (x2)

1– C failed to take reasonable care for their safety
— special considerations for emergencies/difficult situations; age; rescuers, nature of duty

2 – And failure contributed to claimants damage
—contributing to accident which led to damage or contributing to damage

If established, C damaged reduced by a %

274
Q

Remedies

A

Put C in position they would have been in had tort not occurred: backwards looking

275
Q

Compensatory damages

A

Compensatory damages x2 (special v general)

Special damages: specifically provable and quantifiable (eg past expenses for treatment until date of trial/ earnings loss until date of trial )

General damages: future financial losses which cannot be specifically proven (medical expenses that have not been incurred after trial/ future loss of earnings)
Also include compensation for physical injury eg pain and suffering caused by broken arm
“Pain, suffering and loss of amenity” case law!

Deductions may be appropriate: sick pay, contributory negligence, state benefits

276
Q

Victims death

A

Law reform (miscellaneous provisions) act 1934
Deceased estate bring claim on behalf on deceased before their death
Bring claim that deceased would have brought

Fatal accidents act 1976
Allows claim by certain dependents of the deceased for their own losses caused by losing someone that they depended upon

277
Q

Employer’s liability vs vicarious liability

A

Employers liability
Principles of negligence where employee claims against employer
claimant must always be an employee of the defendant
Negligence in the work place: employee suing employer for breach of duty they owed to employee

Vicarious liability
Not a tort! But a MECHANISM that can be used by claimant to sue an employer for a tort that has been committed by one of the employer’s employees
Employer is not at fault, rather their employee is at fault

278
Q

Employer’s liability

A

Duty x4
Safe/competent employees
Safe/proper plant and equipment
Safe place of work/premises
Safe systems of work

Singly duty: take reasonable precaution to ensure an employee’s safety

Duty is personal and non delegable: regardless of who employer uses to carry out task; the ultimate responsibility is with employer

Negligence in work place so to establish employer’s primary liability need to go through sequence!

Have to ensure that claimant is an employee of defendant!

279
Q

Vicarious liability

A

Vicarious liability x3

Tort has been committed by party A

Party A is employee of party B (or akin)
–Economic reality test! Ready mix concrete
–Paid in exchange of service/mutuality of obligations
–Party B had control over party A eg tasks
–All contractual factors are consistent with employment relationship
–If not employee or independent contractor then akin to employment test? x5
–Party B lent party A to another party: who is the employer?
—-General rule: original employer remains liable
—-Unless other employer had the most control + can also find dual liability!

Tort committed during course of employment
–Close connection test x2
—-Nature of employees activities/job
—-Sufficient connection between position of employee and wrongful conduct to make it just for employer to be liable

280
Q

Psychiatric harm

A

Type of loss in negligent claim — Not a tort!

281
Q

Psychiatric harm – approach

A

1) identify recognised psychiatric harm (fear, mental grief, distress does not count)

2) is a duty of care owed?

282
Q

Psychiatric harm – types of victims

A

Types of victim: actual, primary, secondary victims

Actual: physical and possibly psychiatric harm

Primary: suffer psychiatric harm as a result of reasonable fear for physical fear (objective)

Secondary: no physical but suffer psychiatric due to fear for someone else (not in danger zone)

Rescuers do not have special status! Only be primary victims if in reasonable fear of their safety

Actual and primary victims are smaller in numbers than secondary victims — fear about size of secondary means court takes a restrictive approach

283
Q

Psychiatric harm – method

A

Actual victim: apply usual sequence for negligence

Primary victim: starting point — establish that physical harm was reasonably foreseeable then sequence

Secondary victim: Alcock criteria x4-5
1 – Was psychiatric harm reasonably foreseeable? (Not enough that physical harm would be foreseeable)

2 – Is there proximity of relationships between the claimant and ‘victim’?
Presumed in certain relationships eg parent/child but can be rebutted
Not presumed (eg siblings) can in theory be rebutted but has not been successful

3 – Is there proximity in time and space?

4 – Was the psychiatric harm shock-induced?
As opposed to gradual realisation

5 – Is it fair, just and reasonable to impose a duty of care?
Eg policy considerations like floodgates

Short element in adapt: beyond primary and secondary victims considers psychiatric harm in assumption of responsibility cases

284
Q

Pure economic loss

A

Type of loss! Not a tort! Affects duty of care but still need to consider full sequence!!

285
Q

Pure economic loss
Definition x3 through examples:

A

Economic loss not flowing from damage to person or property (eg bad investment, missed contractual opportunity, loss of inheritance)

Loss arising from damage to property of another (eg auction could not run because of a cow disease)

Defective items: product that was always defective (property has not been damaged, rather it is not worth what you thought it was)

286
Q

economic loss
general rule + exceptions

A

General rule: no duty of care is owed in respect of pure economic loss (for acts!!)

Exceptions x3
1 – Wills: beneficiaries owed duty of care from solicitors (eg negligent drafting = loss of inheritance)
2 – References: person writing owes subject of reference an accurate reference
3 – Negligent misstatements (no duty of care owed for negligent acts casing PEL)

287
Q

economic loss tests

A

Where there is no precedent, case law! Hedley Byrne v Heller: 3 tests

1– Reasonably reliance x3
Claimant must have relied on D advice (factual issue)
It was reasonable for C to rely on D advice (legal issue: eg disparity of knowledge/context)
D knew or ought to have known that C was relying on advice

2– Voluntary assumption of responsibility: but for the absence of consideration, there would be a contract

3– Special relationship of trust and confidence
The parties seeking advice trusted the other to exercise a degree of care and ti was reasonable and they knew or ought to have known it would be relied upon

288
Q

PEL – Where a claim is made by a third party who relies on advice prepared for a different person/purpose

A

Rules of Hedley Byrne Heller + x4

D must communicate to third party (identifiable) or know that it would be

D must know the purpose

D must know that 3rd party would rely without independent enquiry

C acted upon advice to detriment (reasonable to expect D to protect C form that loss)

289
Q

PEL –Disclaimers

A

Disclaimers
Subject to unfair contact terms act 1977 (B2B) or CRA 2015
Only if reasonable or fair!

290
Q

land-based torts

A

completely different to negligence so no sequence

291
Q

Private nuisance
Who can sue?

A

Claimant must have a legal interest in land affected

292
Q

Private nuisance
Who can be sued?

A

x3

1) creator of nuisance

2) occupier of land where nuisance originates
For nuisance created themselves
For nuisance created by others eg contractors if reasonably foreseeable consequence of task
If created by trespasser then occupier can be sued if they continue or adopt nuisance

3) owner of land
Where land occupied by tenant, landlord not usually liable unless x2
Created nuisance
Or authorised: participating in it or leasing with high likelihood of nuisance

293
Q

Private nuisance
claim

A

x4
1) indirect interference
Eg smells, sounds, fumes, vibrations
Contrast with physical invasion (ie direct interference = trespass)

2) recognised damage (no PI or PEL)
Must be reasonably foreseeable
Physical damage to property or
sensible personal discomfort (SPD, S = relating to senses) or
SPD needs to materially interfere with ordinary human comfort
Consequential economic loss (not pure economic loss!)

3) continuous act
Ongoing, minor exceptions (adapt!)

4) unlawful interference (unlawful = unreasonable: balancing act between parties! And D cannot argue that C moved to the nuisance)
Time and duration
Locality (use of land = physical damage, court will not consider locality + likely unlawful)
Abnormal sensitivity
Malice
Lack of care
Excessive behaviour
Public benefit

294
Q

Private nuisance
Defences

A

Prescription
Statutory authority
Consent
Act of third party
Act of God
Contributory negligence

295
Q

private nuisance
remedies

A

Remedies (x3) — see adapt

Damages (rarely)

Injunction

Abatement — self help remedy, no need for court order but must give notice unless emergency

296
Q

Public nuisance
Who can sue?

A

Individual — if they have suffered special damage (over and above the class, or different loss and does not need to have an interest in the land!)

Local authority (if it has suffered in its own behalf or on the behalf of inhabitants)

Attorney general

297
Q

Public nuisance
Who can be sued?

A

Creating or responsible for nuisance
Losses to be recovered is wide: PI, property, CEL, PEL and inconvenience (ie SPD)

298
Q

Public nuisance
Claim

A

Claim x4
Act or omission

One-off event (no need for continuous act)

Class of his majesty’s subjects (no exact n, depends on facts, significant % of community)

Materially affects comfort and convenience (harm being caused to reasonable comfort and convenience to class: CEL, PEL, PI, property, SPD but must reasonably foreseeable)

299
Q

Public nuisance
Remedies

A

Remedies x2 (no abatement)

Damages (rarely)
Injunction

300
Q

Public nuisance
Defences

A

Defences x5 (no prescription)

Statutory authority
Consent
Act of third party
Act of God
Contributory negligence

301
Q

Rylands v fletcher

A

Rule: protects against interference from defendant’s land due to an isolated escape

Who can be sued? Who can sue? Same as private

BUT only for loss of property damage and CEL

Claim x5
1) C BRINGS onto land + accumulates (not naturally occurring)
2) for OWN PURPOSES + something likely to do mischief (high threshold)
3) must be an actual ESCAPE (where D has control to where D no longer has control of land)
4) causes FORESEEABLE HARM (no need for escape to be foreseeable harm but rather the harm is)
5) NON-NATURAL use of land (D use had to be extraordinary and unusual re to standards of day)

Defences: same as private but
No prescription
and +2:
common benefit + act or default of claimant

Remedies same as private nuisance (but damages most common since types of loss property/CEL)

302
Q

Trespass to land
Who can sue?

A

Anyone with a legal interest in land affected (own it or be in possession of it)
Demised/rented land: claimant is tenant or licensee in possession rather than land owner

303
Q

Trespass to land is actionable per se + notes

A

it does not require any actual damage!!

Note:
Trespass includes airspace! But statute so that aircraft reasonably above is not trespass
Below ground too! Unless absurd

304
Q

trespass to land

A

Claim x2

1– Direct and physical interference
Entering C’s lands
Remaining on C land when permission has been revoked
Doing something that is not permitted on C’s land
Placing objects on C’s land without permission

2– Interference was intentional
Must intend the direct action that results in the trespass but need not intend to trespass
Eg voluntarily hit tennis ball and accidentally goes on neighbour’s land is trespass as you intend to hit ball
Intention can be implied (case law)

305
Q

trespass to land
defences

A

x3

Permission/consent

Legal authority: eg policy

Necessity: to protect public or private interest

306
Q

Trespass to land
remedies

A

Remedies x5

Damages (most common)

Injunction

Re-entry (self help but only reasonable force)

Recovery of land (court order for D to be removed)

Mesne profit (D have made a profit/saved expenditure while wrongfully occupying land)

307
Q

Occupiers liability

A

Extension of negligence but based in statute

Loss caused by state of premises

Statute x 2: occupier’s liability act 1957 (visitors) AND occupiers liability 1984 (non visitors)

visitors
1 – occupier + premises + visitor = duty
2 – standard of care
3 – fall below standard?
warnings
causation/remoteness: assume unless
defence: consent + CI

non-visitors
1 – occupier + premises + visitor MIGHT = duty only for PI (not property)
2 – duty owed x3
3 – standard of care
warnings easier than visitors
causation/remoteness assumed
defence: consent + CI

308
Q

Occupiers liability – visitors

A

1) Occupier + premises + visitor = duty owed in relation to personal injury and property damage
–Occupier
sufficient degree of control over premises to justify imposition of duty (wheat v lacon)
Can be more than one occupier: eg independent contractor and anyone living there at the time
–Premises
Broad definition!
Any fixed of moveable structure, including any vessel, vehicle or aircraft
–Visitor (by common law so defined by case law)
Express permission (can be limited! On the basis of certain conditions: area, time or purpose)
Implied permission: eg postman
Doctrine of allurement (not as relevant): children visitors as lured easily
Lawful authority: from statute eg medical professionals
Contractual permission

2) standard of care (s2(2) OLA 1957): D needs to do all that is reasonable so that visitor is reasonably safe
–Visitor (not premises) that must be safe
–Objective test: does not matter if D thought they did well
–Context: reasonable care depends on visitor concerned! Eg awareness of vulnerability!!
–Higher standard of care for children! s2(3)(a)
–Lower standard of care for occupiers that are skilled visitors s2(3)(b) eg electrician

3) below standard of care (same test as general negligence)
Likelihood of harm
Magnitude of harm
Practicality of precautions….

Warnings: effective if enables visitor to reasonably safe
Where danger is, what danger is and how to avoid it

Independent contractors
If D acted reasonably in entrusting work to the independent contractor x3: hiring, selecting, supervising

Causation and remoteness: assume caution and remoteness are made out unless seems otherwise

Defences x2
Consent
Contributory negligence

309
Q

Occupiers liability – non-visitors

A

1984: non visitors

Occupier + premises + non visitor MIGHT = duty owed in relation to personal injury (no protection to property of trespasser)

Occupier + premises = same as 1957

1 – Non visitor = on land with no invitation whose presence is unknown or if known objected

2 – Duty owed if x3
Occupier/D are aware of danger or have reasonably ground to believe that it exists
Occupier/D knows or have reasonably grounds (eg facts) that someone is in the vicinity or may come into the vicinity
Occupier/D reasonably expected to offer some protection (objective)
Standard of care
s1(4): care as in reasonable in all circumstances that entrant does not suffer injury on the premises by reason of the danger concerned

3 – Below standard
Same as general negligence: eg likelihood and gravity of harm etc
Court unlikely to have the same expectations as for visitors

Warnings
s1(5) duty may be satisfied if occupier takes all reasonably steps to give warning or discourage
Easier to satisfy for non visitors than visitors (eg physical barriers can be enough for non visitors)

Causation and remoteness: assume okay unless issue arises

Defences x2 (adapt case law)
Consent
Contributory negligence

310
Q

Occupiers liability – Exclusion or limitation classes

A

Exclusion or limitation classes — OLA 1957 (not OLA 1984 — the act is silent! But likely common law)

s2(1)
Restrictions x4

s(3) OLA 1957:
occupier cannot by contract exclude common duty of care
So where an occupier is bound by contract to allow visitors to use or repair premises, the duty of care owed to visitor cannot be excluded or restricted by contract

UCTA 1977 (B2B)
Cannot exclude death or personal injury from their negligence
Can only excluded other types of loss if term/notice satisfy reasonableness

CRA 2015
Between consumer and trader
Like UCTA cannot exclude death or personal injury from their negligence
Can only excluded other types of loss if term/notice is FAIR

Common law
Common humanity principle
Minimum legal standard of care which can never be excluded by agreement or notice

Would conscientious person with D’s defendants knowledge, skill, resources be reasonably expected to have done something which would have avoid the accident rather than rely on exclusion or restriction

311
Q

Product liability
Two aspects

A

Two aspects: Consumer Protection Act 1987 and liability negligence

CPA and negligence not mutually exclusive but if both succeed claimant cannot be compensated twice

312
Q

Consumer Protection Act 1987

A

CPA 1987
Strict liability regime: parties liable without showing fault on their part

Definition from s2(1)
– Product
– With a defect
– Causes damage
– Persons as 2(2) who are liable

313
Q

CPA 1987 – product

A

any goods or electricity
Includes any component or raw materials comprised in another product
Sections 1(2) CPA 1987

314
Q

CPA 1987 – Defect

A

Safety of product is not such as people generally are entitled to expect

So focus is at what people generally can expect: consumers perspective not manufacturer’s (does not matter that manufacturer took reasonable care!)

Higher standard than reasonable care!
A v National Blood authority

Relevant consideration include (s3(2))
Manner and purpose for which marketed
What might reasonably be expected to be done with it
Time of supply

315
Q

CPA 1987 – Damage

A

Damage
Death , PI, any loss or damage to property (CEL too but note use of property)

PEL not recoverable

Product itself not recoverable
s5(4): damage must exceed 275

Property is ordinarily intended for private use s5(3) = consumer protection (cannot recover for business as business cannot suffer personal injury or death)

316
Q

CPA 1987 – Persons liable for damage

A

Persons liable for damage s2(2)

The person who produced the product or held themselves out as the producer
Any person who has imported the product into the UK from outside the UK in the course of business

Producer
Manufacturer
Person who won/abstracted product (eg coal)
Person who carried out the process (neither manufactured or abstracted eg agriculture)

Supplier might be liable — If claimant asked details of producer within reasonable time and supplier failed to ID producer then claim can be brought

If more than one person liable: jointly and severally liable s2(5)

317
Q

CPA 1987 – defences

A

Defences x7: s(4)

a) Defect attributable to compliance with statute or retained EU regulations

b) Person proceeded against did not at any time supply the product to another

c) Otherwise than in the course of business (?)

d) defect did not exist in product at relevant time

e) state of scientific knowledge at the relevant time could not have detected it (limited no fault defence where fault is caused by limited scientific knowledge)

f) defect due to subsequent product that product is found in
Importance of relevant time s4!

s6(4) contributory negligence is also a defence

318
Q

CPA 1987 – method

A

Product? Defective? Damage? Identify people liable? Defences? Limitation?

319
Q

CPA 1987 – prohibitions + limitations

A

Any attempt to exclude or limit liability is prohibited s7

Limitation: claim must be brought within 3 years of later date
– Date injury and/or damage occurred
– When claimant became aware or should have become reasonably become aware
– 10 years after product was put into circulation by D: absolute defence

320
Q

product liability – Negligence (Donoghue v Stevenson case law)

A

Is a duty owed? Precedent! Manufacturer owes duty of care to user of product (not just end user/final purchaser) and manufacturer owes duty to a party that comes into contact with product (even if did not use or buy)

Breach, causation, remoteness, defences, remedies

General negligence: loss of product itself is PEL and generally not recoverable

Legal causation: where goods were going to be examined between manufacture and use by consumer then harm caused to consumer is not the manufacturer faults (NAI) yet there must be a reasonably probability of immediate inspection

321
Q

important note re psychiatric harm (actual, primary and secondary victim) AND PEL

A

don’t forget the rest of the cause of action: breach, causation, remoteness, defences, remedies